Wikipedia:Reference desk/Science: Difference between revisions

From Wikipedia, the free encyclopedia
Content deleted Content added
Spinningspark (talk | contribs)
→‎Centrifuge: a space elevator would be useful for that, too
Line 277: Line 277:
::::It might be easier to use a [[Gravitational slingshot]], though I couldn't say for certain as I don't have much expertise in this area. [[User:AlmostReadytoFly|AlmostReadytoFly]] ([[User talk:AlmostReadytoFly|talk]]) 13:18, 8 May 2008 (UTC)
::::It might be easier to use a [[Gravitational slingshot]], though I couldn't say for certain as I don't have much expertise in this area. [[User:AlmostReadytoFly|AlmostReadytoFly]] ([[User talk:AlmostReadytoFly|talk]]) 13:18, 8 May 2008 (UTC)
::::: You'd need a booster to get into a trajectory around another planet or you couldn't get the angles right from what I gather, so no savings. This is from an old Discover Magazine article [http://discovermagazine.com/2003/oct/breakheaven] ''"Meanwhile, mechanical engineer Stephen Canfield of Tennessee Technological University in Cooksville and others are investigating using a whiplike tether to hurl a probe from Earth orbit toward another planet, eliminating the need for a booster rocket. "The idea is to store up a lot of energy by spinning the tether. Then you release that energy by briefly capturing a payload on the tether's end," Canfield says.'' They also had another rotating system they proposed that tossed and caught loads until it reached escape velocity the concept drawing for it looked sort of like a ninja [[shuriken]] with only 2 prongs. (Can't find that one. I had filed it somewhere for later reading.) I admit I'm quite nebulous on the concept, too, but it must work out somehow. The only tether I know of that they tried in earth orbit fried because they hadn't thought they'd get that much juice from just dangling a wire and there was a problem with isolation. Can't figure out how they'd make use of the induction in the spinning tether but maybe the two are not related. You'd get less power in a lunar orbit, but you wouldn't have to deal with an atmosphere. [[Special:Contributions/71.236.23.111|71.236.23.111]] ([[User talk:71.236.23.111|talk]]) 21:56, 8 May 2008 (UTC)
::::: You'd need a booster to get into a trajectory around another planet or you couldn't get the angles right from what I gather, so no savings. This is from an old Discover Magazine article [http://discovermagazine.com/2003/oct/breakheaven] ''"Meanwhile, mechanical engineer Stephen Canfield of Tennessee Technological University in Cooksville and others are investigating using a whiplike tether to hurl a probe from Earth orbit toward another planet, eliminating the need for a booster rocket. "The idea is to store up a lot of energy by spinning the tether. Then you release that energy by briefly capturing a payload on the tether's end," Canfield says.'' They also had another rotating system they proposed that tossed and caught loads until it reached escape velocity the concept drawing for it looked sort of like a ninja [[shuriken]] with only 2 prongs. (Can't find that one. I had filed it somewhere for later reading.) I admit I'm quite nebulous on the concept, too, but it must work out somehow. The only tether I know of that they tried in earth orbit fried because they hadn't thought they'd get that much juice from just dangling a wire and there was a problem with isolation. Can't figure out how they'd make use of the induction in the spinning tether but maybe the two are not related. You'd get less power in a lunar orbit, but you wouldn't have to deal with an atmosphere. [[Special:Contributions/71.236.23.111|71.236.23.111]] ([[User talk:71.236.23.111|talk]]) 21:56, 8 May 2008 (UTC)

: An interesting feature of a [[space elevator]] is that, if you had one, you could also use it as a centrifugal launch platform: it has to extend some distance beyond [[geostationary orbit]] anyway, in order to provide a counterweight to the lower portion, so anything thrown off the top will get flung away from the Earth — the energy for this ultimately coming from the Earth's rotation. —[[User:Ilmari Karonen|Ilmari Karonen]] <small>([[User talk:Ilmari Karonen|talk]])</small> 12:44, 11 May 2008 (UTC)


== Pls, helping to answer this question !? ==
== Pls, helping to answer this question !? ==

Revision as of 12:44, 11 May 2008

Welcome to the science section
of the Wikipedia reference desk.
Select a section:
Want a faster answer?

Main page: Help searching Wikipedia

   

How can I get my question answered?

  • Select the section of the desk that best fits the general topic of your question (see the navigation column to the right).
  • Post your question to only one section, providing a short header that gives the topic of your question.
  • Type '~~~~' (that is, four tilde characters) at the end – this signs and dates your contribution so we know who wrote what and when.
  • Don't post personal contact information – it will be removed. Any answers will be provided here.
  • Please be as specific as possible, and include all relevant context – the usefulness of answers may depend on the context.
  • Note:
    • We don't answer (and may remove) questions that require medical diagnosis or legal advice.
    • We don't answer requests for opinions, predictions or debate.
    • We don't do your homework for you, though we'll help you past the stuck point.
    • We don't conduct original research or provide a free source of ideas, but we'll help you find information you need.



How do I answer a question?

Main page: Wikipedia:Reference desk/Guidelines

  • The best answers address the question directly, and back up facts with wikilinks and links to sources. Do not edit others' comments and do not give any medical or legal advice.
See also:


May 4

Blue and green eye color in Africans

Does anyone have information on a Klein Wollendorf Syndrome" When I attended the Univ. of Pittsburgh back in the 70's I was doing some research in color preference and I happened on this book Titled the Klein Wollendorf Syndrome which documented the occurences if bright blue eye color in sub saharan Africans. They also documented the occurences of people who were born with emerald green eye color and the white of the eye was yellow instead of white. The book included many pictures of the people with the eye colors. The authors said widespread documentation was difficult because in previous generations many of the children born with the eye colors were killed at birth because they were considered cursed. I have lost my bibliography so I don't have a publisher or dates. If any one has any information, please respond. —Preceding unsigned comment added by Chaniatreides (talkcontribs) 06:06, 4 May 2008 (UTC)[reply]

I expect you are referring to Klein-Waardenburg syndrome (OMIM: 148820), also known as Type III Waardenburg syndrome. It results from a mutation in PAX3 and, among other things, manifests with melanocyte deficiencies (hence the eye colours). A PubMed search with the term should provide all the info you need. By the way, the book you refer to is probably: Jenni Soussi Tsafrir, Light-eyed Negroes and the Klein-Waardenburg Syndrome, London, Macmillan, 1974, ISBN 0333140729. Rockpocket 06:49, 4 May 2008 (UTC)[reply]
We really do have a page on everything! ;-) Waardenburg syndrome--Lisa4edit (talk) 09:46, 4 May 2008 (UTC)[reply]

A four-hour day

I'm wondering just how life might be different if the days were 4 hours long (defined as 3600 seconds, where a second is defined in relation to the speed of light as is current). Would we still have the same sort of circadian rhythms? -mattbuck (Talk) 06:41, 4 May 2008 (UTC)[reply]

Hold on, are you talking about an actual day (i.e. the earth's rotation around the sun) being 4 hours or some random definition of a day as being 4 hours?. If it's the former then our circadian rhythms would presuambly have evolved in such an environment and would therefore be best adapted to a 4 hour day. If the later, well it would be irrelevant out circadian rhythm would still be best adapted to a 24 hour day Nil Einne (talk) 07:44, 4 May 2008 (UTC)[reply]
There was a study a few decades ago (before the Internet). They put some subjects in a bunker, cut off from any outside light source. They were not allowed any clocks and could choose when to switch on their electric lights and declare it "daytime" and when to sleep equating "nighttime". Most of the subjects adjusted to a "day" longer than 24 hrs. (I'll see if I can find that mentioned somewhere.)--Lisa4edit (talk) 09:36, 4 May 2008 (UTC)[reply]
Seems to be mentioned on Circadian rhythm Nil Einne (talk) 12:43, 4 May 2008 (UTC)[reply]
That was it, the results were refuted I see. Thanks for pointing that out. I had apparently never read that article far enough. --Lisa4edit (talk) 16:11, 4 May 2008 (UTC)[reply]
An experiment with an 8-hour "day" has been ongoing for several hundred years. Large numbers of humans have been placed in artificial environments and required to be active for four hours and then be (more or less) at leisure or sleeping or four hours, sometimes for years at a time. Some humans adhered to this schedule from childhood until (relatively) old age. I refer of course to the watch system of the (british) navy and other navies. -Arch dude (talk) 23:33, 4 May 2008 (UTC)[reply]

What would a YY chromosome person look like?

Is it possible that a person has YY chromosomes? Is it possible to create a person from two Y chromosomes from a male? If not, just for fun what do you think a person with such chromosomes would act like or look like? —Preceding unsigned comment added by 70.219.182.238 (talk) 07:46, 4 May 2008 (UTC)[reply]

It is possible for humans to have YY, but only in combination with a X, giving them XYY syndrome. XYY people tend to have slightly lower IQs an increased risk of learning difficulties. You can even get people with XYYY and XXYY syndrome, though they tend to have more severe difficulties.
The human Y chromosome is tiny and contains very few genes. It largely contains those genes necessary for becoming male, SRY for example. In humans the X contains lots of genes many of which have nothing to do with sex determination and are essential for life. Examples include a gene to make blood clotting proteins. So, at least in humans, everyone needs at least one X or else they do not develop. Therefore a person with YY sex chromosomes would probably look like a jelly bean, before aborting in utero. Rockpocket 08:26, 4 May 2008 (UTC)[reply]
Is it possible for the Y chromosome to be genetically engineered to be able to hold the necessary genes, making it possible for just YY human beings? —Preceding unsigned comment added by Jue2 (talkcontribs) 08:34, 4 May 2008 (UTC)[reply]
Well, currently the technology is not available for humans, but it would probably be feasible to do that in experimental animals. That would be the functional equivalent of putting the SRY region (the gene that makes you a "male") into an X chromosome. This occasionally happens in humans due to chromosomal translocation. The result is individuals that have female haplotypes but male phenotypes. The only major problem is that those males are infertile. Speculating now: If you could pick all the essential genes on X on put them on a Y (lets call that Y*) you would have all the genes required for life. But YY* people would still have problems, because they would have two doses of the genes on the Y chromosome, including SRY. That would result in the same difficulties seen in XYY males. Females get around the dosage problem by lyonisation, but males could not do that. They way around this would be to engineer a Y*O male (with just one modified Y chromosome). Would that male survive and be healthy. Who knows?
By the way, it turns out that YY embryos abort before implantation, thus in humans a YY would only last a week or two so after conception before aborting. Rockpocket 09:03, 4 May 2008 (UTC)[reply]
So it would be easier for women to exist without men, than men to exist without women. That is a very scary thought. What gene in the Y chromosome decides gender? The Y chromosome has few few genes compared to the X so, why couldn't engineers move that gene in the Y to the X chromosome? Jue2 (talk) 09:22, 4 May 2008 (UTC)Jue[reply]
Yes. Womankind could theoretically exist without men when human cloning becomes a reality (because all you need then is a womb in which to carry the XX clone). Why else do you think all those male dominated societies have outlawed human cloning ;)? As I said above, the principle male sex determining gene in humans is SRY and nature occasionally moves SRY into the X chromosome by unequal chromosomal crossover, resulting in XX male syndrome. Rockpocket 10:01, 4 May 2008 (UTC)[reply]
I find the thought amusing. A species without females--now that would be scary. Imagine Reason (talk) 02:44, 5 May 2008 (UTC)[reply]
So check out New Mexico whiptail and be afraid, be very afraid :-). 87.81.230.195 (talk) 08:08, 30 March 2009 (UTC)[reply]

Time Travel is Impossible

Time travel just can't happen, otherwise we already would know about it. Not to mention the paradoxes it would cause. Travel back in time and kill my mother before I was born. I guess my question is, "does anyone believe time travel is possible? If so please explain it to me." —Preceding unsigned comment added by Jue2 (talkcontribs) 09:30, 4 May 2008 (UTC)[reply]

Our Time travel article may be of interest to you. I suppose that there is at least someone who believes that it's possible. --hydnjo talk 10:47, 4 May 2008 (UTC)[reply]
There are consistent theories that allow for time travel, but they're quiet restrictive. For example, you can't generally travel back to before the time machine was created. Exactly how it would work with paradoxes, I don't know - I expect you would simply find that you can't change anything because you were there doing exactly the same thing "the first time round", and just didn't see yourself. (The main alternative theory is that you would travel back to an alternate reality and change that one, rather than your own.) --Tango (talk) 13:09, 4 May 2008 (UTC)[reply]
The expression Time travel usually is understood to only move the traveler in time. If you'd allow travel to another destination (or another universe) you remove quite a few roadblocks. Moving a person also makes things a whole lot less likely. If you could figure out how to send the information to assemble a person (as proposed for SciFi "transporters") things would also become easier. Traveling to the past I'd still think highly unlikely. But I keep encountering things that everyone used to think were impossible. ( see "Humpty Dumpty" for a recent event.) Lisa4edit (talk) 15:13, 4 May 2008 (UTC)[reply]
Time travel is not imposssible, I have already told you that tomorrow - don't you remember? SpinningSpark 15:40, 4 May 2008 (UTC)[reply]
The real question is, "Can an all-powerful god alter the past?" Imagine Reason (talk) 22:37, 4 May 2008 (UTC)[reply]
Why would an all-powerful god need to alter the past if he was all powerful and all knowing? I've managed to go back in time before. It was easy, but really noisy as well. bibliomaniac15 Do I have your trust? 03:58, 5 May 2008 (UTC)[reply]
I was only concerned with the ability to change the past. An all-powerful god may not be all-benevolent. He may well decide to play dice with our lives and change our past on a whim. Imagine Reason (talk) 15:56, 5 May 2008 (UTC)[reply]

Time travel paradoxes vanish when parallel universes are considered. Amelia Earhardt had no role as an aviatrix in World War 2 in the parallel universe where she vanished in the 1930's. Judge Crater never got appointed to the U,.S. Supreme Court in the parallel universe where he vanished mysteriously. Charles Lindbergh Jr. never became President of the United States in the parallel universe where he was kidnapped and killed as a toddler. In the parallel universe where a time traveller altered history the alteration would be accepted as part of that time stream's history. Edison (talk) 03:44, 5 May 2008 (UTC)[reply]

Another argument for the impossibility of time travel is that if there were ever in the future a year when time travel became possible, then a traveller would have announced his presence in our time or in our past, and we would all know that time travel was possible. In the parallel universe model of reality, there could be many time streams where time travel has been shown to be possible, just not in this one (yet). Consider then whether it is ever the case than some supposedly deranged individual is picked up and tells the police/psychiatric personnel that he is from the future. Would we even hear about it? If the time traveller were not incautious enough to announce his presence that way, but were only allowed to arrive with no equipment, clothing, or other physical evidence inconsistent with this time, and screened to act as a scientific observer, he might pass unnoticed (but certainly not without having caused this time stream to be altered in some way). Even such a prudent observer would inevitable get the last place on the elevator, causing someone to miss his medical appointment, causing their cancer not to be detected in time. Or he might take a seat in a restaurant which otherwise would have been occupied by one half of a couple who therefore did not meet, marry, and give birth to a world leader, as in the Butterfly effect. Edison (talk) 12:40, 5 May 2008 (UTC)[reply]
That whole issue with nobody coming back and telling us they're from the future disappears if you impose the "you can't travel back to before the time machine was created" restriction, which I believe is imposed by most (if not all) ideas for time machines based on General Relativity. --Tango (talk) 12:59, 5 May 2008 (UTC)[reply]
It is easire to envision time travel where the "booth" is the arrival point, because the traveller thereby does not arrive inside another person or a tree, but there are probably ways around the claimed general relativity prohibition. Edison (talk) 18:54, 5 May 2008 (UTC)[reply]
the question of time paradox is largely contingent on your theory of what time is to begin with. The logical answer to the most basic time paradoxes is "you couldn't because you didn't" or "it's possible because you did." Basically, the argument is that "changing the past" is a meaningless phrase; because the past already occured, it can't be changed. Think Twelve Monkeys or the first Terminator movie. --Shaggorama (talk) 09:24, 8 May 2008 (UTC)[reply]

Does any one know what this is

Hi I would like to know what article this could go in. I found this in the Blue Mountains New South Wales. I may even have better images of this

. --User:Adam.J.W.C. (talk) (talk) 10:38, 4 May 2008 (UTC)[reply]

I guess you can't wait till it's matured into a moth. :-) [1] narrows it down to LYMANTRIIDAE or "Tussock Moth" "are so-named because the Caterpillars of many members have four long dense dorsal tufts of hair. Many also have other hair pencils, and also two coloured dorsal glands on abdominal segments six and seven." --Lisa4edit (talk) 12:03, 4 May 2008 (UTC)[reply]
If you have lots of time you might find it here [2]Lisa4edit (talk) 12:41, 4 May 2008 (UTC)[reply]
I forwarded your identification request to Wikipedia:Wikiproject Lepidoptera. If they don't contact you soon, try going over to the project page and picking out a few contributors to contact directly in case the project page doesn't get regular visitors. --Shaggorama (talk) 09:38, 8 May 2008 (UTC)[reply]

Constructing a lunar orbital station by firing materials from the moon

My dad is writing a science fiction novel and has a Clarkesque desire for realism. He's described a situation to me but the maths is pretty complicated and well beyond my knowledge. The figures used below are our guesses at plausible numbers, if these guesses are unreasonably high or low for the practicalities of the situation, please say so and we can revise. The situation is as follows:

Suppose we are constructing a space station in an equatorial orbit around the moon at a height around the order of 300km. It is being constructed from materials mined on the moon, and they are being launched up to meet the station by a linear accelerator on the moon's surface. The materials are launched in ~200 tonne payloads and the accelerator can impart acceleration no greater than, say, 4 G. The accelerator can be built on an incline but if it can be laying flat to the ground this would be preferable. Suppose the construction platform begins with a mass of 1000 tonne, this will of course increase with each payload and the added mass and momentum will obviously alter the orbit of the platform itself. The payloads are capable of minor course alterations but aren't equipped with large rockets or anything. The payload "catching" mechanism on the construction platform requires payloads travelling no faster than 0.1 m/s (or something else plausible). Is this situation workable, and what kind of length / launch speed / angle / timing are required for the launcher?

It seems to me like we'd need the payload to reach the platform at the payload's apoapsis (highest point) and the platform's periapsis (lowest point). However, the fact that the platform will be changing orbit with each payload makes the whole thing too difficult for me to see how to approach since I don't know enough about orbital mechanics to know how the orbit will be changing. Any ideas? Thanks, Maelin (Talk | Contribs) 10:43, 4 May 2008 (UTC)[reply]

The orbit of the platform won't change - the mass of a satellite doesn't feature in the equations for it's motion (it cancels out). As long as it stays significantly less than the mass of the moon, you don't have to worry. There will be a transfer of momentum when you catch each payload, but for the catching to work you'll need them going quite slowly relative to each other anyway, so I think you can ignore that. Have you read out article on mass drivers? I think that's what you're describing. It doesn't have many technical details, but it might help a little. What you probably want is the platform to be in a circular orbit, and then the payloads launched at a pretty shallow angle (horizontal would be fine). I think you would then need the payloads to fire small rockets at the appropriate point to circularise their orbit slightly lower than the platform, then when they catch the platform up they fire again to lift them up to it and reduce their relative velocity so the catching can work. I'm not entirely sure about how launching things into orbit with mass drivers works - most applications I've seen seem to involve giving them escape velocity, but putting things into orbit should be possible, I'm just not sure how big a rocket you would need for the circularisation (certainly smaller than you would need for an actual launch). --Tango (talk) 13:21, 4 May 2008 (UTC)[reply]
If the space station is in any particular orbit, and the momentum of a 200 tonne addition is added to it, without firing a rocket to match orbits, the orbit of the combination will inevitably be different from that of the space station initially. I see two non-rocket solutions. The ugly one is that the space station starts in its desired orbit. Each time a payload docks with it, the orbit would change somewhat. To avoid this, the payload could fire off a small part of its mass at a high velocity (spring? cannon? electromagnetic thruster? compressed gas?) This could have the same effect as firing a thruster. The downside would be that these correction ejections would then be flying around the moon in orbit, unless they were fired off with escape velocity, ideally to ultimately impact the Sun or perhaps some planet the Moon dwellers didn't like (Heinlein had a story of throwing rocks at the Earth from the Moon, as I recall). The elegant solution is to have the initial part of the space station in an orbit which differs from the final desired orbit in a carefully calculated way. Then each time it catches a payload, the orbit of the combination moves toward the desired orbit. When all the payloads have been caught, the orbit has shifted exactly to the desired orbit. (Difficult to calculate? Aw gee, orbital mechanics is HARD!) Edison (talk) 13:45, 4 May 2008 (UTC)[reply]
Requiring a relative velocity of less than 0.1 m/s sounds like matching orbits to me. I expect small station keeping thrusters on the platform would be enough to correct for any relative momentum not shed before catching. --Tango (talk) 13:55, 4 May 2008 (UTC)[reply]
There is a cumulative effect here. One payload you might get away with if it is small and slow, but if you are trying to significantly increase the size of the station, eventually you are going to have to correct the orbit in some way. Fuel for thrusters can only arrive through the payload which I think is what the original concept is trying to avoid. Don't forget that any payload launched ballistically from the surface is going to end up in an orbit which will intersect its starting point (ie it is going to try and fly through the mass of the moon). Therefore there MUST be a correction to the payload by some means to insert it into a working orbit. If this is done by the station catching it, it will inevitably impart a momentum to the station in an undesirable direction. You might be able to do something clever with tethers though. SpinningSpark 14:17, 4 May 2008 (UTC)[reply]
As long as the exhaust velocity is greater than the relative velocities of the payloads when caught, bringing fuel up shouldn't be a problem. The OP said the catching mechanism required the relative velocity to be less than 0.1m/s, so station keeping should be easy. It's slowing the payloads down (or speeding them up, depending on perspective) that's hard - tethers could work, and are certainly more interesting to read about than rockets. --Tango (talk) 14:27, 4 May 2008 (UTC)[reply]
You could employ a tether or [3] might work. You could at least use it to get rid of some need for corrections as far as I understand it. This is way out of my league but maybe s.o. else can develop the idea, or say why it won't fly. Lisa4edit (talk) 15:57, 4 May 2008 (UTC)[reply]
It might be possible with 2 accelerators 90 degrees apart. They would cancel each other out. Catching the payloads wouldn't be a problem either. Each accelerator sends one payload into orbit. the 2 payloads collide and the resulting single payload would have a very low velocity relative to the station. I doubt that its really feasible but it should good enough for a science fiction story.Em3ryguy (talk) 19:13, 4 May 2008 (UTC)[reply]
Why would they collide? I can't see a way to get them to be in the same place at the same time. --Tango (talk) 19:41, 4 May 2008 (UTC)[reply]
"The payloads are capable of minor course alterations". If their orbits are in the same plane then they must cross someplace. Also the collision doest have to be anywhere near the station. After the collision the resulting single payload can then drift slowly to the orbiting station on the other side of the moon. Half of the payloads will do so in a slightly lower orbit and half in a slightly higher orbit. As a result, the orbiting stations orbit would not be changed.Em3ryguy (talk) 20:38, 4 May 2008 (UTC)[reply]
Their orbits will cross, sure, but they won't both reach the crossing point at the same time. Also, the term "orbit" is a little misleading - the payloads will crash back to Earth before completing one complete orbit if they don't do something to circularise it. --Tango (talk) 20:42, 4 May 2008 (UTC)[reply]
They dont have to actually collide. They can use many mile long tethers to simply orbit one another. We are talking about fiction here, yes?Em3ryguy (talk) 21:42, 4 May 2008 (UTC)[reply]
We're talking about hard science fiction - the idea is that it is actually physically plausible (given sufficiently advanced engineering). There are all kinds of things you can do with tethers, but I'm not sure what you're suggesting... --Tango (talk) 22:19, 4 May 2008 (UTC)[reply]
The collision of the 2 payloads circularizes the orbit. Thats the whole point.Em3ryguy (talk) 02:33, 5 May 2008 (UTC)[reply]
The idea is ridiculous, even if the launch was synchronised perfectly so that the two payloads did meet at aposelene the collision would not do anything to help match orbits with the station. It will just result in crushed payload canisters. A collision does not add energy to the system so it cannot possible achieve the orbit desired. As for the tether, if you launch at 90 degrees, the two payloads are going to end up tens of thousands of miles apart before they meet again - that's a lot of string to reel back in!SpinningSpark 02:44, 5 May 2008 (UTC)[reply]
90 degrees apart in longitude. they are 1000km apart. After the collision the orbit of the combined object should be circular.Em3ryguy (talk) 05:50, 5 May 2008 (UTC)[reply]
I may possibly owe you an apology for an AGF failure, that actually works, at least with one sample orbit calculation I did in an attempt to prove you were talking nonsense. However, the idea is still ridiculous. The orbital radius required is 3x105. The mass of the moon is 7.35x1022 times G=6.67x10-11 gives μ=49x1011. Plugging all that in to the vis-viva equation gives an orbital velocity of 4000 m/s. Assuming the original launch is in a highly elliptical orbit so that 1/a > 0, then the velocity of the payload prior to impact is of that, ie 5700 m/s. So, you have two objects travelling at 5700 m/s, one in a rising orbit, one in a falling orbit, meeting at approximately right angles (closing velocity is 8000 m/s). The collision is required to be elastic, a) because loss of energy will lose orbital speed and fall back to the moon and b) an inelastic collision will destroy the payloads. You therefore need some kind of catcher mechanism on the payloads. I cannot even begin to imagine a mechanism that would work at those speeds without, as I said before, resulting in crushed, not to say vaporised, payloads. SpinningSpark 09:27, 5 May 2008 (UTC)[reply]
I've made a silly mistake in the calculation above. The orbital radius is not 3x105, it is 3x105 plus the radius of the moon which comes out to 2x106. Luckily for the world and the environment, I do not launch rockets professionaly. All the velocities are around 40% of that stated. The closing velocity is 2400 m/s (4800mph). Still a lot more than can be handled by a catcher but launching in a less elliptical orbit would considerably reduce the closing velocity. The limiting circular orbital case has zero closing velocity. I am beginning to change my mind, there may be some mileage in this idea after all. SpinningSpark 11:04, 5 May 2008 (UTC)[reply]
You are using absolute velocities. You have to use relative velocity. they are both in nearly circular orbits so their relative velocity should be small. The simplest way to estimate the relative velocity would be to determine their average absolute orbital velocity then calculate the angle at which their orbits cross.Em3ryguy (talk) 11:27, 5 May 2008 (UTC)[reply]
Is my statement that the limiting case of circular orbits has zero closing velocity not the same thing? For the calculation above of highly elliptical orbits the closing velocity is correct and they will meet at right angles ie at the individual absolute velocities. SpinningSpark 12:15, 5 May 2008 (UTC)[reply]

Ignore the station for a moment and concentrate on each payload. A payload is ~200 tonne. If yo uuse a mass driver on the moon's surface and have only small thrusters to use later, the mass driver cannot place the payload onot a circular orbit. The payload goes ballistic as it leaved the mass driver, which means that it is in a lunar orbit that will come back to the same point (i.e., the mouth of the mass driver) unless quite a bit of additional energy is applied somewhere, and the thrusters you describe cannot do this. So you need to find a way to impart a supstantial amount of energy to your payload after it leaves the mouth of the mass driver, If your mass driver is aimed horizontally from the top of a very high peak, You will have an entire lunar orbit in which to add energy before the payload hits the mass driver, and you will not need to add very much energy. to get into a non-intersecting orbit (an extremely low lunar orbit.) From there, you can continue to add energy at a slow rate until you reach the 300Km orbit. The most cost-effective way to add energy will depend on your scenario. One possibility is to place a small mass driver on the payload and shoot a few rocks backwards to get into extremely low lunar orbit, and then use an ion drive for the rest of the (slow) trip to 300Km. You would have a many payloads in transit. Salvage the mass driver and ion drive when the payload reaches teh station and return them to the surface to use on the next payload. -Arch dude (talk) 23:16, 4 May 2008 (UTC)[reply]

It is possible to do it without circularising the orbit, you just have to fire the payload at sufficient speed that it's apoapsis meets the platform. You then need some way to catch the payload, and it will be travelling at a relative velocity significantly greater than the 0.1 m/s the OP was after. You would then need quite significant station keeping efforts to stop the the platform moving. I think the end result is that you either need to relax the requirement on the catching velocity, or relax the requirement on how much thrust the payloads can generate (using rockets, ion drive, tethers, whatever). Either would work. --Tango (talk) 23:46, 4 May 2008 (UTC)[reply]
For the station to "catch" the payload, it must deliver enough energy to the payload to move the payload from a lunar-intersecting orbit to a 300Km circular orbit. A passive "catch" will merely average the momentum of the station with the momentum of the payload. After many "catches" the station will intersect the moon if the payloads have the same momentum vector. Aha! is there a set of mass driver positions on the moon's surface that can impart a zero average momentum on the station? Each payload will need to contribute enough total energy to circularize its own orbit, and the station will need to store the extra energy, so we do not have a zero-velocity intercept and we need an energy-transfer mechanism at the station such as another mass driver acting as a catcher. -Arch dude (talk) 00:45, 5 May 2008 (UTC)[reply]
A rotovator can solve the <0.1m/s problem. Here's a NASA study into how it might be done from Earth [4]. In the case of the moon you might not need to launch at all, just pick it up off the surface!SpinningSpark 03:10, 5 May 2008 (UTC)[reply]
I don't understand... if each payload can circularise its own orbit, we don't have a problem... --Tango (talk) 12:56, 5 May 2008 (UTC)[reply]
The constraints of the original question imply a ballistic launch with just the capability for "minor corrections". Circularising the orbit in nearly all launch scenarios is a major delta-v event and has gone outside the scope of the question as stated IF achieved from the payloads own on-board resources. Anyway, I was only trying to think outside the box and give an answer more interesting than rockets. Picking up the payload directly from the surface with a rotating tether certainly fits that bill imho. SpinningSpark 18:08, 5 May 2008 (UTC)[reply]
I was replying to Arch dude, not you. --Tango (talk) 21:53, 5 May 2008 (UTC)[reply]
No one has looked at the viability of the proposed launch system yet. Allow me to have a go. From my previous calculation above (both the flawed and the unflawed versions) we have,
and it is going to be convenient to use;
The radius of the moon is 1.7 x 106 For an orbit as near circular as possible to give good match with the station a near horizontal launch is needed that just makes it to the 0.3 x 106 height required. The semi-major axis of this orbit is;
and the launch velocity required is given by;
The length of the linear accelerator required can be determined from;
since the accelerator is specified to be 4G acceleration
and,
You need to build an accelerator around 40km long to make this work. SpinningSpark 12:07, 5 May 2008 (UTC)[reply]
So get those solar powered bulldozers to work piling up lunar soil to make the launch platform. Edison (talk) 12:44, 5 May 2008 (UTC)[reply]
40km sounds doable to me, given the kind of future engineering we're talking about. I think the accelerator is possible. --Tango (talk) 12:53, 5 May 2008 (UTC)[reply]
If it left the lunar surface tangentially at a point and continued in a straight line 40 km. how high would the terminus have to be? This would determine, along with the angle of repose of lunar soil, the mass which would be needed to construct it, unless it was built like a roller coaster out of a trellis structure or a series of towers. Edison (talk) 18:52, 5 May 2008 (UTC)[reply]
According to the back of my envelope, the end would be 460m high. In lunar gravity, that shouldn't be too hard to achieve. I'm not sure the track needs to be level, though, it may be able to follow the surface without too much difficulty. Or, you could just find somewhere with an appropriately angled hill (or close enough, and make just small adjustments). --Tango (talk) 19:27, 5 May 2008 (UTC)[reply]
40km is no big deal -- we build structures on that scale all the time. Based on similarly large structures (roads, railroads, etc.), assuming you had enough manpower, it would take about a year to build. --Carnildo (talk) 21:39, 5 May 2008 (UTC)[reply]
Building it straight-line (ie tangential to the curved surface of the moon) is uneccessary. It is only the angle at the point it leaves the accelerator that counts. It could be built on a geodesic (lunadesic?) with just the final section angled to the desired trajectory - like an aircraft carrier deck. However, you may not even want to do this depending on the orbit you are trying to achieve and the payload recovery scheme adopted. A flatter track gets a less elliptical (more circular) orbit and it may only be necessary to angle it up sufficiently to clear obstacles like moon mountains. SpinningSpark 06:23, 6 May 2008 (UTC)[reply]

This is the dad who asked the question - it's over thirty years since I did physics. Lunar orbit velocity is about 1820m/s, so I see the problem as horizontal and vertical components. Horizontal component to achieve is 1820m/s, and vertical component needs to be such that lunar gravity of 1.622m/s/s will decrease it to zero at 300KM height. Thus, a small amount of force should then be able to circularise the orbit as the payload is already at orbital velocity. But I am not sure if my logic is correct. I also can't recall how to calculate the vertical component, and thus the angle of launch and the launch velocity and the length of the launcher. The 4G (i.e. 40m/s/s) is a nominal figure. Thanks for the comments guys. —Preceding unsigned comment added by 192.158.61.140 (talk) 06:28, 7 May 2008 (UTC)[reply]

I'm not sure you can just resolve it into components like that, since which direction is "horizontal" will change with time (due to the curvature of the Moon's surface). I think the conventional way to put something in orbit using a mass driver would be to launch (almost) completely horizontally and then, at apoapsis (greatest distance), fire rocket motors (or use any other form of propulsion) to circularise the orbit. My back of the envelope calculations show that the required delta-V for the launch would be around 1.8km/s and the required delta-V for the circulisation would be around just 0.1km/s. I expect you can afford to just slap a rocket motor to the payloads and get your 0.1km/s, it seems pretty small in comparison to the energy you're using in the initial launch (although that energy doesn't need to be lifted, which obviously helps quite a lot). --Tango (talk) 01:57, 8 May 2008 (UTC)[reply]

The Solid

Hello. I dissolved a solid into water. I added silver nitrate and the solution turned milky white. I added barium nitrate and the solution did not change. I added potassium thiocyanate and the solution turned reddish brown. I know that I am not allowed to post homework questions but I guess that the solid is a salt like sodium chloride. What is the solid? Thanks in advance. --Mayfare (talk) 17:15, 4 May 2008 (UTC)[reply]

"I know that I am not allowed to post homework questions[...]What is the solid?". Oh come on now...at least try posting your reasoning in detail and asking someone to see if you are doing it correctly and getting the right answer. DMacks (talk) 19:36, 4 May 2008 (UTC)[reply]
You will probably want a table of solubilities. Your textbook may provide a simple one; we have Solubility table which may be a bit overwhelming. Check whether each combination of the ions present in solution will precipitate. Nimur (talk) 16:31, 5 May 2008 (UTC)[reply]

gf has atopy

Hey. I like my girl, and I'm attracted to her, but she has pretty bad atopy all over her body and it's a real turn-off. It's hard because I can't be honest about it, and here (Japan) there are a lot of people with atopy but I'm not sure if they give them proper treatment because they have a tendancy (the country and the people do) to be scared away from stronger drugs. That means that she might be stuck with it for life. I don't know what to do because it's really hard for me when we get intimite and stuff, but I don't want to be a heartless asshole to her. Any advice? Heh. 218.229.72.166 (talk) 20:28, 4 May 2008 (UTC)[reply]

I assume you're referring to eczema, in which case I don't think we can help you. I don't think there is anything non-medical you can do about it, and we can't give medical advice. I suggest you talk to her about it and suggest she get treatment, if she isn't already. As for what treatment to get, I'll leave that to the doctors. --Tango (talk) 20:45, 4 May 2008 (UTC)[reply]
Yeah eczema. The problem is she is getting treatment, or at least I'm pretty sure she is. It's a very common to see people with eczema around Tokyo (from what I can tell); I think it has to do with genes and the environment that they are forced to live in. I think it also has a lot to do with the treatment they receive here. There are loads of medicines that you can get in the west (let's say the US) that you can't get here; most people won't touch birth control pills, Tylenol is very hard to come by (and the most of the alternatives suck), and even stuff like protein supplements are incredibly weak here. Original research of course. Hmmm... 218.229.72.166 (talk) 21:09, 4 May 2008 (UTC)[reply]
It can sometimes help to specifically ask the doctor for something stronger. They may be reluctant to suggest it, but might still prescribe it if asked. --Tango (talk) 22:16, 4 May 2008 (UTC)[reply]
I guess you've already read Atopy, Eczema and Atopic dermatitis. Our Allergy article may also hold some clues. Treatment varies by country and individual physician, but no one has come up with a "cure all" or "magic bullet" yet. This is not a case of one place having a cure and patients in the other not having access to it. Individual causes and results vary and some treatments like Corticosteroid can turn out to be worse than the original condition. Avoidance of the allergic trigger is your best bet. That may be easy, like a change in diet, switching detergents, avoiding certain materials in clothing and bedding, or it may be impossible if the trigger is ubiquitous. What is beneficial to some (e.g Omega-3 oils, oatmeal cream) can be a trigger to others (sea-food allergy, grass allergy). Finding a trigger can be a lengthy process. Rotation diets that avoid related food families for 2 weeks in succession could help narrow down food triggers. (OR: that worked for my cousin who had tried everything including cortisone and psychotherapy). Long holidays in as different an environment as you can find can also aid in narrowing things down. Good luck. --71.236.23.111 (talk) 23:06, 4 May 2008 (UTC)[reply]
Careful - you're bordering on giving medical advice there. We can't do that. If the OP wants medical advice, he needs to go to a doctor. --Tango (talk) 23:33, 4 May 2008 (UTC)[reply]
Sorry, my bad. I had not intended to. Despite the fact that it very rarely happens we didn't have any page on rotation diet or the like (maybe I just haven't found it yet under a different heading?). What it so puzzling to some people who are new to allergies is that there are no clear answers. It's not like some other conditions where "If A, apply B" gets comparatively consistent results. That's why you're way ahead in the game once you've identified the triggers. I certainly did not want to give any medical advice. I was merely trying to caution against second guessing the lady's physician too hastily, just because there are no immediate results. Sorry for not phrasing it less ambiguously. 71.236.23.111 (talk) 04:39, 5 May 2008 (UTC)[reply]
I can't give any medical advice (not just because it's not allowed, but because I don't have any to give). However, I can give relationship advice, and however you choose to deal with it, I would suggest you don't just try to ignore it. If it really bugs you now, it's not likely to bug you any less over time. You don't have to be a heartless asshole to say -- nicely and without sounding like you're pressuring her into anything, and certainly not when you're about to be intimate -- that hey, honey, I think they might be able to do something about the eczema. If the condition is treatable, then you're both suffering from it for no good reason; I doubt she's enjoying it either. And when I say nicely? I do mean nicely. "You should get that looked at," for example, doesn't qualify. "I hate to say this, but that really turns me off," doesn't, either. "You know, I just read somewhere that the new drugs for treating that type of skin are really efficient, have you thought about trying them out?" would probably work better. I mean, if she's the type that is going to absolutely take offense no matter what, then it'll happen, but at least you can minimize the damage. I mean, this is the type of stuff people have to learn to talk about in relationships anyway, so you're going to have to bite the bullet sooner or later -- if not on this, then on something else of a comparable nature... -- Captain Disdain (talk) 01:43, 5 May 2008 (UTC)[reply]
your relationship probably won't last very long if you are "turned-off" by your partners body. If she's receiving treatment, then the problem is really yours, not hers. If you can't learn to adjust, or at least be honest with her about how you feel (despite potentially activating her insecurities about her disorder) then you are probably never going to be that happy with the realtionship. Consider at least confiding in a friend about this or if you have one, talking to a therapist. --Shaggorama (talk) 09:44, 8 May 2008 (UTC)[reply]

The science of baseball

Please inform the intellectual side of my brain about the science of baseball. The physics of throwing fastballs, the advantage of a pitchers mound, how atoms play a part. Etc. I hope it isnt too far out to suggest quantam mechanics, and/or 'the butterfly effect" will affect the outcome of the game. If it weren't for page space, bandwidth, etc I would only halfway jokingly ask you to break down a typical game atom by atom :)--Baseball and and and Popcorn Fanatic (talk) 21:44, 4 May 2008 (UTC)[reply]

Quantum mechanics will have no noticeable effect on baseball - it only has a affect on very small scales, eg. individual atoms and smaller, a baseball is much larger than that. Chaos theory (the butterfly effect) may have a small impact, but I doubt anything particularly significant. Newtonian mechanics should be all you need. You might find our Trajectory article interesting. Aerodynamics also plays a role - particular if you put spin on the ball, as I understand it. If you want to get down to the atomic level, the reason the ball bounces back when it hits the bat (rather than going straight through - matter is mostly empty space with just a few tiny particles every now and then, so there would be plenty of room for the ball to go through the bat) is due to the electrons repelling each other due to electrostatics (it's probably much more complicated than that - it's not my field, but I think that's the basic idea). If you have any specific questions about a particular part of the physics of baseball, feel free to ask, otherwise I'm not sure what else to say. --Tango (talk) 22:13, 4 May 2008 (UTC)[reply]

Some people ("grr...mother!") think I read too much into this game. When I say I want to know everything, I mean EVERYTHING! Every player's RBI & slugging percentage. Every players on base history all the way back to college. Even the molecular structure of the beer consumed by the drunk standing next to me (soon to be vomited on my shoe). Whats the psychology behind him? I'll bet he treats his kids bad.

I also want to know where I can get information on how all MLB teams chose thier uniforms.We all like vacuum cleaner physics and other trivial science stuff. That's my argument here.--Baseball and and and Popcorn Fanatic (talk) 22:35, 4 May 2008 (UTC)[reply]

Googling for "The science of" baseball gets you several thousand places to look. Please feel free to ask for specific information or explanation. Otherwise it might be easier to define which wikipedia pages are not likely to have much that in some small way affects some aspects of the game or event of baseball. Another way to go is to start with one aspect like e.g. Newtonian mechanics, Beer or Decision making and then work your way to the pages that link there. Happy studying. 71.236.23.111 (talk) 23:34, 4 May 2008 (UTC)[reply]
Doc Edgerton actually did some work studying baseball physics. Check it out. --98.217.8.46 (talk) 02:03, 5 May 2008 (UTC)[reply]
Seems to me that you're looking for Adair's The Physics of Baseball, ISBN 0060084367. -- Coneslayer (talk) 11:45, 5 May 2008 (UTC)[reply]
Thus far, physics has been addressed, but the questioner asks for the science of baseball. What about biomechanics, sports medicine, metabolism? This is a different slice of science, but just as relevant to baseball as the physics of a ball trajectory. Nimur (talk) 16:39, 5 May 2008 (UTC)[reply]

Water table in natural pond.

Hi there,

I have recently bought a house (Wales, UK) that has a back garden that is on a slope going upwards. At the top there is an old pond that has been left to its own devices for many years. So during my landscaping I syphoned all of the water out of it and cleared the foot of leaves etc from the bottom. To my horror there was no liner, only a clay bottom. The pond is about 3M X 1.5M and roughly 0.5M deep. There are 3 areas in the pond that I sink down deeped into the mud. I left the pond over night and when I visited it in the afternoon the following day the water level was pretty much back to where it originally was. So I have 2 options, get rid completely using some sort of drain or keep the pond. The pond idea sounds very appealing as my old neighbour had a beautiful pond with Koi in it. However, due to the nature of the pond being clay it would be impractical as the fish would cloud the water and the water would clog the filters with clay. So I was wondering if I were to put a lining inside the pond would the weight of the water inside the lining equal the pressure stopping the water from rising?

Many thanks Paul —Preceding unsigned comment added by Bwganllwyd (talkcontribs) 22:19, 4 May 2008 (UTC)[reply]

Are you sure the water rose from the ground and it didn't just rain overnight? It seems odd for the water table to be that high, especially at the top of a slope... Either way, you should be fine with a plastic lining - the people at your local gardening centre should be able to advise you further. The water shouldn't push itself up any higher than it would be normally, so as long as your garden isn't flooded, the water in the ground around the pond should still be below ground level. --Tango (talk) 23:40, 4 May 2008 (UTC)[reply]
Water tends to go exactly where it wants to go, if the underground dynamics lead water to come out at the top of a hill (which is not impossible), beware of trying to fight it. Generally, a liner won't change the pressure relationship. Also, in my experience, ponds lined with clay tend to be crystal-clear, there is no particular reason that the clay will enter the water column. That said, observe Tango's comment above about the possibility of rain. Franamax (talk) 23:47, 4 May 2008 (UTC)[reply]

This sounds more like a spring or a seep. It is difficult to maintain an actual water table at the top of a slope. For a literal waters table, if you add a flexible plastic liner, then the water level will be exactly at the level of the water table. However, since the water table fluctuates, the liner will inevitably displace towards the surface. It would be better to work with nature and take advantage of the natural inflow of water. If you can arrange for an outflow, you will have a nice natural open-loop system with no need for pumps and filters. From your description I suspect that there is a hidden outflow of some sort anyway. -Arch dude (talk) 00:28, 5 May 2008 (UTC)[reply]


Hi there, Thanks for the replies so far, the first time I emptied the pond it rained and the pond filled back up, but there is no way that that amount of rain fell. I have since emptied the pond several times without there being any rain and it has still filled up. I have been seeking advice on several sites, here is a link to one of them with some pics on it: http://www.inspectorsjournal.com/Forum/topic.asp?TOPIC_ID=7257

Just to explain the pics, I am excavating land from the bottom to try to level the garden out. The pond is at the top of the garden.

Cheeers Paul —Preceding unsigned comment added by Bwganllwyd (talkcontribs) 04:44, 5 May 2008 (UTC)[reply]

Before you put down a pond liner you'd have to find out where your water comes from or what Arch dude says is right. You'll end up with an artificial pond on top of muck and the liner will "float up". I'm no pond expert, but what you took out seems to have been a natural garden pond. They are tricky to get going because you rely on a balanced ecosystem instead of pumps and filters. Things get a lot easier to maintain if you have an inflow like a spring or seep (instead of just rain and surface run-off) which seems to be the case here. That will help with mixing up thewater column and oxygenate the water, see Hypoxia (environmental). Plants provide shade to prevent excessive algae growth. Animals like snails, dragonflies and frogs usually arrive on their own after a while. The clay (or sometimes tile) bottom keeps water from draining and allows only a limited amount to percolate through the ground. [5] The tricky part is that, if you get too much evaporation in summer you may have to top the pond off with tap water and that can damage the eco-system. Natural ponds can get "stinky" at times even if they are perfectly balanced.
What you propose to put in is an artificial garden pond or water garden. "If fish are kept, pumps and filtration devices usually need to be added in order to keep enough oxygen in the water to support them." Pumps and filters will silt up easily in a clay bottom pond. So you need to put in a liner and there's the problem because the water that fills your pond now will no longer be able to get into your pond. Lisa4edit (talk) 16:12, 5 May 2008 (UTC)[reply]

Spider

Can someone identify this spider:

Please? –Sidious1701(talkemail) 23:48, 4 May 2008 (UTC)[reply]

Do you have a larger picture? I wouldn't be able to identify it with any picture, but I'm not sure anyone will be able to identify it reliably from such a small shot. --Tango (talk) 00:07, 5 May 2008 (UTC)[reply]
No, that's the only one I got...sorry. –Sidious1701(talkemail) 01:31, 5 May 2008 (UTC)[reply]
Similarly, I don't think I'll be able to identify it, but it might help others if you gave a few details about where the photo was taken, and maybe the time of day. Confusing Manifestation(Say hi!) 03:43, 5 May 2008 (UTC)[reply]
I think we can only safely say that it is an Orb-weaver spider.--Lenticel (talk) 12:05, 5 May 2008 (UTC)[reply]
I found it under a table, it was about sunset, and that's all–Sidious1701(talkemail) 20:51, 5 May 2008 (UTC)[reply]
Which country and region was the table in? Graeme Bartlett (talk) 21:36, 5 May 2008 (UTC)[reply]
It was in Louisiana, and it was outside.–Sidious1701(talkemail) 00:31, 7 May 2008 (UTC)[reply]
Is it like the A. bruennichi image (left) in Argiope article? As Lenticel, an orb weaver, found in warmer areas... Julia Rossi (talk) 09:10, 7 May 2008 (UTC)[reply]
I don't know what julia is talking about, I seem to remember louisiana being prety damned warm. --Shaggorama (talk) 09:49, 8 May 2008 (UTC)[reply]
Isn't that was Julia's saying? It's an orb weaver that was found in a warm area, so she named a species that fits that description. --Tango (talk) 18:02, 8 May 2008 (UTC)[reply]
My bad, misunderstood. retracted. --Shaggorama (talk) 19:46, 8 May 2008 (UTC)[reply]


May 5

Aresol Can

Why in Aresol Cans and spray paint cans is there ball inside that makes a noise when you shake it? x

Please reply the fate of the world rests upon it... —Preceding unsigned comment added by 80.176.73.246 (talk) 00:51, 5 May 2008 (UTC)[reply]

The ball is there to mechanically agitate the contents of the aerosol can when you shake it, that is, it helps you mix the contents. For instance, if you try spraying a can of red paint without first shaking it up, you get lots of propellant and very little paint. The world can rest safe... Franamax (talk) 01:12, 5 May 2008 (UTC)[reply]
It is very difficult and quite unsafe to insert a wooden stick into the aerosal can to stir up the paint medium and pigment, so the little ball is necessary. Edison (talk) 03:32, 5 May 2008 (UTC)[reply]

Other "dry ices"?

Moved from Talk:Dry ice: Is there any other substance that forms a "dry ice"? Samw (talk) 03:09, 5 May 2008 (UTC)[reply]

Most substances have some range of pressure and temperature at which sublimation happens. Read the article for other examples of normal temperature and pressure sublimation. SpinningSpark 03:21, 5 May 2008 (UTC)[reply]
Thanks! So would you call naphthalene a "dry ice"? Or does the concept of "dry ice" even make sense scientifically and it's strictly a marketing term (which it was originally)? Samw (talk)
I wouldn't call it "a dry ice". Actually I've never thought of "dry ice" as a generic description for anything, rather just a name for a specific thing (solid CO2). Naphthalene is not "ice" in the colloquial sense (it's not cold), so I would just call it a "solid". Maybe a "volatile solid". DMacks (talk) 05:19, 5 May 2008 (UTC)[reply]
Usually "ice" refers to water ice, and the special exception of dry ice is the name for solid CO2. Not all cold solid crystals are ice; otherwise, steel and copper and quartz could be "ice". Nimur (talk) 16:43, 5 May 2008 (UTC)[reply]
In astronomy, they call any solid an ice that is lighter than rock. Ammonia ice and methane ice for instance as well as water and carbon dioxide.SpinningSpark 16:50, 5 May 2008 (UTC)[reply]
Getting slightly off topic, but I think astronomy is a bad guidance for such definitions. For astronomers, stars consist of hydrogen, helium and "metal"---where "metal" is everything heavier than helium... --Dapeteばか 19:11, 5 May 2008 (UTC)[reply]

To have something resembling "dry ice" you need to have a substance with a triple point pressure that is higher than the atmospheric pressure and a sublimation point below room temperature (this second requirement is so that the substance feels cold). From the common substances listed in this table [6], only acetylene (and CO2) meets the requirements, assuming you want your dry ice to be dry at sea level, which is 101 kPa. If you are willing to move up to the mountains, a couple other substances could work. For example, xenon and nitrous oxide could form dry ices in Mexico City, where the ambient pressure is around 78 kPa. And if you use a vacuum pump with low enough pressure, many other substances can be made to behave like dry ice. A couple of the substances in the table, graphite and uranium hexafluoride, have high enough triple point pressures so that they will never be a stable liquid under normal pressure, but their sublimation point is higher (for graphite much higher) than room temperature, so I am reluctant to call them "ices". --Itub (talk) 12:52, 6 May 2008 (UTC)[reply]

Power to weight ratio

When examining the power to weight ratio of an automobile, comparing it to another, horsepower per lb. or kg. is considered. I'm wondering if a torque to weight ratio should be given consideration. If so, should it be given equal consideration as horsepower, a certain amount of consideration, or none at all? Note: peak RPM, aerodynamics, and other values are not important to me at this time, just power to weight ratio. Thanks. MoeJade (talk) 04:41, 5 May 2008 (UTC)[reply]

That's a good question/comment. I venture to say yes. One of the most important factors when considering the acceleration of an automobile is the power to weight ratio, however HP is given as peak. Torque peaks at lower RPMs, so I would say that an important variable would be low end torque to weight ratio. That would be an excellent criteria for determine the cars ability to accelerate quickly. Wisdom89 (T / C) 04:45, 5 May 2008 (UTC)[reply]
Depending on the circumstance, it should definitely be given consideration. Wisdom89 (T / C) 04:46, 5 May 2008 (UTC)[reply]

-Thanks for the fast response, Wisdom89! So would you give torque to weight ratio equal consideration, averaging power to weight and torque to weight together? Or does power to weight take precedense (sp?), with torque considered, but not given as much weight? MoeJade (talk) 04:54, 5 May 2008 (UTC)[reply]

That's tough. I think if you were going to calculate both ratios, they should be kept separate. You'll probably find a positive correlation between the two anyway. In other words, high power to rate ratios correspond to high torque to weight rations. With such a relation, the power to weight would probably take precedence. Wisdom89 (T / C) 05:35, 5 May 2008 (UTC)[reply]
Another thing that we have to bear in mind is that at any given torque, the engine is producing a certain amount of (horse) power, so that reinforces our positive correlation. This is probably why you never see the torque to rate ratio advertised or touted about. Wisdom89 (T / C) 05:46, 5 May 2008 (UTC)[reply]
Technically that is true only if the vehicle is moving. If it isn't currently moving, mechanical work is not being done (and thus output power is zero). But that may not be a very important measure for most people anyway. Most of the time I think you only have to worry about overcoming inertia (and some small amount of static friction), so overall torque or force doesn't matter all that much. --Prestidigitator (talk) 08:29, 5 May 2008 (UTC)[reply]
Even if an engine is idling and the crankshaft is being spun, then power and torque are still being generated. Wisdom89 (T / C) 12:44, 5 May 2008 (UTC)[reply]
I was trying to point out a case where the two would not be correlated. That wouldn't be at idle but instead when the engine is engaged and trying to move the vehicle when there is a significant force resisting initial acceleration from a stand-still. That is a case where force (torque if we want to ignore gearing and wheel characteristics for the vehicle) is going to be significant, but output power is zero until it "breaks free". While weight might increase the amount of static friction that has to be overcome a bit, I think significant resistive force would really come if the vehicle is stuck or starting on an up-hill gradient.
Besides, aside from that needed to overcome a small amount of friction, no torque/power is required to keep the crankshaft moving at idle. Your efficiency for any engine during that condition is going to be very close to zero, and attempting to measure the amount of torque/power is likely to significantly change the value anyway unless you are very careful about how you measure/deduce it. --Prestidigitator (talk) 18:55, 5 May 2008 (UTC)[reply]

O.k. Good input. Basically, a friend and I were comparing two vehicles for potential acceleration. Car A has 221 HP, 236 f/lb. of torque, and weighs 1430 kg. Its power to weight ratio is 0.154, or 0.154 HP per kg. Car B has 216 HP, 152 f/lb. of torque, and weighs 1180 kg. Its power to weight ratio is 0.183, which is notably better than car A. However, car A has 236 f/lb. of torque, or a torque to weight ratio of 0.165, which is substantially better than car B's torque to weight ratio of 0.128. Observing power to weight ratio only, car B wins easily. But, with car A's significant amount of torque,and its significant torque to weight ratio, it's a tougher choice than it looks, to me, anyway. Any additional comments on this are welcome. MoeJade (talk) 13:52, 5 May 2008 (UTC)[reply]

I think Car A would have better low-end acceleration, but Car B would eventually go faster. However, gearing ratios would also come into play. If you can find the cars listed in a Road & Track or Car & Driver magazine test, they often show detailed full-on acceleration numbers. Franamax (talk) 16:29, 5 May 2008 (UTC)[reply]

Does an Electron Occupy Space? Why?

There is a paragraph in article Electron I don't get it well:

"The electron is currently described as a fundamental or elementary particle. It has no known substructure. Hence, for convenience, it is usually defined or assumed to be a point-like mathematical point charge, with no spatial extension. However, when a test particle is forced to approach an electron, we measure changes in its properties (charge and mass). This effect is common to all elementary particles. Current theory suggests that this effect is due to the influence of vacuum fluctuations in its local space, so that the properties measured from a significant distance are considered to be the sum of the bare properties and the vacuum effects (see renormalization)."

As my understanding, it tells us that an electron DOSE occupy space (its radius is 2.8179 × 10−15 m according to the article) and also it somewhat explains why an electron occupies space. But I'm afraid it doesn't help for understanding the reason why electron occupies space, especially:

  • "when a test particle is forced to approach an electron, we measure changes in its properties (charge and mass)"
  • "the properties measured from a significant distance are considered to be the sum of the bare properties and the vacuum effects"

the sentences above are confusing to me...dose anyone know what do they mean?

Additional question: What if we force two electrons collide? Will they just overlap with each other and then pass through without collision?

(solving the questions above might involve some QFT and/or QED but they are too tough to me) - Justin545 (talk) 06:06, 5 May 2008 (UTC)[reply]

While that paragraph does leave much to question, I think the answer is definitely yes. The electron does occupy space. It is just really small compared with the nucleus of the atom. Thus, when a test particle is forced to approach the electron, it causes changes that can be measured. The problem with small things is that trying to measure and define their properties actually changes their behavior. So, we can only guess about their real behavior and measure from a distance so as to not affect the results.

In answer to the second question, since they occupy space, they can't just go through each other. They will collide and bounce apart. Leeboyge (talk) 07:22, 5 May 2008 (UTC)[reply]

I'm not sure. But I think things collide that's because of the fundamental interactions (strong interaction, weak interaction, electromagnetic force and gravitation) between them. If we are able to overcome the repulsion and all other forces between the two electrons, I think they will overlap and go through each other when they approach to each other. Even though they occupy space! (please correct me if I'm wrong) - Justin545 (talk) 08:14, 5 May 2008 (UTC)[reply]

Sticking my nose temerariously into a question well outside my expertise: I think the answer is that, as of 2008, no one really knows whether the electron (in the sense of the bare charge, not the cloud of virtual electrons and positrons that surround it) occupies space. My understanding is that QED takes it to be a point charge, occupying no space, and having therefore infinite density (and by the way also infinite charge -- the cloud of virtual particles surrounding it "shield" most of that charge). QED works extraordinarily well, but the extent to which it faithfully represents the underlying reality, as opposed to simply being a collection of hacks that get the right answer -- again, nobody knows. --Trovatore (talk) 08:28, 5 May 2008 (UTC)[reply]

Telling is the sentence after the statement of the electron's "radius": "This is the radius that is inferred from the electron's electric charge, by using the classical theory of electrodynamics alone, ignoring quantum mechanics." I take that to mean it helps in some physical problems (collision cross sections maybe?), but shouldn't necessarily be interpreted as a literal "size" as we think of it in intuitive macroscopic terms. --Prestidigitator (talk) 08:40, 5 May 2008 (UTC)[reply]

I'm afraid it depends on what you mean by "occupy space". If you want the everyday-scale notion of occupying space to extend down to the quantum level then I think you're pretty much forced to say that electrons do occupy space, since most of the space supposedly "occupied" by solid objects only has electron orbitals in it. The space occupied by an electron in this sense has nothing to do with any intrinsic size. A hydrogen atom and a He+ ion both have a single electron orbiting a nucleus, but the former is larger than the latter because of the different nuclear charge, even though it's an identical electron that occupies most of the space.

Protons and neutrons do have an intrinsic size: they're bound states of more fundamental particles (much like an atom is), and the bound state has a characteristic radius (much like an atom does). People have proposed preon theories where the electron is a composite particle, but none of them have had much success. If an electron were composed of preons then I suppose it would have a size in this sense, but I don't know much about this.

Strings (from string theory) have a characteristic size but don't really occupy space, being one-dimensional subsets of a three-or-more-dimensional space.

It seems fairly likely that future physical theories won't have a concept of "space" any more (except as a large-scale approximation), which will make this question even harder to answer.

2.8179 × 10−15 m is the classical electron radius. Ignore it, it's meaningless. -- BenRG (talk) 10:28, 5 May 2008 (UTC)[reply]

I could be wrong, but I think what happens if you collide electrons is Møller scattering. --98.217.8.46 (talk) 15:17, 5 May 2008 (UTC)[reply]

To make the conclusion, I presume:

  1. (classical and modern view) An electron occupies NO space. It is just a macroscopic illusion to say that an electron occupies space as the article Electron did.
  2. (classical view) An electron is just a tiny, movable electric field with definite position. Because the movable electric field has mass and inertia, when it feels a net force it will accelerate with .
  3. (classical view) Because the electron is nothing more than an electric field with mass , when we force two electrons move toward to each other they will finally pass through each other WITHOUT collision. Moreover, they will overlap (perfectly) halfway while they move toward each other. (however, Pauli exclusion principle tells us "no two identical fermions may occupy the same quantum state simultaneously" so I'm not sure if it holds in modern physics)
  4. Electrons are not solid/stiff objects/entities so they don't collide. The collision is just an illusion due to the repulsion Coulomb forces between them. So saying an electron occupies space is meaningless.
  5. The sentences "Hence, for convenience, it is usually defined or assumed to be a point-like mathematical point charge, with no spatial extension. However, when a test particle is forced to approach an electron, we measure changes in its properties (charge and mass)." in Electron should be removed or rewritten as it misleads the reader to think an electron occupies space.

the conclusion above may sound ridiculous. Please point out my errors if any. - Justin545 (talk) 01:53, 6 May 2008 (UTC)[reply]

Two electrons can't be forced to move towards eachother until the overlap, since that would require infinite energy (the force follows an inverse square law, so approaches infinity as they electrons approach each other). --Tango (talk) 16:56, 6 May 2008 (UTC)[reply]
This is false. Because electrons are ultimately probability clouds, the amount of charge at any point in space is infinitesimal. Infinitesimal charges can be seperated by neglible distance without requiring infinite energy, and as a consequence electron clouds can overlap (and do routinely in all atoms with more than 1 electron). Dragons flight (talk) 18:51, 6 May 2008 (UTC)[reply]
True, but Justin listed that item as being the classical view, and it's incorrect from the classical viewpoint. --Tango (talk) 12:33, 7 May 2008 (UTC)[reply]
From the classical viewpoint, two electrons could not overlap because . If there will be infinite potential.
From the quantum viewpoint, two electrons could overlap. For example, suppose is a wavefunction of two electrons with position , respectively. Then will be the probability of finding the first electron between interval and the second electron between interval . If is well-designed such that:
  1. and overlap. (which implies and )
  2. is very close to so they are almost at the same point
then we have a chance (with non-zero probability) of finding the two electrons arbitrarily close to each other! However, two electrons overlap may sound like to violate Pauli exclusion principle. But as long as the two electrons with different momentum they are not in the same quantum state. (again I could be wrong. please point out my errors) - Justin545 (talk) 02:11, 7 May 2008 (UTC)[reply]
I added scare quotes to "classical electron radius" in the electron article because it's not really an electron radius, but that doesn't mean electrons don't occupy space! There's no Platonic notion of occupying space that we can appeal to here. It's a matter of how you want to define the English phrase. If you define it in such a way that electrons don't occupy space, then neither does anything else, which makes the phrase useless. I think that's reason enough to use some other definition.
Also, it's pretty hard to make the case that electrons are at all point-like to begin with. In the original quantum mechanics (now taught to undergrads) you start with a quasiclassical theory where electrons are point particles and then "quantize" that, which introduces wave behavior. (The scare quotes here are because quantization (physics) isn't actually quantization. Horrible terminology.) But in quantum field theory you start with a quasiclassical field theory of electrons, so even at the classical level the electrons are spread out and occupy space to the same extent that electromagnetic fields do. When you "quantize" this theory you get field phonons which are referred to as particles, not because they are but because the term was already grandfathered in by the time QFT was developed. The only sense in which electron field quanta are particle-like is that they tend to spatially localize when they interact with a thermodynamically irreversible system (like a cloud chamber). I still don't grok why this happens, but it has something to do with quantum decoherence.
Probably the most misleading part of QFT is Feynman diagrams. They're not pictures of pointlike particles propagating and interacting in spacetime, they're graphs (in the computer-science sense) and their origin is combinatorial. Any integral of the form , where P is a polynomial, can be evaluated with Feynman diagram techniques (for some value of "any"). The idea is to write P = Q + R where Q contains only terms of degree 2 or less, and then Taylor expand R to get , which gives you a series of relatively easy Gaussian integrals. The term of this series can be represented by a collection of Feynman diagrams with interaction vertices. The diagrams always look like QFT diagrams with lines and interaction vertices, whether or not there's anything resembling space or time in the integral. There's a nice discussion of this in chapter I.7 of Quantum Field Theory in a Nutshell by A. Zee, which I just found out is available online. This expansion is extremely useful in QED, where the series converges rapidly, but not so useful in QCD, where it doesn't. There are other approaches to evaluating the integral which give you a totally different "picture of what's going on". I love the book QED, but Feynman went way overboard with his claims about the particle nature of light and electrons. In fact chapter 2, where he talks about photons only, is completely classical in almost every respect. The idea that pulses of light take every possible path from the source to the target, and every path contributes equally to the final amplitude, is just the path-integral form of Maxwell's equations. His discussions of reflection, refraction and diffraction are classical. It's crazy to claim that these arguments show the particle nature of light unless you're willing to argue that Maxwell's theory is also a particle theory. What does show the particle nature of light is the fact that the detector (photomultiplier) registers individual clicks of equal amplitude in low light. That's the only genuine quantum mechanics in the whole chapter. -- BenRG (talk) 16:33, 7 May 2008 (UTC)[reply]
BenRG: do you really think those scare quotes are necessary? I went back and saw your edit. I think "So called" is a little harsh as well. Granted, contemporary theory no longer describes the electron this way, but the paragraph qualifies the term as such. Scare quotes would imply that there is something intrinsically wrong with the phrase, which there is not. Qualifying the radius described as Classical gives it a timeframe during which the description was perceived as accurate theory. It would be like putting scare quotes around phlogiston or n-ray. Thoughts? --Shaggorama (talk) 19:57, 8 May 2008 (UTC)[reply]

Question regarding electricity

This isn't homework, it's part of my revision material and I need some help with understanding it. I'm not asking for an answer.

A filament bulb is connected to the 230V mains power supply and a meter indicates that 31 coulombs of charge flows in a period of 60 seconds. Calculate: a) the total electrical energy transferred to another form by the lamp b) the power dissipated by the lamp

Am I right in assuming that 31 coulombs of charge per 60 seconds is roughly 0.52 Amps since an amp is the flow of charge per second? My problem is i've forgotten whichever equation to use to calculate the electrical energy transferred.
As for part B, i'm right in assuming I need to use P = I^2 x V for this?

Regards, CycloneNimrodTalk? 10:47, 5 May 2008 (UTC)[reply]

Your 230V mains is AC Alternating current.71.236.23.111 (talk) 11:38, 5 May 2008 (UTC)[reply]
Your revision question is badly worded, but your calculation is right. You need Joule's Law to work out the energy and power. --Heron (talk) 12:22, 5 May 2008 (UTC)[reply]
You are right on the first statement but the second formula you give is wrong, it is . Ignore the statement above about alternating current, that has nothing to do with it. SpinningSpark 12:28, 5 May 2008 (UTC)[reply]
Thanks for your replies :) Regards, CycloneNimrodTalk? 17:18, 5 May 2008 (UTC)[reply]

A curiosity of the English usage difference between the UK and the U.S. is the British use of "revise" when we would say "review" or "study." On this side of the pond, to "revise" would only mean "to edit and make changes." Edison (talk) 18:44, 5 May 2008 (UTC)[reply]

You're editing and making changes to your knowledge of the subject, I suppose. --Tango (talk) 19:18, 5 May 2008 (UTC)[reply]
For once I'd agree that the U.S. grammar in this sense is correct, I think it makes more sense to 'review' than to 'revise', but alas, it's what i've been taught to say ;) Regards, CycloneNimrodTalk? 19:38, 5 May 2008 (UTC)[reply]
Actually the word is French in origin (ultimately Latin) and literally means to "to look at again" so it is entirely cognate with review. I suppose that the modern sense of "altering" arose because it is natural to find fault and change something after reviewing it. Anyway, when I was at school I felt it was not insignificant that the word was next to "revile" in the dictionary which is exactly how I felt about the process. SpinningSpark 20:06, 5 May 2008 (UTC)[reply]

GMT and UTC

How different are these two times? Is the difference always the same or does it vary?

Thank you. Wanderer57 (talk) 13:34, 5 May 2008 (UTC)[reply]

Try reading GMT and UTC, I expect you'll find your answer in them somewhere. --Tango (talk) 13:47, 5 May 2008 (UTC)[reply]
Here's a short answer. The term "UT" was basically invented as a replacement for "GMT" that, for political reasons, did not mention Greenwich. As time measurement improved, it was realized that GMT (UT) as it had been interpreted did not provide a suitable basis for timekeeping, and several kinds of UT were developed: UT0, UT1, UT2, and UTC. You can think of them as kinds of GMT if you like, or you can think of GMT as meaning a specific one of them. They all vary with respect to each other, but UT1 and UTC are never more than 9/10 of a second apart. Now see the articles for more detail. --Anonymous, 18:44 UTC, May 5, 2008.

White teeth

I've got a couple of questions I hope somebody can answer. Thank you.

  • if you drink black tea, and then brush your teeth immediately, how great is the risk that your teeth may become somewhat tainted anyway?
  • can your teeth become tainted to some degree even if you use a straw to drink black tea?
  • what kind of foods should one avoid in order to get whiter teeth? Mind you, I don't mean foods which damage teeth in general, just those which stain the teeth.

Thanks again. 83.37.4.200 (talk) 14:23, 5 May 2008 (UTC)[reply]

Look at Tannin to get a start. Eliminating all these foods from your diet doesn't seem a good idea. Antioxidants help your body stay healthy. Lisa4edit (talk) 14:54, 5 May 2008 (UTC)[reply]
No, it wouldn't be a good idea, it seems :) And as for my first question, has anyone got an answer please? 83.37.4.200 (talk) 14:58, 5 May 2008 (UTC)[reply]
I'm not sure avoiding foods will ever get you whiter teeth—what you mean is what kinds of foods can you avoid to avoid staining your teeth. That's not the same thing (your teeth will never "get whiter" on account of avoiding foods. You can, though, make them "get whiter" by various bleaches and etc., which are not at all expensive these days). Note that over-brushing is not a great idea in and of itself — you can damage the enamel, especially if you brush immediately after eating. Chewing sugarless gum after a meal is a better idea—the saliva cleans the teeth, restores the pH, and is non-abrasive. (I am not a dentist! But I read it in the New York Times not long back.) I have no clue though whether anything you do would have any effect on staining of that sort. --98.217.8.46 (talk) 15:09, 5 May 2008 (UTC)[reply]
Talking of chewing gum the Xylitol page might interest you. But that won't whiten your teeth either.Lisa4edit (talk) 15:58, 5 May 2008 (UTC)[reply]
I understand that peoples teeth naturally become less white as they grow older as the enamel thins and the inner part of the tooth shows through. 80.0.98.228 (talk) 10:01, 12 May 2008 (UTC)[reply]

Henry IV of France and gonorrhoea

I'm working on an account of Henry IV's love life. In his biography of Henry, David Buisseret writes: "Towards the end of the month he had another attack of gonorrhoea, which gave rise to a temporary but alarming heart condition". Could anyone tell me if temporary heart conditions can indeed be brought on by gonorrhoea? (Historians sometimes repeat this sort of thing without question, so I need to check this out.) The incident is important because Henry nearly died. Many thanks. qp10qp (talk) 15:06, 5 May 2008 (UTC)[reply]

If the bacteria spread through the body as "Disseminated Gonococcal Infection" endocarditis can occur. See http://www.ncbi.nlm.nih.gov/pubmed/1728091 William Avery (talk) 15:35, 5 May 2008 (UTC)[reply]
Many thanks. I will trust it. qp10qp (talk) 21:47, 6 May 2008 (UTC)[reply]

tuberculosis (moved from WP:HD)

what is the method of minimising resistance to treatment in tuberculosis?172.159.163.56 (talk) 16:27, 5 May 2008 (UTC)[reply]

Antibiotic resistance may be of use. Resistance to tuberculosis is pretty much the same as in other bactera. The best ways to reduce resistance are to be picky about who you give antibiotics to and to make sure that they finish their course of antibiotics. If bacteria aren't killed off completely, they'll likely come back with a resistance and hey presto, you have yourself a new strain of resistant bacteria. Of course it's not quite that simple, but it's along those lines ;) Regards, CycloneNimrodTalk? 17:16, 5 May 2008 (UTC)[reply]
The principles specific to antituberculosis treatment: [1] never use only one antibiotic; combination therapy is mandated. It is important to assess the resistance pattern of the specific strain of tuberculosis infecting the patient, as using an ineffective antitubercular may leave one effectively treating with only one (or no!) effective antibiotics. This is an exception to the usual admonition to avoid polypharmacy. [2] directly observed therapy: patient compliance with prescribed medication is monitored and enforced by public health officials. - Nunh-huh 22:54, 5 May 2008 (UTC)[reply]

Gondwanan distribution and Laurelia

There are two species of Laurelia, one native to the Andes - Laurelia sempervirens (Chilean Laurel, Peruvian Laurel, Peruvian Nutmeg) and the other found in New Zealand - Laurelia novae-zelandiae (Pukatea) .

When I saw one of these trees, growing at Enys in Cornwall, UK, yesterday, the Head Gardener said they were examples of "Gondwanan distribution". I have looked at the Gondwana article, but this is very brief on the topic, which is a REDIRECT to Godwana.

I would like to know more about the differences between the two species and when, approximately, they diverged, due to the separation of New Zealand and South America from the Gondwanan continent. Vernon White . . . Talk 19:24, 5 May 2008 (UTC)[reply]

I shall leave the differences to the botanists. As for the divergence, as I understand it, dating the Gondwanan distribution is not easy. For example, in a discussion on ferns, Te Ara Encyclopediaof NZ suggests that some distribution was windborne. this Te Ara page also discusses the time span, ocean distribution and later isolation. Googling Pukatea + Gondwanan yields 39 results which might bear investigation. Gwinva (talk) 05:17, 6 May 2008 (UTC)[reply]

engineering question

Which field fo engineering is the most multidiciplinary i.e. which concerns the widest range of topics from maths, physics, chemistry, geography, biology etc. Clover345 (talk) 20:53, 5 May 2008 (UTC)[reply]

Possibly constructions with biopolymers while exploiting advantages geological features. You know these disciplines are just groups into which we categorise knowledge, they interconnect depending on how do you define there disciplinesBastard Soap (talk) 21:21, 5 May 2008 (UTC)[reply]

Environmental engineering would be my suggestion. I'm not certain I'd recommend it as an area with stellar career opportunities, though. Lisa4edit (talk) 21:33, 5 May 2008 (UTC)[reply]
I would propose Electrical Engineering. After discussions with many of my colleagues, I have reached this conclusion. In 2008, "electrical engineering" may refer to the base science and the applied technologies in fields as diverse as:
I personally know "electrical engineers" who work in all of the above areas. The job market is huge - software companies, aerospace, communications, academia... Nimur (talk) 00:50, 6 May 2008 (UTC)[reply]
An interesting question. I would suggest that mechanical engineering would be as this covers a wide range of diciplines including electrical / electronics engineering, both on the micro scale, for example nanobots and macro scale, for example, lifts and ventilation in buildings. Civil engineering mainly covers all static objects while mechanical covers moving objects. Civil may also be multidiciplinary but usually only involve large scale building projects. Environmental engineering is a sub-dicipline of civil engineering. Tbo 157(talk) 18:04, 6 May 2008 (UTC)[reply]
Where they put Environmental Engineering depends on your U. MIT and Stanford put it with the Civs but lots of the smaller ones either have a separate department or put it somewhere else. In many other countries it is a separate discipline, too. 71.236.23.111 (talk) 06:56, 7 May 2008 (UTC)[reply]
Multidisciplinary with regard to engineering usually means it contains many aspects of engineering - e.g. electrical, civil, mechanical, electronic, aeronautical - rather than the multiple subjects you mention. However, from my experience, I would suggest that civil most closely fits your definition of multidisciplinary. There's the maths and physics, along with a lot of work on environmental/geographical elements, and fluid dynamics (wind flows). It lacks chemistry and biology though... LHMike (talk) 09:12, 8 May 2008 (UTC)[reply]

Atomic vibration

Can anyone please explain to me what makes the little buggers vibrate about?Bastard Soap (talk) 21:16, 5 May 2008 (UTC)[reply]

What kinds of atoms, in what situation? If you're talking about air at room temperature, then most of the molecules are not vibrating. They're moving and rotating, but not vibrating, because the first vibrational energy level above the non-vibrating ground state is too high compared to the average thermal energy. If you're talking about some solid material like a metal, then it is vibrating, because unlike vibrations of gas molecules, there is effectively no lower limit for the energy of a phonon.
So to sum up, the answer to your question is that when things are vibrating, they're usually vibrating because of heat. Heat is the microscopic motion of atoms. —Keenan Pepper 23:55, 5 May 2008 (UTC)[reply]
That seems a little circular to me. They're vibrating because of heat, which is vibration? --Tango (talk) 00:35, 6 May 2008 (UTC)[reply]
Heat is a form of energy. If you put energy into a system the atoms will move more, i.e. they will have more heat.--Shniken1 (talk) 00:44, 6 May 2008 (UTC)[reply]
In thermodynamics (as opposed to everyday usage) temperature and heat have very precise definitions and are not synonymous. Temperature is a measure of the thermal energy of an object, whereas heat is the energy exchanged between an object and its surroundings if they are at different temperatures. If an object is at a lower temperature than its surroundings then heat energy will flow into it, but this energy does not necessarily increase the object's temperature - the object might instead do work by expanding against an external pressure, or it might change state. Conversely, it is possible to increase an object's temperature above that of its surroundings by doing work on it - by compressing it, for example, or by boring a hole in it - or to reduce its temperature by making it do work - by allowing it to expand, for example. Gandalf61 (talk) 09:34, 6 May 2008 (UTC)[reply]
Heat is the macroscale result of atomic level vibration. They are the same. The faster the atomic vibration, the higher the bulk sample temperature. —Preceding unsigned comment added by 131.111.100.44 (talk) 16:30, 6 May 2008 (UTC)[reply]

I knew that heat makes them vibrate, my question is why is it so?Bastard Soap (talk) 18:32, 6 May 2008 (UTC)[reply]

Heat is energy. If you add heat to a molecule, either it becomes higher in kinetic and/or potential energy. Motion is kinetic energy... DMacks (talk) 21:48, 6 May 2008 (UTC)[reply]
Indeed. In thermodynamics, heat is just a type of energy flow. If you add energy to an object, some of it may go into doing work that we can measure at a macroscopic level - expansion, for example. Some of it may go into breaking inter-molecular bonds - a change of state. And some of it may go into increasing the vibrational and kinetic energy of each individual molecule. We cannot observe this molecular energy directly, but we can observe it indirectly, because the object then tends to transfer heat energy back to its surroundings in accordance with the second law of thermodynamics. What we call "temperature" is just a measure of the average molecular energy of an object. Gandalf61 (talk) 22:26, 6 May 2008 (UTC)[reply]

My question is still why and how do the molecules vibrate? I know that the energy of heat has to go somewhere, I just don't understand what system causes it to vibrate.Bastard Soap (talk) 17:26, 7 May 2008 (UTC)[reply]

This is one of those "just-because" answers. When a molecule increases in energy, that has to be reflected somehow. The somehow's include increases in translational, rotational, vibrational and excitational modes. Vibration is just one of the ways to store the extra energy. How? Energy is transferred to the molecule and taken up by the particular mode. Why? Because that is the most appropriate mode to accommodate the energy. There are no easy answers when it comes to really tiny things. Franamax (talk) 10:00, 8 May 2008 (UTC)[reply]
If you are asking about the mechanisms that transfer heat energy from the surface of an object or container into the molecules that may lie deep inside it, then the canonical answer involves the three mechanisms of conduction, convection and radiation. Radiation involves energy transfer via electromagnetic waves; conduction involves the transfer of energy via inter-molecular forces; convection involves the macroscopic movement and mixing of a fluid. There are other, less frequently quoted, mechanisms such as thermodynamic phonons in crystals. Gandalf61 (talk) 10:34, 8 May 2008 (UTC)[reply]


May 6

Dino-mite question

What are the 10 newest dinosaurs? --DinoDude08 (talk) 00:33, 6 May 2008 (UTC)[reply]

What do you mean by "newest"? Most recently discovered, or most recently lived? --Tango (talk) 00:36, 6 May 2008 (UTC)[reply]
General Motors, Ford, and Chrysler are starting to look like really good contenders for at least three of the ten positions.
Atlant (talk) 13:14, 6 May 2008 (UTC)[reply]
i think it's safe to say that Dinodude meant newest discovered.... but that can be a tricky question since paleontologists are reclassifying old bones all the time... added to that the debate that sometimes occurs when "new" dinosaurs emerge, which is actually a lot more often than you might think. you just want what the latest news in paleontology is, check out [www.palass.org this], or this. -ΖαππερΝαππερ BabelAlexandria 20:49, 6 May 2008 (UTC)[reply]
Ok. Dino Dude joking aside. Currently birds are believed to have evolved from dinosaurs. So, as our article says "they are the only living dinosaurs". Have a look at the Hoatzin which seems to have regained some dinosaur traits. (Claws on their wings.) Non-bird dinosaurs are usually thought to all have perished in the K-T extinction. So singling out 10 species would be rather arbitrary. There are occasionally bones found above that layer, but scientists think that's because they were moved later. As far as recent discoveries are concerned, it can take a very long time from the moment the bones are dug up, to the point when the bones have been prepared, to the identification of the species and finally the announcement. Look at Xenoposeidon for example. Since it is much more common that only one or two bones remain, rather than an entire specimen, scientists are very careful before they announce a new species. It is not uncommon, though, for bones to get relabeled to belong to a different species than the one originally identified. A quick search resulted in following other candidates for a "top ten" list, but please don't view this as correct and complete:
Paluxysaurus, Eocarcharia dinops, Kryptops palaios, Rugops primus, Velafrons, Nemicolopterus crypticus, Albertaceratops nesmoi, Gigantoraptor erlianensis, Mei (dinosaur), Onychonycteris was a bat and may not qualify
There was mention of a recent find in China, but I could not get at the information. Hope this helps. Lisa4edit71.236.23.111 (talk) 23:19, 6 May 2008 (UTC)[reply]

About NARCOTICS:

About NARCOTICS: I wanted to know can anyone help me find out all the slangs and everything there is to know about illegal narcotics from herb to cocaine, soft to hard, numbers, everything?? —Preceding unsigned comment added by KTTK2008 (talkcontribs) 00:39, 6 May 2008 (UTC)[reply]

What you want is www.erowid.org. Trust me, they have everything on every drug you can imagine, from caffeine to LSD. Chemistry, law, effects, even experience reports. Have (safe) fun. --Shaggorama (talk) 09:56, 8 May 2008 (UTC)[reply]

Genetic code

Do I understand correctly that if I have genes, which represent a high probability of my contracting a certain ailment that an insurance company can not use that information to deny insurance to me or as the basis for increasing my insurance premium? --Schaum 01:29, 6 May 2008 (UTC)

This is bordering on legal advice, which we are not really qualified to dispense. I suggest asking your/a lawyer. Wisdom89 (T / C) 01:30, 6 May 2008 (UTC)[reply]

Nonsense. --Schaum 10:52, 6 May 2008 (UTC)

In most places, the insurance company can use such information freely to jack up your rates. It's only in the very rare jurisdiction where a law has been passed to prevent this that they have to think twice about doing it. - Nunh-huh 01:41, 6 May 2008 (UTC)[reply]

If you're in the US, see Genetic Information Nondiscrimination Act. However, be aware that (1) that link is only to a Wikipedia article, which could be misleading, (2) the act has not yet been signed into law, and (3) we don't yet know how effective it will really be in meeting its intended purposes. --Anonymous, 04:44 UTC, May 6, 2008.

When I first learned about actuary tables I recall that insurance companies included anything they could correlate. In fact I learned that is what made insurance possible. Consequently I do not see them doing otherwise under any circumstance short of pulling their license to do business which would require a law that I am sure they will be sure is never passed. --Schaum 10:52, 6 May 2008 (UTC)
I agree. I also hope, though, that within the next few years we'll have (in the United States, anyway) turned health coverage into a public service (like police and fire coverage) and so these sorts of things will be moot concerns. I understand that those in a for-profit business need to make money, but I think it's stupid to have healthcare be a for-profit business in the first place. --98.217.8.46 (talk) 12:26, 6 May 2008 (UTC)[reply]
98.217.8.46 i'd read a bit more about public-provided health insurance schemes before deciding it is foolish to have for-profit provisions of healthcare. Most of the developed world have for-profit provisions of water, energy supplies and food-stuffs (all essential to supporting life) without major concerns. Drug research is also for-profit and is potentially the reason for such innovation (though patent/copyright law also potentially cause slow down of development). I recommend the chapter on healthcare from the book 'The Undercover Economist' - it mentions a very interesting health-system. I forget which country it is, but i think it is Sri Lanka or somewhere in the orient which had one that, for me, made the most sense (a somewhat hybrid of government, individual and insurance provision) —Preceding unsigned comment added by 194.221.133.226 (talk) 14:38, 6 May 2008 (UTC)[reply]
I've read into it—I think more people benefit from easy, affordable basic care than do, on the whole, from all of the supposed research and drug production that the money is paying for (and I think the costs of the privatized healthcare are far too high, both economically and from a human point of view). (And the economics of water, energy, and foodstuffs are highly regulated in most countries, though they have a superficial appearance of a free market on the client side of things.) Anyway, that's a different question, and a debate at that. We can stop this here. --98.217.8.46 (talk) 03:27, 7 May 2008 (UTC)[reply]

Washingtonion Palm trees

I have read the Wikipedia essays on Palm Trees but cannot find an answer to the following question: What is the life expectancy of Washingtonian Fillifera, Washingtonian Robusta and Medjuel date palms? Particulaly in the southern California area. Thanks, WSC —Preceding unsigned comment added by 75.85.203.191 (talk) 02:17, 6 May 2008 (UTC)[reply]

I'm not sure about date palms (Phoenix dactylifera), but Washingtonia filifera (California Fan Palms) and Washingtonia robusta (Mexican Fan Palms) can easily get to be 100 years old. There are numerous plantings of these palms in southern and central California that date from the early 20th century. I suspect that they can live much longer than that if in the right environment. W. filifera is, of course, native to the desert in southern California.--Eriastrum (talk) 15:37, 6 May 2008 (UTC)[reply]

Alaskan Mosquitos

How do Alaskan Mosquitos survive the winter? Do they fly south before winter? Do they hibenate? 203.94.145.126 (talk) 04:42, 6 May 2008 (UTC)[reply]

Mostly the mosquito eggs laid at the end of summer just wait to develop until it gets warm again. Dragons flight (talk) 04:57, 6 May 2008 (UTC)[reply]
They carry off bears to feed on during the winter.
Seriously, Dragons flight has it right. The eggs survive the winter and hatch again in spring. The adults simply die off. Which is a good thing. Those suckers are huge. There's a reason Alaskans call them the "state bird." -- Kesh (talk) 20:57, 6 May 2008 (UTC)[reply]

plastic containers

Before throwing a large number of various plastic food product containers in the trash where can I find information about personal methods of recycling such as using soda bottles as seedling starters, etc.? --Schaum 06:23, 6 May 2008 (UTC)

Okay then, where best to ask this green question, if not the Wikipedia reference desk? 71.100.14.205 (talk) 01:12, 7 May 2008 (UTC)[reply]
Most councils or government agencies will offer recycling tips. Check out these, for starters: [7], [8] [9], [10] and so on (try googling). http://www.recyclethis.co.uk/ looks good, at first glance. But remember that not all plastics are suitable for reuse. Gwinva (talk) 01:27, 7 May 2008 (UTC)[reply]

IMAX type 3D Camera doing time lapse photography!!??

OK, did I think of this first? Beautiful films of plants growing in time lapse AND 3D!!

Know what I'd really like to see? In Australia, after one of our bushfires, the whole scene looks like a blackened hell hole. Then in the space of several months, it regenerates. Oz bush is DESIGNED to burn and regrow. And no one has EVER filmed it in time lapse. It would look so coooool. National Geographic, where are you? And I would like a credit on your film. And some money. Myles325a (talk) 08:06, 6 May 2008 (UTC)[reply]

How do you know no one has filmed it with time lapse photography before? Also you don't a special camera special camera to do 3D photography just 2 identical cameras set at a fixed distance apart, see Stereoscopy Nil Einne (talk) 15:41, 6 May 2008 (UTC)[reply]

The size of the atom according to the ancient atomists

I've posed this question here as well, from WP:RD/H, in case anyone can give further scientific insight. PeterSymonds | talk 10:26, 6 May 2008 (UTC)[reply]

From this article, [11] I got this quote:

Some controversy surrounds the properties of the atoms. They vary in size: one report—which some scholars question—suggests that atoms could, in principle, be as large as a cosmos, although at least in this cosmos they all seem to be too small to perceive

Can anyone tell me more? Who was it who suggested atoms could be so huge? Were they suggesting another universe populated with a single gigantic atom?

Thanks Adambrowne666 (talk) 10:23, 6 May 2008 (UTC)[reply]

I can't find that quote in that article. Without context, it's difficult to work out what it means. It seems to be a philosophical article, which would suggest it doesn't really mean much at all. --Tango (talk) 13:35, 6 May 2008 (UTC)[reply]


Sorry, Tango, you're right; it was the wrong link - I've fixed it now.Adambrowne666 (talk) 21:18, 6 May 2008 (UTC)[reply]
Ok, even with context I have no idea what it's talking about. It does provide a reference, though - see if you can find a copy of "Diels, H. and W. Kranz, 1951, Die Fragmente der Vorsokratiker, 6th ed. (Berlin)." and find "68A47" in it (I'm not sure what that means, chapter and page, or something, maybe). Of course, it looks like it's in German, so if you don't speak German you will need to find a translation... --Tango (talk) 23:47, 6 May 2008 (UTC)[reply]
This sounds remarkably similar to a theory i've heard of which says the universe is contained within a quark? Regards, CycloneNimrodTalk? 17:10, 6 May 2008 (UTC)[reply]
Keep this in context. Thomas Kuhn showed that often the meaning of a scientific term changes after a paradigm shift in a way not perceived by the proponents of the new paradigm. Our notion of the atom is almost certainly incomensurable with what the ancient greeks meant; atomic theory as we know it today didn't really start to develop until the time of the alchemists, and what we believe now has been so drastically affected by quantum mechanics it would be incomprehensible that if you and democritus (or even if you and an alchemist) had a conversation about atoms that the two of you would be talking about the same thing. --Shaggorama (talk) 10:05, 8 May 2008 (UTC)[reply]

Thanks, everyone Adambrowne666 (talk) 23:20, 10 May 2008 (UTC)[reply]

Why the contraction of pronator quadratus produces pronation, and not supination? Think of it, unlike pronator teres, it is not attached to a more proximal fixed point like the medial border of the humerus. This muscle runs absolutely perpendicular to the radius and the ulna.

I have thought about it, and feel that when the forearm is supine, there's no further margin for supination, so the muscle acts to bring the radius closer to the ulna, and not vice versa. And, when the forearm is pronated fully, the muscle must be getting so much relaxed (consequently, also shortened) that its contraction cannot further shorten the fibers, hence no movement occurs in this position.

Well, this is just a guess. It'd be nice if someone approves of it, or corrects my ideas. Regards.

PS: I have posted the same doubt on the talk page of the above article. Didn't get any response. Any way, it's not a "high profile" muscle ;). So, that's understandable.

Ketan Panchal, MBBS (talk) 17:48, 6 May 2008 (UTC)[reply]

Pronator quadratus runs from the lateral (outside) of the radius to the medial border of the ulna, and is found in the anterior compartment of the forearm. Thus when the arm is in a supine (palm up) position, contraction of pronator quadratus acts to roll the outside of the radius toward the ulna, which is pronation. It may help to visualize this by placing your right forearm palm up in front of you and then pinching your ulna and radius with the index finger (on the radius) and thumb (on the ulna) of your left hand. The pinching will simulate contraction of pronator quadratus, and will show the action of the muscle. -- Flyguy649 talk 06:01, 7 May 2008 (UTC)[reply]

The Law of Gravity

I was in science class yesterday doing a chapter on gravity. I asked the teacher whether or not gravity was a law and my teacher said that gravity was only a theory. This sparked a classwide debate, which resulted in my getting a detention for "disrupting class." My teacher will refuse to explain why gravity is a "theory." I think that gravity is a quite obviously a law. I need to know: am I correct? Could someone help me? Thanks. 31306D696E6E69636B6D (talk) 18:01, 6 May 2008 (UTC)[reply]

Newton's law of universal gravitation can be proved mathematically and is an accpeted physical law See responses below. However most of science is just accepted theory based on "what works". If you look through the history of science, theories and laws have constantly been debated and adapted using both mathematical and experimental evidence. Sorry about the detention. I really don't see how sparking a debate can be a valid reason for a detention unless it was during a test or something. Tbo 157(talk) 18:11, 6 May 2008 (UTC)[reply]
What do you mean when you say it "can be proved mathematically"? It's not even a correct model of reality—hence the formulation of general relativity. In response to the question, the distinction between a scientific law and a scientific theory is essentially meaningless, and often based on historical use. We talk about the "law of universal gravitation" and the "theory of relativity", even though the latter is certainly more correct than the former. If your teacher really said "only a theory", I would criticize that use—"theory" is quite a strong word in science, and is only used when the evidence is very strong. It doesn't mean "guess" or "hunch" like it might in everyday use. -- Coneslayer (talk) 18:22, 6 May 2008 (UTC)[reply]
  • (edit conflict) Indeed, the first response is wrong. Not only is Newton's law of universal gravitation not proven, it is, on the contrary, disproven and has been replaced by Einstein's General Theory of Relativity, which is a better description of reality, but also quite likely wrong. Strict proof is only possible in mathematics. In science, a theory is a self-consistent explanation of a set of observations that allows testable predictions. A "law" is a concise statement of (usually cental parts of) a theory that has been very successful in making correct predictions. Newton's theory, while certainly wrong, is a good enough approximation to be still very useful, so we still talk about the "Newton's law of gravity". In fact, a good theory is the best you can hope for in science. And, while I'm nitpicking anyways: Gravity is neither a law nor a theory, but rather a force ;-) --Stephan Schulz (talk) 18:31, 6 May 2008 (UTC)[reply]
Well, Newton did prove the law of gravitation mathematically - but only after taking Kepler's laws of cosmic motion and his own law's of motion as postulates. The great step forward Newton made was to show that both terrestial and celestial motion could be described by a common set of laws. All scientific theories have to start with something, ad what you can prove mathematically depends on what the "givens" are that you are assumming. We constantly modify the "givens" in the light of scientific experiments. We constantly try and reduce the number of givens we have to assume but we will never eliminate them entirely, there will always be at least one law that must be accepted without proof and from which everything else is derived. SpinningSpark 18:38, 6 May 2008 (UTC)[reply]
There's a common misunderstanding about what the word "theory" means. A theory is stronger than a law. A law is what you get when you do lots of experiments, plot them on a graph and draw a trendline - the formula for the trendline is the law. A theory is something which explains why that law holds, and allows you to predict more laws. "Only a theory" is nonsense, since a theory is the strongest thing you have in science. --Tango (talk) 18:29, 6 May 2008 (UTC)[reply]
You are drawing a narrow distinction between "law" and "theory" that is not well reflected in the way that the terms are used in practice. See physical law for discussion of characteristics often associated with "laws". Ultimately though it is a semantic distinction that is much more important to historians of science than it is actually to scientists. Dragons flight (talk) 18:37, 6 May 2008 (UTC)[reply]
To quote that very article: "Simply stated, while a law notes that something happens, a theory explains why and how something happens." I think that's pretty much what I said. --Tango (talk) 20:36, 6 May 2008 (UTC)[reply]
My concern, which may or may not have been what you intended, is that your original statement describes laws very emprically and suggests (at least in the way I read it) that laws describe relationships where the explanation isn't understood. In practice though, the expression of most laws isn't merely empirical and is motivated by some underlying theoretical understanding. Hence the line between laws and theory is inherent fuzzy. Maybe I reacted to your original statement differently than you intended. Dragons flight (talk) 16:26, 7 May 2008 (UTC)[reply]
Yes, I was a little unclear. Laws are strongly supported by empirical evidence. They are generally explained by theory, but don't need to me (I believe Kepler's Laws were purely empirical for a while before Newton showed they could be derived from his Universal Law of Gravity, for example). Theory, on the other hand, is mainly explanation and isn't particular supported by evidence directly - theories predict laws, which are then supported by evidence. --Tango (talk) 16:33, 7 May 2008 (UTC)[reply]
What if gravitomagnetism is proven right? What if antigravity turns out to be possible? What if antimatter turns out to be repelled instead of attracted by matter? Em3ryguy (talk) 20:16, 6 May 2008 (UTC)[reply]
Then we'll try and come up with a new theory that fits the new observations. That's how science works. --Tango (talk) 20:36, 6 May 2008 (UTC)[reply]
Wasnt that my point? its a theory. a law would be a rule that is not and cannot ever be broken. conservation laws, presumably, cannot be broken. Em3ryguy (talk) 20:58, 6 May 2008 (UTC)[reply]
Actually, conservation laws can be broken, albeit for an extremly short amount of time. I believe it's the law of conservation of matter. I don't know of an article about it, but on the science channel there was something that said particles can pop into existence but pop back out in such a short time that there is no effect. It had something to do with the Heisenburg Uncertainty principle allowing it.Zrs 12 (talk) 01:52, 7 May 2008 (UTC)[reply]
That's a fundamental misunderstanding of Scientific law and Scientific theory. Law is simply a mathematical proof, while a theory is a broad description of what happens.
That said, I'm confident gravitomagentism will never be proven. Gravity and electro-magnetism are polar opposites (pardon the pun). Magnetism is extremely powerful at close distances, but rapidly loses effectiveness once the distances increase. Gravity retains its strength over long distances, but is overpowered by magnetism on the small-scale. They work very differently. -- Kesh (talk) 21:04, 6 May 2008 (UTC)[reply]
You're confused in your example. Gravitomagnetism is a thus far impossible to measure property of general relativity that is unrelated to electromagnetism except that the equations have a similar structure. Dragons flight (talk) 21:09, 6 May 2008 (UTC)[reply]
Ahh, thank you for the explanation. The only times I've heard "gravitomagnetism" it was from self-styled Internet Einsteins who thought they had solved every problem in the universe by simply replacing gravity with electromagnetism. The math was horrid, and the logic was worse. -- Kesh (talk) 21:10, 7 May 2008 (UTC)[reply]
It is possible for laws to be broken. There's no way for us to know that V will always equal IR (Ohm's law) - it always has up until now, and everything suggests that it always will, but there is still a non-zero chance that it's just been a big coincidence. A law is *not* an absolute statement - that's a "theorem" and is a purely mathematical thing. You might have a theorem that says "given this theory, such and such will happen", for example, given Newton's Universal Law of Gravity, we can mathematically prove the shell theorem, but we still can't be sure that the result of that theorem will always hold, since we can't be sure the law will always hold. --Tango (talk) 12:39, 7 May 2008 (UTC)[reply]
Try this, which works in my world.
  • In the humanities etc, "theory" is synonymous with "hypothesis".
  • In the sciences, "theory" is synonymous with "model", and specifically, a model that explains all the existing observations.
Succinct. --Danh, 70.59.79.108 (talk) 23:02, 6 May 2008 (UTC)[reply]
Close enough. This is also why it's hard to explain to some people that just because a scientist calls something a theory doesn't meant that it's random conjecture. Proper scientists don't speak in absolute terms, but that doesn't mean they lake confidence, that just means they're real scientists. 206.126.163.20 (talk) 00:23, 7 May 2008 (UTC)[reply]
The way I would state it is, both laws and theories are build on axioms (things that must simply be assumed true, probably based on experience and observation). A law can be quite simple (energy is conserved) or it can be more complex (entropy always increasing in time, the definition of entropy being pretty non-trivial--at least quite a bit more than for energy). A theory takes a set of axioms and builds something somewhat complex on top of it; generally a large set of rules and conclusions. Both must be logically consistent in order to have a chance at being useful. We would generally call something a "law" or remove its status as mere "theory" once its axioms have been tested/experienced enough that we are pretty assured of their truth. If a law or theory is completely sound logically, testing the axioms may be done at least in part by testing the law/axiom's conclusions. That's just my take on it though. Think about it and see if it makes sense for you. --Prestidigitator (talk) 00:07, 7 May 2008 (UTC)[reply]
Consider this: Not only has Newton's law of gravity been proven to be but an approximation, but some physicists doubt that Einstein's theory of relativity is complete. Imagine Reason (talk) 20:04, 7 May 2008 (UTC)[reply]
People know Relativity isn't complete since experiments on very small scales deviate significantly from it. --Tango (talk) 22:21, 7 May 2008 (UTC)[reply]

Infinity

I'm not sure if this belongs here or the mathematics desk or possibly even the philosophy desk, but i'll ask it anyway. Is infinity accepted as being real or is it simply a state which we haven't reached? If that makes sense. I.E. If you divide 100 by 3 you get 33.3 with the .3 recurring. Does this go to infinity or is there a limit?

Regards, CycloneNimrodTalk? 19:01, 6 May 2008 (UTC)[reply]

Oh, I don't just mean it as a mathematical context either, anything where infinity is said to exist e.g. at the event horizon of a black hole? Regards, CycloneNimrodTalk? 19:03, 6 May 2008 (UTC)[reply]
A Repeating decimal is infinite, yes. Other than that, I'm not sure I understand the question. Friday (talk) 19:05, 6 May 2008 (UTC)[reply]
It's a mental and mathematical abstraction. Wisdom89 (T / C) 20:19, 6 May 2008 (UTC)[reply]
this is prolly better taken up with the philosophers, because i'm sure someone will come up with something extremely complicated and technical.... but i believe that science and math and people in general accept infinity as being real... it's the basis of our concepts like dimensional space and counting. -ΖαππερΝαππερ BabelAlexandria 21:04, 6 May 2008 (UTC)[reply]
I'll give an example. Time. I think most people agree that there must have been a start to time, or do we keep on getting infinitely closer to seeing the beginning? Regards, CycloneNimrodTalk? 21:14, 6 May 2008 (UTC)[reply]
Time is a convenient concept created by human beings in order to get up the next morning for work, or know not to go to work on Sunday. It has little to do with infinity. There are entire mathematical branches that deal with infinity starting with calculus ("approaching") and complex numbers (z), but that is scientific. It appears you first questioned maths, then ended off on a more philosophical note. Perhaps infinity was given to us by God, denoting that which cannot be obtained by mere mortals. I can go on. The bounds of one's imagination are infinite. Sandman30s (talk) 21:35, 6 May 2008 (UTC)[reply]
Infinity, or its many variants, is a mathematical construct. In the example of 33.333..., or more interestingly 0.999..., the "infinite number" of decimals places merely means that "for any decimal place you find, there is one after it". In the physical sciences, the appearance of infinity - or a singularity - as the solution to an equation generally translates as "things break down here". For example, the event horizon of a black hole is a "removable singularity", meaning that it's weird, but after a change of coordinates it disappears. The centre of a black hole, however, is an "essential singularity", which translates as "general relativity can't tell you what happens at the centre". Confusing Manifestation(Say hi!) 23:19, 6 May 2008 (UTC)[reply]
A fun thought experiment that relates to infinity and time (and space) is Xeno's_paradox#Achilles_and_the_tortoise. -- JSBillings 13:28, 7 May 2008 (UTC)[reply]

What does "qsp" in reference to gelatin or perhaps other polymers mean?

I found a reference of "qsp" to gelatin for which I couldn't find a definition on Google. Based on what I did see on Google, it seems to be a unit of measurement for gelatin and perhaps polymers. In the case I refer to, it said "gelatin qsp 100%" Can anyone help define/explain this further?

Thanks! —Preceding unsigned comment added by 65.209.203.254 (talk) 19:11, 6 May 2008 (UTC)[reply]

I web searched on "gelatin" together with "qsp" and eventually found some examples where qsp seemed to mean specific viscosity. However, on looking this up, I find that the symbol for it actually uses the Greek letter eta (looks like n with the bottom right corner extended), not the English letter q. In a Google search on both "specific viscosity" and "qsp", many of the results are papers that cannot be freely downloaded, but some hits -- for example this paper in PDF, the 10th hit when I did the search -- show qsp in Google's search synopsis while the actual document has an eta and sp.
So I'm guessing that qsp is being used to mean specific viscosity and the q is a mistake for eta, perhaps originating due to a character-set problem, or else an alternate symbol used if Greek letters are not available.
Either that or I'm completely wrong.
I see that Wikipedia once had a WikiProject Polymers that intended to produce an article on specific viscosity, but the project appears to be moribund. --Anonymous, 20:03 UTC, May 6, 2008.
I think that this probably refers to the reaction quotient for solubility, Qsp. See also Solubility equilibrium. Rmhermen (talk) 16:03, 7 May 2008 (UTC)[reply]

If you saw this on a food, drug or cosmetic, it might have actually been USP for United States Pharmacopeia which would indicate the gelatin met standards set by USP. ike9898 (talk) 21:48, 7 May 2008 (UTC)[reply]

Swinging

Why does swinging one's legs when on a swing help? —Preceding unsigned comment added by 76.69.240.138 (talk) 22:05, 6 May 2008 (UTC)[reply]

Well, I expect it's about moving your centre of mass, which imparts energy to the swinging. Your weight can be modelled as a point force acting through your centre of mass, and the swing itself as an inextensible string. At rest, the swing hangs straight down, through your centre of mass, so the forces balance out. If you move your centre of mass, you generate a turning moment, which moves the swing as it tries to find equilibrium again. -mattbuck (Talk) 23:53, 6 May 2008 (UTC)[reply]
Scientific American had an article in their Amateur Scientist section in March 1989 (unfortunately they don't have that accessible online) The interesting thing was the discussions that article started. Our swing article has a pdf link at the bottom. This site has video that might help answer your question. [12] Lisa4edit (talk) 03:11, 7 May 2008 (UTC)[reply]

Bicycles

Why is it easier to balance a bicycle when it is moving quickly. I asked someone this, and he said that it was because gravity was causing the bike to go in centripetal motion, but I don't think that makes much sense because it should only account for the bike turning. I've tried looking for the answer online, but I haven't been able to find consistent answers. Thanks. —Preceding unsigned comment added by 76.69.240.138 (talk) 23:09, 6 May 2008 (UTC)[reply]

Don't nail me to this, but I think it has to do with the force vectors. While you are stationary any forces to the left or right will take full effect. While you are moving forward the main momentum/force is in that direction and the left and right tilts will have less effect. 71.236.23.111 (talk) 23:34, 6 May 2008 (UTC)[reply]
Wikipedia has an article on everything....Bicycle and motorcycle dynamics--Shniken1 (talk) 23:43, 6 May 2008 (UTC)[reply]
Conservation of angular momentum. It's the spinning mass of the tires that help keep you upright. If you add weight to the tires and go at the same speed, it will be even harder to tip over. Here's a video.--Duk 03:30, 7 May 2008 (UTC)[reply]
Took the liberty of fixing your broken link. —Keenan Pepper 05:24, 7 May 2008 (UTC)[reply]
Actually, the effect of angular momentum is very minor. It has the role of a steering force, but is not a significant righting force. Our article, mentioned above, discusses all the effects in much detail. --Stephan Schulz (talk) 05:35, 7 May 2008 (UTC)[reply]
The effect of angular momentum depends on speed (squared). At slow speeds it doesn't do much, at fast speeds and with heavy tires it does a lot. Riding a large motorcycle is a good example, you can practically hang off to one side and the bike will not tip. --Duk 14:53, 8 May 2008 (UTC)[reply]
When you are moving forward on the bicycle, and begin to tilt to one side you use the handlebars to stop the fall by turning the bicycle that way. When you are stationary it does not help! Graeme Bartlett (talk) 22:04, 7 May 2008 (UTC)[reply]
I think that's mostly true for low speeds, but once you get going fast enough you don't even need to touch the handlebars. --Duk 03:01, 8 May 2008 (UTC)[reply]
But that's still not due to the gyroscope effect. You may be right for motorcycles -- their wheels and tires may have enough moment of inertia, and be rotating fast enough, to gyro-stabilize the bike. But for bicycles it just doesn't contribute much. The tires are intentionally light (because you have to spin them up with your own muscles -- you wouldn't want to do that to a motorcycle tire). Rather, the self-righting effect comes from the geometry of the bike, the way the force from the road tends to act on your front wheel to move it back to the center. --Trovatore (talk) 19:50, 8 May 2008 (UTC)[reply]

Anatomy and physiology

Describe the role of the thymus gland and the Gut-associated tissue (GALT) in the establishment of active immunity —Preceding unsigned comment added by ELVIOTA (talkcontribs) 23:43, 6 May 2008 (UTC)[reply]

This looks like a homework question, as as it says at the top of the page, we will not do your homework for you. If you have a specific section you're having trouble with, and you have made an effort to do it youself, THEN we will help. -mattbuck (Talk) 23:50, 6 May 2008 (UTC)[reply]
Have you read our article on Thymus - that would certainly be the place to start. --Tango (talk) 23:50, 6 May 2008 (UTC)[reply]
For the other part look at Gastrointestinal tract and follow the links there.71.236.23.111 (talk) 01:58, 7 May 2008 (UTC)[reply]

May 7

degree of benefit

Where might I obtain the data to determine if the installation of Internet capable water meters might reduce fuel consumption by various utility suppliers? 71.100.14.205 (talk) 01:08, 7 May 2008 (UTC)[reply]

You could try asking the suppliers. Otherwise, try googling for some study into them. --Tango (talk) 12:41, 7 May 2008 (UTC)[reply]

Infinity of space

Today my Physics teacher was discussing how the universe can go on forever or it can be like a snowglobe, and no one can know what's outside the globe. I'm extremely intrigued by the unknown factors in this and was wondering if someone could point me in the right direction to go read more into this (typing in "past what we think is the universe" didn't get me anywhere haha). Thank you! Evaunit♥666♥ 02:09, 7 May 2008 (UTC)[reply]

We've got an article on Shape of the universe, but it gets a bit technical. There's also the issue of the observable universe versus the "whole" universe. The problem about talking about things "outside the universe" or "before the beginning of the universe" is that all of our physics is based on observations inside the universe, so it's hard to say that they apply to things outside that. Confusing Manifestation(Say hi!) 03:22, 7 May 2008 (UTC)[reply]
I think your teacher is talking about the Ultimate fate of the universe and whether the universe will continue to expand such that the energy density approaches zero (infinite universe i.e. Heat death of the universe), or will it contract such that the energy density approaches infinity (reaches a maximum "snow globe" i.e. Big Crunch). Information cannot travel faster than light so the snowglobe is the size of light traveling since the big bang and there should always be a boundary, even in the collapsing model. --DHeyward (talk) 06:04, 7 May 2008 (UTC)[reply]
Data in the past decade seem to be pointing to a hyperbolic, infinite universe that does not contain enough mass to stop expansion. Imagine Reason (talk) 20:00, 7 May 2008 (UTC)[reply]
One weird idea that alot of people seem to have trouble with is that although the universe may have a contained shape, that doesn't mean that anything lies outside the shape. When physicicsts talk about "the universe," they are attempting to describe everything that exists. If the place where everything exists is in some dimension spherical (which it's probably not, by the way), then nothing lies outside of that sphere because the sphere just is everything that exists. Contemporary scientific theory has raised the possibilty of multiple universes, so maybe this should be qualified as "everything that exists for us" or as Confusing Manifestation pointed out "the observable universe." --Shaggorama (talk) 20:09, 8 May 2008 (UTC)[reply]

chemical, mineral, vitamin make-up of menstrual blood?

I have read that menstrual blood contains all the contents of regular blood, such as plasma, but doesn't it also contain vitamins and minerals and growth hormones? Also I just read of an English firm that is recruiting women to submit their menstrual blood because it contains stem cells. A landscaper friend of mine regularly uses birth control pills to make all his bedding plants look spiffy, which implies that mammalian hormones stimulate plants- and perhaps plant hormones stimulate people?- (Spring fever and all that) Really, I am just looking for a layman's list of the contents of menstrual blood.People777 (talk) 05:34, 7 May 2008 (UTC)[reply]

Menstrual "blood" is a misnomer: what is shed during menstruation is the lining of the endometrium. "Menstrual fluid" is closer to correct. There's blood mixed in there, but it's not the main constituent. I don't know where you'd find an analysis of it, but I'm sure it contains as many vitamins or minerals as you'd find in any body tissue. It certainly contains endometrial cells, cervical mucus and vaginal secretions. As to the birth control pills: plant hormones and human ones are similar; plant sterols and estrogen are synthesized by similar pathways. - Nunh-huh 07:14, 7 May 2008 (UTC) (actually, there are papers, but I don't know that there's online access: e.g. Am J Obstet Gynecol. 1965 May 15;92:183-8 14281826 (P,S,E,B) THE CONTENTS OF MENSTRUAL FLUID: AN ANALYSIS OF 260 SAMPLES FROM HUMAN FEMALES. M S BURNHILL, C H BIRNBERG[reply]
Have you looked at our Menstruation article? Blood as the body's transport medium contains many substances including vitamins. Phytoestrogens are plant substances that mimic estrogen. 71.236.23.111 (talk) 07:37, 7 May 2008 (UTC)[reply]

A new Vector Energy Interpretation of Quantum Mechanics - what to do?

This derives from a very thorough paper (external) that examines relativity if time were treated as a vector. To the best of my ability, the derivations look valid. Combined with the transactional interpretation one can derive an "offshoot" interpretation that treats a negative energy vector as a photon traveling backwards in time. The interpretation's main premise is that these photons are absorbed by empty space and since energy is added, must be conserved as in pair production (other things conserved too); or the tunnel effect being a particle in an energy well converting all mass/energy into a negative vector energy photon emission and disappears. Outside the energy barrier empty space absorbs a negative vector photon and the particle appears there.


The problem is that this can be used in several ways, all IMHO have merit.

One of these appears to be absurd at first; it is much deeper, though not many I feel will see the deeper significance. So I don't know what to do?

any help appreciated

reference user:fx303 page for the 'absurd' view. —Preceding unsigned comment added by 67.174.224.78 (talk) 06:07, 7 May 2008 (UTC)[reply]

Hello. Can you supply us with anything at all to read on the subject, or at least point to where/under what name this paper has been publicised? Scaller (talk) 14:42, 7 May 2008 (UTC)[reply]
I think the OP was referring to User:Fx303#Vector_Energy_Interpretation_of_Quantum_Mechanics. -- JSBillings 01:01, 8 May 2008 (UTC)[reply]

Amount of zinc gluconate in Zicam

I understand the formula is propietary, and I have been looking for the patent, but I was wondering how much zinc is actually in it in mg. This is due to problems of zinc destroying nasal tissue, although I'm not sure of the mechanism.

Thanks, JD139.225.81.128 (talk) 14:57, 7 May 2008 (UTC)[reply]

Could time be expanding?

Hi. Have physicists considered that time could be expanding like space is? Or does the standing definition of 'time' and the physics around it somehow make this impossible? What if time were expanding (somehow), but we just haven't noticed it (either in real life or theoretically)? I wish I could give more 'punch' to this possibility, but I'm no physics expert... Thanks in advance, Kreachure (talk) 16:44, 7 May 2008 (UTC)[reply]

I'm not an expert either but i'm pretty sure time is part of the whole space idea. If space expands, time should expand proportionally and it'd make no difference? Probably wrong, but hey, worth a shot! Regards, CycloneNimrodTalk? 16:59, 7 May 2008 (UTC)[reply]
We mostly define distance in terms of the time it takes light to travel over that distance. I suspect you could describe the expansion of the universe as mathematically equivalent to a static universe with a changing time dimension. As we live in the universe, if the two descriptions are equivalent (as I suspect) then it is probably easier for us mortals to assume the nature of time is not changing. Dragons flight (talk) 17:09, 7 May 2008 (UTC)[reply]
The big bang cosmological model doesn't contain anything that I'd describe as "expanding time". The concept doesn't make much sense because expansion is something that happens over time. When you say that something is expanding you mean it's smaller at an earlier time and larger at a later time. You could probably come up with some sense in which "time is expanding"—for example, the time you've been alive is "expanding" at a rate of one second per second—but probably nothing very interesting or meaningful. -- BenRG (talk) 17:28, 7 May 2008 (UTC)[reply]
Well, let's pull an Einstein and think about what time is: a measurement of how long it takes for some regular interval to happen. Let's say we had a light clock with a 1 second interval—that is, a photon bouncing between two mirrors and it takes it one second to get from one mirror to the other. Now let's say the space between the mirrors expands with space so it takes longer than 1 second for the photon to go over the space: do we redefine the second, or do we say that that particular clock no longer measures what it used to measure? I would think the latter. But I think the rub is that if all space was expanding evenly, you'd have no way of knowing, though: how would you know your clock no longer worked the same way it used to, without another clock to compare it to? If the same expansion of space affected that clock too, in exactly the same way, then in effect there'd be no way to determine the difference. If, however, the expansion was detectable in one way or another (which our current expansion is, because it affects different scales more than others), then it would be easy to say that the clock had just gotten inaccurate. Someone correct me if I'm reasoning totally wrong here. --98.217.8.46 (talk) 18:23, 7 May 2008 (UTC)[reply]
First you have to define time. In an attempt to be overly general (so don't respond with quotes out of your physics book to clarify the specifics), there are two views on time. One view is that time is a dimension (like length, width, and height) that contains events. Objects travel through the time dimension and measure it by the time distance between events. Another view is that time is not a dimension, nothing travels through it, and it does not contain events. It is merely a value placed by humans to make formulas work properly. If you use the first definition, it is possible to claim that the time dimension is expanding since it has the same properties as the other three dimensions. If you use the second definition, time cannot expand because it doesn't exist. -- kainaw 18:45, 7 May 2008 (UTC)[reply]
The definition of the term "expanding" breaks the symmetry between time and space. Even if time can somehow expand, it can't expand in the same way that space does. It's like asking whether space expands with distance. There's a weird circularity in the idea. You can think of time as an implicit dimension that parametrizes change, or you can think of it as an explicit dimension like space, but you can't have both at once. In the spacetime picture there's no implicit change. Objects don't move along their worldlines, they simply are their worldlines. Changing the past isn't a physical impossibility, it's a logical contradiction. (Actually that's true of either notion of time.) If the original question was whether the time dimension can expand over the course of some other (implicit) time interval, then the answer is no, that doesn't make sense. If the question was whether it can expand over the course of the same time that it itself measures, then I think the answer is still that it doesn't make sense. -- BenRG (talk) 20:12, 7 May 2008 (UTC)[reply]
To expand on my comment above. You generally describe the structure of the universe as a whole through the FLRW metric of spacetime:
Where the scale factor a(t) describe the expansion of space as a function of time. However, I am fairly confident that one could construct an equivalent metric:
where b(t) is a new scale factor, and yet I suspect that if you apply the Einstein equations and follow the construction through it could be designed to give rise to the same dynamical structure. The second metric would be described as having static spatial dimensions and a dynamic time dimension, but from the point of view of any quantities observable by people living in the universe they would give exactly the same behavior. Dragons flight (talk) 20:32, 7 May 2008 (UTC)[reply]
P.S. I also think it is possible to construct a system with both dynamic time and dynamic space, but I think there no physically useful reason for doing so as one can always condense all of the variation into one or the other. Dragons flight (talk) 20:39, 7 May 2008 (UTC)[reply]
More importantly, if time was expanding in this way (which I think it probably is), do you think we would be capable of noticing it? in relative terms, the amount of time we experience probably would stay the same, for the same reason we don't notice out bodies expanding as the universe does. relativity is a pretty elegant theory. --Shaggorama (talk) 10:11, 8 May 2008 (UTC)[reply]
Our bodies don't expand as the universe does. The expansion is defined in terms of real physical distances that we can measure; our measuring devices tell us that the universe is expanding at large scales but human bodies aren't. The trouble with the b(t) metric above is that it doesn't describe a measurable expansion the way the FLRW metric does. -- BenRG (talk) 18:05, 8 May 2008 (UTC)[reply]
No, this metric won't work. Make the substitution and it becomes
,
which is the FLRW metric with a(t ') = 1. The observed geometry of the universe requires a non-constant a. Also, b isn't actually a parameter, since you get the same geometry regardless of b. You could try making b a function of r instead of t. I think this would get you a family of geometries distinct from the FLRW geometries (in which case they still don't describe the real world), but I'm not entirely sure. -- BenRG (talk) 18:05, 8 May 2008 (UTC)[reply]
This does work, though: , if you take , i.e. the derivative of the inverse of a. (I'm writing for the big parenthesized part of the metric.) The only caveat is that a has to be invertible (and have no critical points), which rules out a recollapsing universe but doesn't rule out the Lambda-CDM model. For k > 0 the "picture" you get from this is a universe with the big bang at the center and later cosmological times represented by concentric spheres. Distances within the spheres are what you'd expect from a background Euclidean geometry, but distances from the center are timelike and scaled by b(t). So this is something like an expanding/contracting time picture of the FLRW universe. You're free to think of the universe this way, but I think the usual expansion picture is intuitively clearer. (The usual picture can even be understood in Newtonian terms—see this long thread.) -- BenRG (talk) 14:15, 10 May 2008 (UTC)[reply]

The Doolittle Raid

Why wouldn't the U.S. navy install booster rockets to the B-25s of the Doolittle Raid? Wasn't it cheap and proven technology? -- Toytoy (talk) 17:29, 7 May 2008 (UTC)[reply]

Perhaps a better question is "why would they?" Based on our JATO article, it doesn't appear the US had them at this point, and I doubt that they'd be classified as "cheap and proven" at that stage even if they did. Training for the raid had already established that B-25s should be launchable from a carrier, so why would the Navy and Army add extra unnecessary complexity? — Lomn 19:01, 7 May 2008 (UTC)[reply]
The rocket boosters, had they been a proven and available technology, might have allowed greater takeoff eweight, thereby allowing more fuel load (possibly via aux tanks) with greater range and thus the possibility to launch farther from Japan with greater safety for the carrier and less chance of premature detection, or with a greater bombload, or with a greater chance of making it on to a safe landing in China. Edison (talk) 19:43, 7 May 2008 (UTC)[reply]
As I recall (from the book version of Thirty Seconds Over Tokyo), the B-25s were already doing stuff like carrying jerry cans of fuel internally. While I concur that in principle with the higher weight -> more range -> safer mission principle, I don't know that any practical payoff was likely (bomb load vs range is a different matter, I guess, but meh -- it was a propaganda raid. 5% more bombs doesn't affect that). Regardless, a large part of the range problem was the premature launch of the mission due to possible detection, though my personal opinion from reading about this as well as the Flying Tigers is that the US was overly optimistic to expect these aircraft to successfully rebase out of China. — Lomn 19:59, 7 May 2008 (UTC)[reply]
Retrospective analysis can always find superior solutions. The Doolittle raid was cobbled together from available pieces in a very short time. With the benefit of hindsight, we can find all sorts of clever improvements. In your scenario, JATO units would have increased the bomb payloads, resulting in much more damage to Tokyo. but in an equally likely scenario, one or more JATO units would have failed catastrophically, causing distruction of a B-25 and renderingthe carrier incapable of launching other B-25s. -Arch dude (talk) 02:33, 8 May 2008 (UTC)[reply]

I can hardly think of a reason why they could not modify a military rocket for a mission like this. The army has been using ground-based rockets for many years. This proven technology shall be adapted for launching bombers on an aircraft carrier's deck. It can decrease the risks.

Installing jerrycans on a bomber seems to be ... well ... stupid. These cans are heavy and not very efficient. Why couldn't they make a large aluminium oil tank?

I find the jerrycan article funny. Allied scientists had to reverse engineer steel oil cans? Was it rocket science to make cheap and durable oil cans? Hitler ordered jerrycans secretly? Was it really such a big deal to make some oil cans for people? Couldn't the Germans make up a good excuse for making oil/water cans? e.g., "These cans are for delivery of cooking oil to the hospitals." -- Toytoy (talk) 12:44, 8 May 2008 (UTC)[reply]

Methinks you overestimate the ease of something like this and the likelihood of success. Getting rockets to work reliably was not an easy task; to do it with manned flight and without causing danger to the launching platform was simply not a risk worth taking in 1942. In any case, thinking backwards like this is a poor historical methodology—all you learn about is yourself, not the past.
As for the jerrycan, I think you also underestimate the materials science and good engineering that goes into that particular steel oil can. Think about it this way: if it was easy to do, they wouldn't have needed to reverse engineer it. They weren't any stupider than people today are. --98.217.8.46 (talk) 13:33, 8 May 2008 (UTC)[reply]
What ground-based rockets did the U.S. have in 1942? The German and the Russians deployed things like the Katyusha but these probably didn't have the power to accelerate a bomber significantly. The development of large rockets by the Germans during the war was a major advance. (And so we acquired their technology and personnel after the war.) See Rocket artillery#World War II, V2 rocket. Rmhermen (talk) 14:36, 8 May 2008 (UTC)[reply]
Digging deeply through our poorly organized material on Wikipedia, Aerojet tested a U.S. design for a JATO rocket in August 1941 but it didn't go into production until 1943. Elsewhere I found, that in the 1941 test, it took 12 rockets to launch a 1,260 lb max weight plane. A loaded B-25 weighed 33,510 lb so we only need to attach about 320 rockets to each bomber! In April of 1942 (the same month as Doolittle), the U.S. began testing a liquid-fueled JATO on a bomber smaller than the B-25; however, development was abandoned in 1944. Rmhermen (talk) 14:54, 8 May 2008 (UTC)[reply]

Proving Technology for Aerospace Applications

Hi all, does anyone know what sort of tests are required to prove a new technology or material for use on aeroplanes or spacecraft? And what issues need to be considered? So far I've considered:

  • An ability to be implemented consistently, reliably
  • Withstanding repeated exposure to the pressure and temperature changes likely to be experienced
  • A knowledge of how to detect flaws and failings in the material or technology.

I have also found, British Aerospace EAP an example of testing aerospace technology.

This is part of revision for an exam coming up. If anyone has any thoughts, knowledge or ideas I'd be very grateful. Many thanks, LHMike (talk) 18:43, 7 May 2008 (UTC)[reply]

Technology Readiness Level has some information on American standards Mad031683 (talk) 19:49, 7 May 2008 (UTC)[reply]
Thanks, that's really helpful. Also found Mature technology linked. LHMike (talk) 00:26, 8 May 2008 (UTC)[reply]

Biology

Can human polyspermy end as a viable embryo (e.g. by mixoploidy etc.)? Eliko (talk) 19:52, 7 May 2008 (UTC)[reply]

Sort of. Such an egg will convert into a pair of diploid zygotes, however. It was never clear to me how that actually happened. Someguy1221 (talk) 23:52, 8 May 2008 (UTC)[reply]
Let me quote from the article Polyploidy:
Triploidy, usually due to polyspermy, occurs in about 2-3% of all human pregnancies and ~15% of miscarriages. The vast majority of triploid conceptions end as miscarriage, and those that do survive to term typically die shortly after birth. In some cases survival past birth may occur longer if there is mixoploidy with both a diploid and a triploid cell population present.
So I'm really eager to know about any source for your statement: Such an egg will convert into a pair of diploid zygotes.
Eliko (talk) 07:28, 9 May 2008 (UTC)[reply]
I'll just say that Nature said it. I could hunt down their own two references, but it's kind of late, so I think I'm out for the night. Someguy1221 (talk) 09:27, 9 May 2008 (UTC)[reply]
And reading the Nature article again, I'll take that partially back. It is never stated that the egg splits into a pair of zygotes, merely that it forms a chimeric diploid embryo that later produces twins. Someguy1221 (talk) 09:29, 9 May 2008 (UTC)[reply]
Thank you for the source and for the news, it really helped me. By the way, this source does not indicate that polyspermy "forms" a chimeric diploid embryo - i.e. as a usual result of polyspermy; it just says that a chimeric diploid emryo - may be a "way" (i.e. a possible "way" out of some possible different results) of polyspermy. Please pay attention to the word "way" (in the report summary). Anyways, thank you again for the news: My original question - whether polyspermy can end as a viable embryo - has been answered. Eliko (talk) 13:10, 9 May 2008 (UTC)[reply]
Oh yes, I realized that after reading it again, last night. But why yearn for ultra-accuracy with my wording when I trust you're going to double-check the source anyway ;-) Someguy1221 (talk) 17:19, 9 May 2008 (UTC)[reply]

Swings and insurance liability

I've seen in other countries swings on which children stand up. I see only swing seats here in America. Does this have to do with the rate of injuries? I find the seats rather boring. Imagine Reason (talk) 20:08, 7 May 2008 (UTC)[reply]

That doesn't sound like a science question to me... just google it and see if you can find some studies. I doubt anyone here just happens to be an expert on playground accident statistics (although anything is possible...). --Tango (talk) 22:16, 7 May 2008 (UTC)[reply]
You can stand up on any swing with a hard seat. You've never seen a swing with a wooden plank for a seat? --Shaggorama (talk) 10:12, 8 May 2008 (UTC)[reply]
Not in public in America you won't (see any swings with hard seats). The legal threat posed by such a deadly item is just too much! So very uncomfortable, not very swingable "sling seats" are all you'll see.
Atlant (talk) 13:14, 8 May 2008 (UTC)[reply]
I never had any trouble standing up on a sling seat. --Carnildo (talk) 20:52, 8 May 2008 (UTC)[reply]
I didn't know if any desk would be appropriate for this question, and I'd watchlisted this page, so...anyway, if I've seen planks in America, I've forgotten about it. I never saw any seats in Asia, either. I did try standing on the seats, but couldn't stand on their round bottoms. Imagine Reason (talk) 01:02, 9 May 2008 (UTC)[reply]

Are old-growth forests being cleared faster than they regenerate?

My question is specifically directed towards the forests in British Columbia, where environmentalists (namely Greenpeace) have targeted most. I know that the UN's Forest and Agriculture Organization (FAO) reports that Canada has maintained over 400,000 hectares of forest cover, but how much of that is old-growth? Does that amount of cover provide old-growth forests to regenerate by the time they are logged again? —Akrabbimtalk 22:06, 7 May 2008 (UTC)[reply]

Although the term Old growth forest is somewhat disputed, from what I know it is generally agreed that once it has been cut down it's gone. What you replant is a managed tree lot usually called Secondary forest. Certain features of old growth forest like natural selection of species growth pattern, influence of dead trees and fallen old logs, established drainage patterns and natural territory boundaries are not re-created once the old growth has been logged. UW researchers studied Mount St. Helens after the blast [13] what is not mentioned in this article is, the study plot that was cleared of fallen and dead logs and replanted actually recovered slower than the plots left to their own devices. Some forestry managers now use limited controlled fires to control underbrush, because they found that dousing all fires just made the next fire bigger and more dangerous. Sustainable forestry is no substitute for old growth because it just means that loggers don't take out more trees than they grow. It's like taking out a creek and putting a concrete lined canal there instead. The water still flows but many other factors can not be re-created. On a positive note, some areas of South American rain forest used to be farmed by natives in ancient times and are now almost indistinguishable from undisturbed growth. We just won't leave a forest alone for a couple of centuries for "old growth" to develop again. Hope this will help with answering your question. --71.236.23.111 (talk) 23:52, 7 May 2008 (UTC)[reply]
Does that mean that if and only we continue sustainable forestry for a couple of centuries, the answer to this question becomes yes? Of course, that doesn't address the amount of old-growth forests there would be. If we continued to cut down trees at our current rate, and always cut down a certain portion of old-growth trees (say, half the ones we cut down are old growth, regardless of how much old growth is left), then I'm willing to bet the answer would be "none". Of course, centuries from now, humanity would probably either be advanced enough for this make an unnoticeable difference, or extinct. Either way, the only practical time frame is short enough for new old-growth to be, for all intents and purposes, nonexistent. — DanielLC 00:01, 9 May 2008 (UTC)[reply]
Nope only if you'd leave your replanted forest alone it "might" happen. What you plant is all the same species and with little genetic variation at that. Trees are planted at regular intervals to enable easy access for machines. They are all the same age. The underbrush is gone. The soil is all stirred up and topsy-turvy. Roads cut through plots depending on local conditions they can either create a wall for water-flow patterns and animal and plant dispersal or a highway for floodwater and opportunistic species (or both). Some modern forestry techniques are trying to copy pages from nature's book, like planting a mix of species, but still while we meddle it won't become old growth. For that to happen you'd have to maybe airlift individual trees out and only cut one every mile or so every 50 to 1oo years maybe. (You might be able to get away with closer together and more often or you might need to do it farther apart and less often.) The trees that you take out would not be available as fallen trunks, but a system can and does usually cope with a little bit of interference. Sustainable forestry just means that there is a tree growing for a tree you cut down. It's as though you'd take the Mona Lisa out of the museum and instead would hang a photograph there. Eventually, in a hundred years or so that photograph might also become art. If it survives a couple of hundred years more it might even become unique art and deserve a place next to the old masters, but if you hang a new picture in the frame every couple of years that's never going to happen. Lisa4edit (talk) 20:48, 9 May 2008 (UTC)[reply]
Based on how things work with farming tribes in tropical rainforests, you could sustainably harvest an old-growth forest by cutting one-acre patches amounting to about 0.25%-0.5% of the forest each year. --Carnildo (talk) 21:20, 9 May 2008 (UTC)[reply]

Question: What is the compound/name of the chelate of calcium produced by plants?

As chlorophyl is a chelate of magnesium produced by plants, I wish to know any chelate of calcium that is produced during the symbiotic relationship of plant roots and soil microbes. Best answered by a SOIL BIOLOGIST.

I have asked several chemists this question and they always say, "EDTA." We are talking about organic chemistry here. Engintinc (talk) 22:50, 7 May 2008 (UTC)[reply]

Well, EDTA certainly will chelate calcium ions, but it's a synthetic molecule, and not really something made by plants. (And if you're interested in calcium specifically, EGTA may be a better bet for a synthetic chelator.) For natural compounds, you're probably looking for a calcium equivalent to siderophore. Unfortunately, I am unaware of any such molecules, although they certainly may exist. -- 128.104.112.85 (talk) 14:37, 8 May 2008 (UTC)[reply]

Conservation of angular momentum

During a physics lab on the conservation of angular momentum, we were asked to sit on a chair could rotate effectively frictionlessly. After firmly placing our feet on the ground, someone spun the wheel, and then we were asked to flip the wheel. Now, as per the conservation of angular momentum, it is expected that we would spin in the opposite direction, and that is what we observed. However, what I don`t understand is what causes the person in the chair to spin. To illustrate my point, let us consider linear momentum. Linear momentum, like angular momentum, is conserved if the sum of the external forces on the system is zero. Now, force is by definition what causes a change in momentum, so the fact that momentum is conserved if there are no external forces is fairly obvious. But, when a stationary object is hit by a moving object, the fact that the stationary object moves is because not because momentum is conserved, but because there is an internal force acting on the object. So, my question is this: when considering angular momentum, what causes the person in the chair to spin? It must be a torque (the rotational equivalent of force). But what is the source of the torque? Also, a perhaps related question, why is it hard to flip a spinning wheel? Can how hard it is to spin be calculated?


Could you please re-explain the scenario in your lab? I'm having trouble following it. Oh, and please remember to sign your posts using four tildes (~~~~). -mattbuck (Talk) 23:22, 7 May 2008 (UTC)[reply]
You're sitting in a chair that can spin, and you're holding a wheel (the wheel can spin; it has handles of sorts going through the center of the wheel). Someone spins the wheel (it is parallel to the floor), and the you flip it 180 degrees so that it's spinning in the opposite direction. Afterwards, you begin to spin. —Preceding unsigned comment added by 76.69.240.138 (talk) 23:30, 7 May 2008 (UTC)[reply]
I'm pretty sure it's Newton's 3rd law - equal and opposite forces. The spinning wheel has angular momentum, and when you flip it, the torque you provide changes the direction of the angular motion 180 degrees, so you receive an equal force in the opposite direction, causing you to spin, conserving (angular) momentum. —Akrabbimtalk 23:38, 7 May 2008 (UTC)[reply]
Yeah, the force making you spin is the reactive force to you moving the wheel. It's the same basic principle as rocket propulsion - sit on the same chair with a heavy weight and throw it away from you and the chair will move backwards. This is just the rotational equivalent. --Tango (talk) 00:35, 8 May 2008 (UTC)[reply]
Okay, I think I get it. But why is it hard to flip the wheel, and how can this be calculated? —Preceding unsigned comment added by 76.69.240.138 (talk) 00:53, 8 May 2008 (UTC)[reply]
You can think of it as the rotational analogue of Newton's 3rd law - essentially, when you try to flip the wheel, you are applying a torque to the wheel (torque is a change in momentum over time, just like force is a change in momentum over time). The wheel applies an equal and opposite torque to you! You may need a pencil and paper to see this properly - the angular momentum vector of the wheel is initially vertical. As you flip the wheel, you're changing the angular momentum vector, and the torque is in the direction of that change; as you flip the wheel, there is a torque with some component in the vertical direction, and the equal and opposite torque on you and your chair produces your angular acceleration. It is "hard" to flip the wheel for the same reason it's "hard" to throw the wheel across the room to give yourself linear momentum - both you and the wheel have rotational inertia! Using the equation in our article on precession (look in the torque-induced precession section), the net torque causing you to spin will be where is the angular speed of your flipping motion, is the spin angular speed of the wheel, is the angular moment of the wheel, and is the angle of the axis of the wheel to the vertical. It is possible to think of the forces "microscopically", i.e. in terms of the forces on each part of the wheel but it is pretty tedious in my opinion. If you want to see that treatement, there's a decent one in the aforementioned article on precession. --Bmk (talk) 01:30, 8 May 2008 (UTC)[reply]
Got it! Thanks a lot. —Preceding unsigned comment added by 76.69.240.138 (talk) 02:49, 8 May 2008 (UTC)[reply]

Momentum of a photon

How can a photon, which is massless, have momentum? —Preceding unsigned comment added by 76.69.240.138 (talk) 23:14, 7 May 2008 (UTC)[reply]

A photon has zero rest mass. However, under special relativity, accelerating particles gain mass. If you were to accelerate a massive particle to the speed of light, its mass would become infinite. Hence it is necessary for a photon to be "massless" for it to be able to travel at the speed of light. Confusing Manifestation(Say hi!) 23:26, 7 May 2008 (UTC)[reply]
You've probably heard of the equation "E=mc2". It describes an equivalence between mass and energy - that equivalence goes a further than that simple formula (which is the special case for objects at rest - a photon isn't at rest, since it always moves at the speed of light). While a photon has zero rest mass, it has a positive energy, and that gives it a momentum. That momentum (in a vacuum, at least) is where h is the Plank constant and lambda is the wavelength. --Tango (talk) 23:32, 7 May 2008 (UTC)[reply]
I never thought about that. that's messed up. --Shaggorama (talk) 10:14, 8 May 2008 (UTC)[reply]
Relativity can be rather counter intuitive... For example, even though photons are massless, they are still affected by gravity in the same way as anything else (a beam of light on Earth will fall at 9.81m/s/s - it's hard to tell, since it's out of sight before it's fallen a noticeable amount, but it does fall). --Tango (talk) 13:28, 8 May 2008 (UTC)[reply]
[repost from old question] Be careful with that equivalence! As described at general relativity, light suffers twice the deflection that one would naïvely expect from analogy with a fast rock. --Tardis (talk) 15:30, 8 May 2008 (UTC)[reply]
Interesting... what causes the difference between a very light particle travelling at 0.999999c and a photon? --Tango (talk) 20:18, 8 May 2008 (UTC)[reply]
all I can think of is that maybe light is refracted by the gravitational time dilation in addition to the distortion of space.Em3ryguy (talk) 21:26, 8 May 2008 (UTC)[reply]
I'm confused. I was told once that it was one half the expected value. Also you might find this interesting-http://www.experiencefestival.com/a/Newtons_aether_model_-_Modernised_Newtonian_theory/id/5333093Em3ryguy (talk) 20:41, 8 May 2008 (UTC)[reply]
It's twice, but that only applies at large scales where the whole gravitational field is involved. In local experiments where the field is uniform, light falls at 9.8 m/s² just like everything else. (You can see this from the equivalence principle: imagine doing the same experiment on a rocket ship accelerating at one gee.) The extra global deflection comes from the fact that the circumference of a circle around a gravitating object is less than π times the diameter. Here's an analogy: take a circular sheet of paper with a pie-shaped wedge missing. Draw a straight line which intersects one edge of the missing wedge and a second parallel line which intersects the other edge. Pull the edges together, making a cone, and extend the lines across the seam in as straight a fashion as possible. Going around opposite sides of the cone has caused them to converge. This deflection is (in a certain sense) independent of the speed of the object, whereas the "ordinary" deflection of course depends on the speed. At light speed they have the same magnitude; at 0.999999c they have almost the same magnitude; at small speeds the "ordinary" deflection dominates. -- BenRG (talk) 01:14, 9 May 2008 (UTC)[reply]
That makes sense. I thought it odd that a photon would behave significantly differently to a very fast moving light massive particle... --Tango (talk) 18:07, 9 May 2008 (UTC)[reply]
In special relativity the relation between energy and momentum is E² = (pc)² + (mc²)², where m is the rest mass. When people say the photon is "massless" they mean that m = 0, which implies that E = |p| c. Both E and p are always nonzero. -- BenRG (talk) 01:14, 9 May 2008 (UTC)[reply]

centripetal motion of car

I was given a question where a car of known mass was turning on a circular road of known radius and coefficient of static friction. The question then asks what's the maximum velocity of the car. That wasn't to difficult, but I have a theoretical question. In this question, I treated the centripetal force as the static friction force, but for this to work the static friction force should be pointing to the center of the road. But it's pointing along the path of the car. Then, how can static friction cause centripetal motion. Wouldn't we need the angle of the tires to treat friction vectorally? —Preceding unsigned comment added by 76.69.240.138 (talk) 23:25, 7 May 2008 (UTC)[reply]

The scope of the question seems to be assuming that the tires are rolling ideally, that is to say no slipping. The torque the axle provides causes the tire to push backwards on the road. Since the wheel is not slipping, that force is solely static friction, as the rubber is not necessarily moving in relation to the road, as each infinitesimal point of the tire touches down on the road in an instant, pushes back, and then is removed from the road, as the tire rolls. —Akrabbimtalk 23:44, 7 May 2008 (UTC)[reply]
I don't think you've understood my question; perhaps I was unclear. I know why the car moves. It is a simple application of Newton's third law. My question is why we treat the centripetal force as equal to μ*m*g. After all, isn't the friction force not directed at the center of the circle of motion. —Preceding unsigned comment added by 76.69.240.138 (talk) 00:18, 8 May 2008 (UTC)[reply]
I think it has to do with the fact that the tires aren't pointing in the direction of travel. I'm not sure about the details, though... My attempt to work it out ended up with the car going in the wrong direction, so I'm clearly missing something... --Tango (talk) 00:31, 8 May 2008 (UTC)[reply]
The car is observed to be moving in a circle at constant speed. Therefore, the car is constantly accelerating toward the center of the circle. Therefore, there a is force acting on the car and the force is acting in the diretion of the center of the circle. The only force acting on the car (in a pure gedankenexperiment) is the friction between the roadway and the tires. Therefore, the direction of this force must be toward the center of the circle. Conclusion: the vector of the force is in this case perpendicular to the direction of motion. -Arch dude (talk) 02:03, 8 May 2008 (UTC)[reply]
From this, we can see that your "obvious" assertion (that the friction force is in the direction of motion), is incorrect. -Arch dude (talk) 02:12, 8 May 2008 (UTC)[reply]
There is no motion. In static friction, the two objects touching (the bottom of the wheels and the ground) remain stationary compared to each other. The rest of the car isn't touching the ground, so its motion is irrelevant. The friction force is in the opposite of the force acting upon it (centrifugal force). — DanielLC 23:48, 8 May 2008 (UTC)[reply]

May 8

Taste vs. Smell

I read our articles on "the 5 senses."

When i get sick, I can't really taste anything. Does this mean that taste is a sub-set of the sense of smell? Are the two linked in some way? If I smell something foul, am I also tasting particles of it?

Can those without a sense of smell still taste, what about the inverse?

Thanks-- —Preceding unsigned comment added by 24.218.24.148 (talk) 00:22, 8 May 2008 (UTC)[reply]

Apparently if you eat an orange with a banana under your nose, the orange tastes like a banana. Don't know where I read that, sorry. LHMike (talk) 00:28, 8 May 2008 (UTC)[reply]
There is definitely a strong relationship between taste and smell. Just try eating something while holding your nose. I don't know the details, though, really... They are the same basic thing, though - both involve detecting certain molecules, either in the air or in things we eat and drink. --Tango (talk) 00:30, 8 May 2008 (UTC)[reply]
The taste+smell thing is called flavor. See the article for loads of info. — Kieff | Talk 01:43, 8 May 2008 (UTC)[reply]
You may be interested in a recent profile of noted chef Grant Achatz, who has partially lost his sense of taste due to cancer. The article discusses the relationship of scent and taste in some detail. -- Coneslayer (talk) 17:56, 8 May 2008 (UTC)[reply]
Smell and taste are both chemical detection methods. While smell primarily works on gases, taste primarily works on liquids. There's lot of overlap though, as many liquids also evaporate to produce fragrant gases, and strong odors also contain gases that can dissolve in the saliva on the tongue. StuRat (talk) 02:03, 9 May 2008 (UTC)[reply]

The smell of any substance is recognized by the Olfactory lobe of the brain. The tiny particles from the smelling substance reach the sensors and trigger some particular chemical reactions. On the other hand taste is perceived by taste buds on the tongue surface. But, generally, we recognize any food items by the combined signals of its smell and taste, and memorise the feeling as taste of the substance. It is like matching any object by the use of two different attributes together (e.g. if something is liquide and white in colour, we think it may be milk). However, when one falls ill (specifically cold and caugh), the smell of the food cannot reach the sensors and hence, the individual fails to get the combined sugnal, what he or she recognises as taste of the materials and it appears tasteless to him/her. Also, sometimes medication with antibiotic drugs affects taste buds and everything appears to have a metallic taste. Dr.Rajarshi (talk) 10:33, 9 May 2008 (UTC)[reply]

Practically speaking, "smell" only samples gases and "taste" liquids because we happen to sniff with our noses and drink with out mouths. There are few bio-molecular reasons that non-volatile liquids cannot stimulate our smell receptors and volatiles our taste receptors. Indeed, in many species animals will sniff up liquid droplets and thereby smell liquids directly, some even have a distinct olfactory sub-organ for doings so.
As others say, you can both taste without smelling and smell without tasting. Losing one does not alter the detection of the other type of chemosensation. But if you lose one, then you lose the ability to determine flavor, which is the perception we assign to the combined sensation of taste + smell. Flavor is what what we really mean when we talk about the "taste" of something in general terms. This is why it appears we lose taste when we can't smell, because our sense of smell contributes a lot more to the flavour that we generally think. In contrast, losing the ability to taste doesn't have too much effect on smell, because we often smell things without tasting them, but rarely taste things without smelling them.
Incidentally the olfactory bulb (or lobe) doesn't recognize smells per se. The odors are detected by the olfactory epithelium, processed by the olfactory bulb and projected to cortical brain regions, which does the recognition and attribution. Rockpocket 01:38, 10 May 2008 (UTC)[reply]
Just try eating bittermelon without tasting at least some of its bitterness. Except in seniors the bitter taste is like 1000x stronger than the sweet taste sensors or something like that. Thanks. ~AH1(TCU) 01:57, 10 May 2008 (UTC)[reply]

Decibels

I read about a set of billboards that show the noise level in decibels of the surrounding area. From an acoustics class I took, I remembered there were a few ways to measure sound levels in decibels. After reviewing the relevant articles in Wikipedia, I deduced the decibel numbers on the billboards are likely sound pressure levels, maybe unweighted or using A-weighting.

When measuring street noise, how large of a difference would there be between the different weightings?

Also in terms of street noise, how much does the location of the sensor affect the decibel reading?

--Bavi H (talk) 00:45, 8 May 2008 (UTC)[reply]

dB (decibel) is a ratio, not an absolute measure of sound. Thus, it is valid to say that "ambient sound increased by 20dB" (i.e, there is 100 times more acoustic power) without specifying any particular absolute level. To specify an abolute level, you must relate the level to a particular absolute standard: 0dB(SPL) is the smallest (absolute) acoustical energy perceptible to a (statistically average) human, and 20dB(SPL) is 100 times as much acoustical energy. The various alternative measures differ in the ways they assign weights to the frequencies in the acoustic spectrum. Each such weighting is a convention that derives from some perspective, not a mathematical or physical fact. The only objective measure of sound at a particular location would be a spectral graph, and even that is only valid for a particular integration period.-Arch dude (talk) 01:35, 8 May 2008 (UTC)[reply]
To answer your question on the different readings you would get with different weightings for street noise - you cannot say because it depends on the exact frequency content of the noise in that particular street at that particular time. Even with unweighted measurements, it makes a difference what frequency range the measurement is filtered to, but again, this cannot be enumerated without reference to the spectrum of the particualr sound you are measuring. SpinningSpark 21:29, 8 May 2008 (UTC)[reply]

Expanding Universe

Thanks everyone for the help so far! But there's still something I'm missing: if the universe is expanding, what is in the space before the universe expands into it? Is it impossible to know how far that area on non-universe goes or if there's something on the other side? Evaunit♥666♥ 00:52, 8 May 2008 (UTC)[reply]

Generally, the Metric expansion of space is defined by the objects in the universe, and the distance between objects. Said simply, the expanding universe isn't expanding into anything, it is simply increasing the distance between objects. -- JSBillings 00:58, 8 May 2008 (UTC)[reply]
Take a non-inflated balloon. Draw some dots on it. Now blow it up and watch the dots - they move apart as the balloon expands. There isn't an edge of the balloon (considered as just a 2D surface) that's expanding into anything, the expansion is caused by the balloon itself stretching at every point. The universe is the same, just with one more dimension. The only problem with the analogy is that the balloon is a 2D surface existing inside our 3D world - the universe doesn't exist within anything, it simply is, so you need to ignore all of the world except the balloon. Consider what an ant living on the balloon would see. --Tango (talk) 02:04, 8 May 2008 (UTC)[reply]
Here's a very concrete thought experiment you can use to understand this concept: Send an astronaut somewhere into space with a stopwatch and a flashlight, and put a mirror very far away from him/her, such that the astronaut and the mirror are stationary with respect to each other. Have the astronaut shine the flashlight at the mirror, and time how long it takes for the light to make the round trip. When cosmologists say "space is expanding", it means that if the astronaut repeats the experiment over time, the astronaut will measure that the round trip time is increasing!! There are a few tricky issues about making the mirror and astronaut "stationary", but they aren't too difficult to figure out. That's basically all we know about this concept of "expanding space". It's anyone's guess what it really means - physicists have come up with an intuitive description of the concept, and a comprehensive mathematical formulism (see general relativity) that seems to describe it. -Bmk (talk) 02:23, 8 May 2008 (UTC)[reply]
PS: They have to be very far apart wait a very long time in between trials for a measurable effect (i.e. not on humanly accessible distances or time scales). --Bmk (talk) 04:48, 8 May 2008 (UTC)[reply]
Mu (negative) --Shaggorama (talk) 10:17, 8 May 2008 (UTC)[reply]

Centrifuge

Seeing as we are now in the era of unmanned space travel for a while, why would it not be a good idea to build a centrifuge in orbit around the earth and use it to catapult unmanned space ships at extreme velocity, thus shortening the time for travel to other planets? Also, as we will be re-entering the 'manned space race to unearthly bodies' in the near future, what effect would this system have on astronauts, considering they are already in practically zero gravity? Would the 9Gs or more affect them? My question is, in a zero gravity environment, how would this work, if at all?--ChokinBako (talk) 03:30, 8 May 2008 (UTC)[reply]

A centrifuge, flinging a ship with that much velocity, would itself be flung back towards to Earth with tremendous speed, would it not? I can't envision how this will work... 206.126.163.20 (talk) 03:37, 8 May 2008 (UTC)[reply]
Well, you could fling two similar ships in opposite directions without altering the centrifuge station's trajectory much, but I don't really see how this system could become more feasible than rockets. Perhaps it could be valuable for unmanned missions, but according to our article on g-force, NASA recommends limiting human acceleration to around 10 Gs for a few minutes of exposure. A centrifuge that is - let's be very generous - 500 m in radius (that's a kilometer in diameter!) with a centripetal acceleration at the edge of 10 Gs would only send a spaceship flying off at 220 m/s (that's 500 mph), which is not terribly impressive considering unmanned spacecraft going to mars average speeds in the 30,000 m/s (75,000 mph) range. Unfortunately, the speed goes up with the square root of radius and square root of acceleration, so if you double the permissible acceleration and double the radius (which seems improbable to me), you only get 1000 mph exit velocity. I think moon-based linear accelerators (using a rail gun or something) for unmanned craft is more feasible. --Bmk (talk) 04:38, 8 May 2008 (UTC)[reply]
Additionally, how are you putting the spacecraft on the centrifuge to begin with? It seems that your options are to either stop and restart the centrifuge or accelerate the craft up to centrifuge speed, neither of which suggests an advantage over just launching the craft normally. — Lomn 12:15, 8 May 2008 (UTC)[reply]
You could load them in at the center, then feed them down the length of the arm from there. Also the 2 arms don't have to be the same length, one side could be shorter but more massive, but you still wouldn't even be able to get velocities out of it that would even be close to the orbital velocity of the centrifuge, and if Bmk's correct it would still be going less than escape velocity. Mad031683 (talk) 15:14, 8 May 2008 (UTC)[reply]
It might be easier to use a Gravitational slingshot, though I couldn't say for certain as I don't have much expertise in this area. AlmostReadytoFly (talk) 13:18, 8 May 2008 (UTC)[reply]
You'd need a booster to get into a trajectory around another planet or you couldn't get the angles right from what I gather, so no savings. This is from an old Discover Magazine article [14] "Meanwhile, mechanical engineer Stephen Canfield of Tennessee Technological University in Cooksville and others are investigating using a whiplike tether to hurl a probe from Earth orbit toward another planet, eliminating the need for a booster rocket. "The idea is to store up a lot of energy by spinning the tether. Then you release that energy by briefly capturing a payload on the tether's end," Canfield says. They also had another rotating system they proposed that tossed and caught loads until it reached escape velocity the concept drawing for it looked sort of like a ninja shuriken with only 2 prongs. (Can't find that one. I had filed it somewhere for later reading.) I admit I'm quite nebulous on the concept, too, but it must work out somehow. The only tether I know of that they tried in earth orbit fried because they hadn't thought they'd get that much juice from just dangling a wire and there was a problem with isolation. Can't figure out how they'd make use of the induction in the spinning tether but maybe the two are not related. You'd get less power in a lunar orbit, but you wouldn't have to deal with an atmosphere. 71.236.23.111 (talk) 21:56, 8 May 2008 (UTC)[reply]
An interesting feature of a space elevator is that, if you had one, you could also use it as a centrifugal launch platform: it has to extend some distance beyond geostationary orbit anyway, in order to provide a counterweight to the lower portion, so anything thrown off the top will get flung away from the Earth — the energy for this ultimately coming from the Earth's rotation. —Ilmari Karonen (talk) 12:44, 11 May 2008 (UTC)[reply]

Pls, helping to answer this question !?

What is the difference between macronutrients and micronutrients? Will the consequences of depriving a plant of a micronutrient be less severe than those of depriving the plant of a macronutrient? —Preceding unsigned comment added by 121.1.58.38 (talk) 03:51, 8 May 2008 (UTC)[reply]

I would recommend you read wikipedia's article on micronutrients. A macronutrient is simply a nutrient which is required in large quantities to sustain life, like water, carbon dioxide, and nitrates for plants. It's hard to answer your second question precisely - can you be more specific? I would suspect removing water or air supply will kill a plant much more quickly than removal of the supply of most micronutrients. --Bmk (talk) 04:22, 8 May 2008 (UTC)[reply]

First of all I would like to differ and negate the answer given above. It was wrongly dealt with. However, to answer your question, both micro and macro nutrients can be defined as essetial elements required for the plant growth and defficiency of which leads to certain metabolic disorders having definite symptoms. Only difference is that micronutrients are necessary in micro quantity while macro nutrients are necessary in large quantity for the plants growth. Carbon, Oxygen, Nitrogen, Hydrogen, Calcium, Magnaseum, Phosphorus, Potassium and Sulfur are macro nutrients, while Iron, Chlorine or Chloride, Zinc, Molybdenum, Manganese, Copper, Boron are major micronutrients. Sometimes Iron is also kept in the list of macro nutrients. So, difficiency of any of any of the nutrients will be similarly sever, and that will have some definite symptoms. —Preceding unsigned comment added by Dr.Rajarshi (talkcontribs) 05:14, 8 May 2008 (UTC)[reply]

I might be missing something here but the good doctor Rajararshi vehemently seeks to "differ and negate" from Bmk's reply, but then paraphrases it almost word for word. Bmk is wrong (I believe) to class air and water as nutrients, but apart from that their answers are much of a muchness Myles325a (talk) 03:09, 9 May 2008 (UTC)[reply]
Both answers agree on the definition of macro and micro nutrients. After that point, I think Bmk and Dr.Rajarshi are talking about 2 different things. I think they are both right.
Bmk talks about whether lack of a micronutrient will kill a plant. I think a plant with an adequate supply of micronutrients could live a long time if its supply of micronutrients was stopped (i.e, it did not get any more, but kept what it had.)
Dr.Rajarshi says that a deficiency in any nutrient, macro or micro, will cause definite symptoms. If a plant needs a particular nutrient to grow normally, lack of that nutrient will stop it from growing, or at least affect its growth.
That's my take on it. CBHA (talk) 04:47, 9 May 2008 (UTC)[reply]

The comment has flickered some controversies, which may not be desirable. But, the terms 'macronutrient' and 'micronutrient' are two definite scientific terminologies and those refer only to the nutrients listed above. There is no scopes left, that both the answers are correct. However, it is absolutely right that, if any of the nutrients (be it micro or macro) is necessary for growth, then deficiency of the same will affect the growth. It is also true that if the minimum amount of nutrient is available, then the proper growth and other functioning will continue, irrespective of the fact, whether it is being supplied from outside or not. Dr.Rajarshi (talk) 09:57, 9 May 2008 (UTC)[reply]

Organic Matter

Organic matter is not advisable to be added to paddy soils. Yes or No? Why? —Preceding unsigned comment added by 121.1.58.38 (talk) 03:59, 8 May 2008 (UTC)[reply]

Decomposition#Plant decomposition, Eutrophication and Hypoxia (environmental) might help. --71.236.23.111 (talk) 06:17, 8 May 2008 (UTC)[reply]

Pic

At 2:05 in [15], there's a picture of a cat with crazy blue eyes. Is that really possible? Black Carrot (talk) 05:23, 8 May 2008 (UTC)[reply]

No, it is photoshopped, to make the kitten look like someone who takes spice. --98.217.8.46 (talk) 20:22, 8 May 2008 (UTC)[reply]

Gingiva Recession

My gums are receding. My dentist says it's probably genetic. But I did not tell my dentist that my sister and mom have recession (see below).

My sister has recession, and I believe she has been flossing once and brushing twice a day. I never had recession, and my dental hygenine is the same. I think I am actually noticing recession. Is that possible? Because I haven't changed how I brush and floss to months ago, but it seems like my gums are receding faster. Also, my mom has recession, but she doesn't floss every day, but I think she at least brushes once a day.

Why can't recession be reversed, as in, why can't I build up bone growth again? How come I lose bone growth and can't gain it back? What are the causes of genetic gum recession? And have there been research to determine if bone can be rebuilt (in terms of gum recssion)? If not, why can't bone be rebuilt (in terms of gum recssion)? And what new research is being done to see if bone can be rebuilt? What are the new techniques? Thanks so much for all your help.68.148.164.166 (talk) 02:45, 7 May 2008 (UTC)68.148.164.166 (talk) 02:48, 7 May 2008 (UTC)68.148.164.166 (talk) 02:48, 7 May 2008 (UTC)[reply]

This is not medical advice and I'm no expert, but you might find more information on the following pages Receding gums, Periodontitis (look at the mention of risk factors there), Osteoimmunology As to your question why it can't be reversed (yet?) may be that despite surprising results (see Refdesk, "Humpty" May 2 ) our understanding of how cells work and what it takes to start and stop them from growing is still poorly understood. Particularly in dentistry, bone grafting and autologous (own) Stem cell treatments are areas with lots of studies on the way. Whether the results will be generally applicable, feasible and when they will reach common practice is another question. Lisa4edit (talk) 04:05, 7 May 2008 (UTC)[reply]
(see Refdesk, "Humpty" May 2 )???68.148.164.166 (talk) 09:40, 8 May 2008 (UTC)[reply]
Look particularly at the Scientific American article linked [16] I hope this works. If not click "Archive" above and look at May 2 in the Science desk archive. 71.236.23.111 (talk) 19:51, 8 May 2008 (UTC)[reply]

subliminal masseging

how to make subliminal messaging to subconcious mind —Preceding unsigned comment added by 117.197.48.133 (talk) 17:09, 8 May 2008 (UTC)[reply]

Subliminal message? Not sure what you're asking. --Ouzo (talk) 17:45, 8 May 2008 (UTC)[reply]
As a note, subliminal massaging would be, well, not that much fun, I imagine. Or would it? --98.217.8.46 (talk) 18:03, 8 May 2008 (UTC)[reply]
Really? Then why do you feel so good today? Nil Einne (talk) 19:34, 8 May 2008 (UTC)[reply]
Let's just get that nice spot right between your shoulder blades. Isn't that nice? Just relax, and let my hands take care of the work.
Huh? I don't know what you are talking about. There's no such thing. Obvious nonsense, that's it. --98.217.8.46 (talk) 20:24, 8 May 2008 (UTC)[reply]
It was once 'obvious nonsense' to propose the Earth was round. Don't dismiss things, unless you have proof to the contrary. Regards, CycloneNimrodTalk? 20:29, 8 May 2008 (UTC)[reply]
I think you missed the joke. --98.217.8.46 (talk) 02:48, 9 May 2008 (UTC)[reply]
Please then, share your subliminal massaging technique. 24.76.169.85 (talk) 20:52, 8 May 2008 (UTC)[reply]
Sorry to post spam, but actually, you can buy such a thing [17] SpinningSpark 20:56, 8 May 2008 (UTC)[reply]
Havat our article on Hypnosis. Julia Rossi (talk) 23:54, 8 May 2008 (UTC)[reply]

Validity of Sigmund Freud's work

From a scientific standpoint, how accurate or valid are Freud's theories? In particular, he introduced the world to the concepts of id, ego and super-ego, the unconscious, and others. Are these measurable? If so, how? If, not, why do they still hold such prominence in western society? —Preceding unsigned comment added by 72.35.21.126 (talk) 20:42, 8 May 2008 (UTC)[reply]

Psychology is a social science for a good reason, it studies empirically how people act but it, usually, doesn't relate all that much to biological function. Yes, we can detect things like depression by changes in chemicals and the like, but things like the ego are unmeasurable beyond visual observation and just listening. Regards, CycloneNimrodTalk? 20:56, 8 May 2008 (UTC)[reply]

There's a lot of current discussion about this, actually. There are some who argue that modern neuroscience has vindicated some aspects of Freud's work and some who are that it has disproved it. There are also those—largely psychoanalysts, I believe—who argue that Freud's description was not meant to be a literal biological description of the mind, but is a set of concepts that maps well onto how the mind functions, and thus serve as tools for affecting what, at some level, has to be the the neurological basis of behavior and emotion (analogous argument: the basic principles of engineering can be used without any knowledge of how they correspond with quantum mechanics; a reductionist approach can be unreliable on larger scales). And there are those who argue that it is a 19th century, overly literary approach to what should be a harder science. From what I can tell it is all over the board at the moment.

The question of prominence in Western society is a difference one, though, from the question of whether modern neuroscientists use Freud. Freud is wonderfully literary and his idea of deep seated drives, secrets, and internal struggle is one that has appealed to Americans in particular since the 1940s. The ideas triggered strong cultural valences in postwar America and still holds sway today, even though most people know jack squat about cognitive behavior and science. --98.217.8.46 (talk) 23:04, 8 May 2008 (UTC)[reply]

Dont forget the idea of psychological energy. The idea that mental processes are subject to 'forces' and 'energies' just like physical processes are. http://en.wikipedia.org/wiki/Psychic_energy. Em3ryguy (talk) 23:25, 8 May 2008 (UTC)[reply]

By the way, this article is a combination of 2 articles. 'psychological energy' which is scientific and 'phychic energy' which isnt. see the discussion page for comments. Em3ryguy (talk) 00:15, 9 May 2008 (UTC)[reply]
That entire article reads as though it's riddled with pseudoscience. It's especially hard to tell as it isn't well sourced. 206.126.163.20 (talk) 00:23, 9 May 2008 (UTC)[reply]
The whole point of psychological energy was to show that mental phenomena are subject to laws just like physical ones. Someone who didnt know what they were doing combined that article with the 'psychic energy' article and screwed it up. Em3ryguy (talk) 00:50, 9 May 2008 (UTC)[reply]
The original version of that article describes Freud's theory of energy as 'psychic energy' and directs the reader to that pseudoscience article. I dont believe that is accurate. Em3ryguy (talk) 01:04, 9 May 2008 (UTC)[reply]
The original version of the article was poorly explained itself. I'm reading it right now and I don't get the point. It says "Some people think everything in the mind can be explained in terms of energy, though mainstream scientists are hesitant to accept this... general things about hormones and neurotransmitters follow". The last part of which could be stripped out, leaving the article with only a single sentence. That's like an uber-stub. 206.126.163.20 (talk) 01:52, 9 May 2008 (UTC)[reply]

Stereographic projection

Stereographic projection example

I need help to make sure that what I wrote at Template:POTD/2008-05-13 actually makes sense. Asked at Talk:Stereographic projection with no response, so trying here. Yes, I know that the article talks about representing spheres with 2-D pictures, and this is kind of the opposite, but it's way down at Stereographic projection#Photography. I mostly just copied-and-pasted from there, but since I didn't really understand this section of the article, I'd like more eyes please. Thanks. howcheng {chat} 21:15, 8 May 2008 (UTC)[reply]

You're close. I think the point opposite actually becomes infinitely enlarged - since the picture has only finite area, it isn't the whole sphere, it's the sphere with a small disc around the opposite point removed (roughly - the picture isn't circular, so it won't be quite a disc). --Tango (talk) 22:20, 8 May 2008 (UTC)[reply]

Nuclear Bomb Powered Rocket Project

I'm intrigued by a project from the 1950s or 1960s for building a spaceship which was powered by a number of nuclear bombs inside itself, going off at regular intervals to propel the vehicle. I wonder why this was even considered, as if one bomb went off, it would set all the others off, probably devastating half the planet in the process. Sure, it would get into space (or bits of it), but it wouldn't be a good way to do it. The fix for this would be to put each bomb inside some sort of material that is resistant to a nuclear blast, but this would just make the bombs go off inside their own casings and nothing would happen. It seems that NASA actually funded this project, and I have heard a rumour (sorry, forgot source) that they are even considering it again now. Is this true and is it even feasible?--ChokinBako (talk) 23:07, 8 May 2008 (UTC)[reply]

Project Orion (nuclear propulsion). First, you don't start the nuclear explosions on Earth, so there's no way for Earth to be "devestated." It would certainly take some radiation from orbital detonations, but if I recall correctly, such an explosion would only have enough radiation to kill ten people (proabalistically speaking, from the increased likelihood of cancer). Once the ship is out of orbit, that would cease to be an issue. As for the explosions destroying the rest of the nukes, this is sovled very easily in the two ways you could do this. If you're firing nukes at the ship from Earth or from an orbital/moon based/whatever platform, then you simply have the explosions take place far enough apart that each nuke is always outside the destructive radius of the previous nuke's blast. If the ship is carrying its own nukes, it simply waits until after each explosion to drop the next one in place. Someguy1221 (talk) 23:45, 8 May 2008 (UTC)[reply]
Ah, I see. So, the idea was that the ship would actually 'let go' of one bomb at a time and let them detonate a safe distance from the ship, but close enough to get the propulsion? The diagrams I have seen in the past just showed a rocket with about ten or twenty nuclear bombs inside it and didn't explain that bit. It seems to make some sense now. Thanks.ChokinBako (talk) 00:54, 9 May 2008 (UTC)[reply]
That's exactly right. The ship shoots the bombs out behind it, then the bomb detonates, and the force of the explosion is picked up by a big plastic plate (the pusher plate) when then imparts the radiant energy as kinetic energy, moving the ship. This YouTube video does a pretty good job of illustrating its pulsing approach right after it gets out of the atmosphere (in the "re-imagining" of the concept by the guy who made the video, he has it taking off with chemical rockets, to avoid fallout problems on Earth). --98.217.8.46 (talk) 02:39, 9 May 2008 (UTC)[reply]
The plan was indeed for the nukes to go off on Earth, as the spaceship was to be far too heavy for a chemical rocket to get it into space. The planners even specified that it would have one of those enormously heavy swiveling barber's chair, just to thumb their noses at the extremely-weight-restricted chemical rocket crowd who eventually won the day at NASA. The effects of atmospheric nuclear explosions were only just becoming known at that time, and when their danger became clear the project was doomed. One of my favorite books is The Starship and the Canoe, a dual biography of Freeman Dyson, who was one of the principal Orion people, and his son George Dyson, who lived in a tree in British Columbia. --Sean 01:03, 9 May 2008 (UTC)[reply]
Oh, I didn't realize that was actually the case. In any event, you could still use the concept as an in-space-only propulsion mechanism, since you can still send up your 400 ton space ship in little pieces, then use the bombs for the heavy work of getting to Alpha Centauri in a meaningful time frame. Someguy1221 (talk) 01:37, 9 May 2008 (UTC)[reply]
Here's a nice government-produced picture of where the bomb would go off relative to the spaceship, and then it would explode, and then push up against the "pusher plate" of the spacecraft, which would be made of a material that would reflect the energy primarily. Following Newton's whatever law, the act of this reflection would exert an equal and opposite force on the spaceship itself, pushing it along. --98.217.8.46 (talk) 02:45, 9 May 2008 (UTC)[reply]
Larry Niven and Jerry Pournelle used this propulsion method in their science fiction novel Footfall. They make the interesting point that for a launch from earth it is the second bomb that is crucial. If the first one fails - well you are still sitting on the deck, have another try. If the third one fails while you are in the air, you have enough time to detonate another. However, if the second one fails, thousands of tons of concrete platform (or whatever you have made the vehicle from) come crashing back to the ground before any corrective action can be taken. SpinningSpark 17:07, 9 May 2008 (UTC)[reply]
The fallout issue is unfortunate, because an Orion engine is twice as efficient in an atmosphere as it is in space: the atmosphere lets you catch more of each bomb's energy. --Carnildo (talk) 21:30, 9 May 2008 (UTC)[reply]

May 9

How many nucleotides are found in all of the exons of the CFTR gene?

How many nucleotides are found in all of the exons of the CFTR gene? —Preceding unsigned comment added by 65.29.80.238 (talk) 01:23, 9 May 2008 (UTC)[reply]

4575 [18] Someguy1221 (talk) 01:32, 9 May 2008 (UTC)[reply]
Well, there are non-protein coding sections within exons 5' and 3', but according to EBI Dbfetch (which is the data base I sometimes use in the lab), there are 4443 nucleotides in the mRNA [19]. Wisdom89 (T / C) 04:57, 9 May 2008 (UTC)[reply]
The mRNA sequence is 4575 long if you end at the last translated codon. The EBI sequence left out the 5' untranslated region, 132 nucleotides long. Someguy1221 (talk) 09:19, 9 May 2008 (UTC)[reply]
Yes, you are correct, and hence my reference to 5 prime non-translated regions. This was just in case the user was possibly interested in just the coding region. Wisdom89 (T / C) 11:16, 9 May 2008 (UTC)[reply]

Voyager Spacecraft

I've read that the Voyager spacecraft are at the edge of the Solar System and are slowing down. Scientists are saying this might be because of some interstellar wind that is pushing against them, making it hard for them to escape the Solar System. Could this not just be the same thing that rockets experience when attempting to reach 'escape velocity' from the Earth (sorry, I forget the correct term)? Could it not be that the Solar System has the same gravitational forces as the Earth, but on a much larger scale?--ChokinBako (talk) 02:32, 9 May 2008 (UTC)[reply]

That's quite true, the solar system (which can be greatly simplified to "the sun", for most purposes like these) does exert gravitational pull on Voyager. However, NASA et al have already accounted for this. What you've read likely means that there's an additional force of uncertain origin at play. — Lomn 02:35, 9 May 2008 (UTC)[reply]
The gravity from the Sun has been slowing Voyager down since it was launched. But it has plenty of velocity to leave the solar system and after a very, very long time reach some other star. Edison (talk) 03:55, 9 May 2008 (UTC)[reply]
I think the questioner may be refrring to the small unexplainned deceleration that has been detected in the Pioneer spacecraft - see our article on the Pioneer anomaly. 199.43.13.101 (talk) 11:05, 9 May 2008 (UTC)[reply]
(There's a great graph somewhere on WP of the probe's speed since its launch. I can't find it right now. If anyone has the link to it, it would be welcome.) 200.127.59.151 (talk) 17:57, 9 May 2008 (UTC)[reply]

Springtime for France

When does spring arrive in France ("arrive" in the phenological definition)? And when does it arrive in England? (although I imagine it arrives around the same time for both) 96.233.8.220 (talk)PrisonersOfLove —Preceding comment was added at 03:04, 9 May 2008 (UTC)[reply]

Would this be what you were looking for Spring (season)? Phenology deals with recording several natural events. It gives dates for a species, based on which next year's date can be predicted, e.g. to within about 55 days (-21, +34) for some flowers. So there is no fixed date. I assume that plants start blooming earlier in the southern part of France than in the northern part, which will be closer to southern England, since both are near the channel (gulf stream influence). But that's speculation. --71.236.23.111 (talk) 18:32, 9 May 2008 (UTC)[reply]

I just need to know when it gets warm and the flowers start blooming and it begins to *feel* like spring in those countries. Maybe someone who's lived there can help? —Preceding unsigned comment added by 96.233.8.220 (talk) 19:53, 9 May 2008 (UTC)[reply]

Varies heavily from year to year. Last year (in southern England), April was the closest thing we got to summer all year. This year April saw snowfall. See The Guardian letters page passim ad nauseam. Algebraist 08:59, 10 May 2008 (UTC)[reply]
France is a geographically diverse country. Marseille is on the Mediterranean. Calais is on the English channel. Alsace is inland. The climates are very different. Google "Marseille climate"[20], "calais climate"[21], and "Alsace climate"[22] -Arch dude (talk) 02:26, 11 May 2008 (UTC)[reply]

what is the term that refers to the earth's surface features?

the word comes from the latin term terra —Preceding unsigned comment added by Amandalsmithlpn (talkcontribs) 03:23, 9 May 2008 (UTC)[reply]

Terrain?--71.236.23.111 (talk) 08:45, 9 May 2008 (UTC)[reply]

Cancer

Is cancer just restricted to mammals and people, or can other living things get cancer? Our article makes no mention of other organisms besides humans getting cancer. I know that some trees can live hundreds of years, and I also know that "...cancer-promoting genetic abnormalities may be randomly acquired through errors in DNA replication, or are inherited, and thus present in all cells from birth.", so it stands to reason that really old trees that have undergone mitosis for ages would be susceptible to cancer (unlike insects who only live for a relatively short time). Thanks for all the info! PS-not specifically asking about old trees, just giving an example. I want to know can ANY living organism get cancer (in a nutshell). --71.98.26.52 (talk) 04:02, 9 May 2008 (UTC)[reply]

The difference between the way that plants and animals grow means that plants generally don't get cancer (because a huge increase in cell growth in a plant will just make a bigger plant, or one with more branches). However, the gall created by a gall wasp is, roughly speaking, a plant cancer. Confusing Manifestation(Say hi!) 04:13, 9 May 2008 (UTC)[reply]
As long as something is made of DNA, mutations are possible. Cancer, as we know it (e.g. tumors and neoplasms), may not be present in every animal or plant but there are other genetic diseases which resemble it. Regards, CycloneNimrodTalk? 08:51, 9 May 2008 (UTC)[reply]

The misleading explanation is removed, thanks for the comments. Dr.Rajarshi (talk) 04:34, 10 May 2008 (UTC)[reply]

Unless I misread, but "uncontrolled cell growth" is not termed metastasis. Wisdom89 (T / C) 11:13, 9 May 2008 (UTC)[reply]
Per Wisdom89, use 'Dr.' Rajarshi's information with caution. The process by which cells become malignant (cancerous) is called carcinogenesis, while the specific steps wherein growth regulation is lost may be called neoplasia. (Note that not all neoplasms are malignant.)
Metastasis meanwhile, is the spread of a malignant tumour from its original site to other locations within the body.
In my experience, the term cancer is used to describe malignant growths in mammals only; uncontrolled growths of cells do happen in other organisms, but usually go by other names. TenOfAllTrades(talk) 13:46, 9 May 2008 (UTC)[reply]
Plants do get cancer, but since plants don't have mobile cells, these cancers can't metastasize. --Carnildo (talk) 21:39, 9 May 2008 (UTC)[reply]
Interesting...thanks for all of the info. So I'm assuming other organisms which have mobile cells (and that live long) can get cancer that matastasizes, such as those sea turtles that live a hundred years? And I'm also assuming that insects, which live a relatively short time, have almost zilch chance of getting cancer? --71.117.40.7 (talk) 13:51, 10 May 2008 (UTC)[reply]

Book-name

What is a book-name?

The term is used in the article Spiny squirrel. Thank you. CBHA (talk) 04:18, 9 May 2008 (UTC)[reply]

Here[23] (scroll down) implies it's a "common name" that isn't in use but written down anyway for description purposes in the hope it catches on, as in: [It is] "desirable that well-chosen names be available... if no popular name be already available, a suitable book name will in time come into use as a common name." Julia Rossi (talk) 10:22, 9 May 2008 (UTC)[reply]

Meaning of a species name

In the article Bearded Barbet, the scientific name of this bird is Lybius dubius.

Why was it called dubius? Thanks, CBHA (talk) 05:04, 9 May 2008 (UTC)[reply]

There are quite a few species with 'dubius' in their name. The term is just what it seems: Latin for the similar word "dubious". My understanding is that species are given this name if the criteria for classifying them as a distinct species or placing them in a specific group is... well, dubious. According to this list of Latin terms used for species classification, the term can also mean "uncharacteristic". Dooky (talk) 09:15, 9 May 2008 (UTC)[reply]

Acute chest pain

What are some causes of acute chest pain that would not found by an echocardiogram or chest x-ray? Mac Davis (talk) 05:52, 9 May 2008 (UTC)[reply]

Chest pain might hold some clues. 71.236.23.111 (talk) 08:39, 9 May 2008 (UTC)[reply]

Just as a forewarning, please read this: Wikipedia:Medical disclaimer. Wisdom89 (T / C) 11:10, 9 May 2008 (UTC)[reply]

Perhaps this can help you [24]. --Taraborn (talk) 14:36, 9 May 2008 (UTC)[reply]

Exothermic reaction

Alkalisation of cellulose with a mixture of ethyl alcohol & sodium hydroxide is an exothermic reaction. Can the heat generated be caluculated in terms of kcals using enthalpies of reactants?59.93.71.174 (talk) 12:11, 9 May 2008 (UTC)[reply]

It could be if you knew the products produced. In this case I suspect you may get a complex mixture including sugars and aldol condensation products, too complex to calculate. Graeme Bartlett (talk) 21:45, 9 May 2008 (UTC)[reply]

CO2 of fans

Is it more environmentally-friendly to have air-con in an office or for everyone to have electric fans blowing all day? 195.60.20.81 (talk) 14:33, 9 May 2008 (UTC)[reply]

It will probably depend on 1. the size of the office space (which gives you the power requirements for the central air), 2. the number of individuals in the office (which tells you how many electric fans there would be), and 3. assumptions about the efficiency of the technology in question (are we talking new, EnergyStar approved air conditioners, or one from decades back?). Not knowing any of those, I would estimate that central air is more energy-efficient under some conditions and individual fans are better for others. Which doesn't say much. --98.217.8.46 (talk) 16:19, 9 May 2008 (UTC)[reply]
: "Environmentally friendly" questions are always a tricky can of worms. What the outcome is depends more on who counts what how than comparable data. That's why I'm not going to drag up study A versus study B, but will give you a couple of factors to consider. First of all, if you get a real scorcher electric fans blowing all day are not going to provide the same level of comfort as an AC unit. They offer comparable performance only in moderate heat. This is particularly true if the heat is accompanied by high air humidity (the rule of thumb says to add 10 to the heat index for high humidity). Running an AC unit will actually dry the air. Airflow patterns for office buildings are usually designed with some degree (ideally a high degree) of care. Airflow patterns from individual fans are haphazard and chaotic. Fans cool by creating high volume air flow. This is a less efficient way of cooling than in the AC which uses a refrigeration cycle to actually remove the heat from the air. Central AC creates most of its noise away from your workers, individual fans running at full tilt can cause considerable noise pollution. Central AC can be kept on at a certain temperature setting overnight. If nighttime temperatures don't drop significantly enough the building will heat up overnight when your individual fans are turned off. This can actually lead to a much higher power consumption when the workers come back in the morning and run their fans to try and cool down the "oven". Such influences depend on individual structures and building materials and are therefore impossible to quantify on a general basis. Central AC can be adjusted following guidelines that take energy saving aspects into consideration. With individual fans you have to rely on the individual attitude of each worker. While we can calculate power consumption rates for a central AC at certain settings, calculating the overall power consumption for individual fans is highly arbitrary. Comparing both systems at maximum levels will not give realistic results. How much of an influence additional losses due to distributed power use for individual fans is I'll not consider now, that in any case would also depend on highly variable local factors. So far for power consumption of the units
Average life of an AC unit and individual fans is comparable at about 12 years. After that time you have only one AC unit to replace which will usually be removed by a professional and may be disposed of in ways with low environmental impact. Choosing the other option you end up with a pile of used fans that are usually disposed of in the ordinary trash. They can have some components that are environmentally taxing. Components that could theoretically be recycled, like e.g.metal components, will not be. Plastic recycling has recently come under fire, ironically from some environmental groups, because of the high energy expense, low efficiency and water use involved. AC units on the other hand often contain refrigerants that contribute significantly to the depletion of the ozone layer and/or are considered a health hazard. Fans with plastic components may emit Plasticizers and while the refrigerant in the AC is contained in a closed system any substance emitted by a fan will be emitted directly into the airflow hitting the worker. This also goes for any other substance detrimental to health that may be contained in the paint (e.g. heavy metals), insulation material, lubricants (hydrocarbon compounds) etc. If this wouldn't make the picture complex enough, comparing the environmental footprint of the manufacture and transport of the units opens a whole new chapter, with results influenced by such diverse factors as distance to the consumer, energy use and various sources of pollution involved in the acquisition of materials and the manufacturing process. Environmental impact of said pollutants and effectiveness of countermeasures. Not to mention packaging. So the answer is: yes or no or 42. ;-) (The above is neither medical nor legal or any other advice except for maybe looking very closely at all the factors before starting to point fingers.) 71.236.23.111 (talk) 17:44, 9 May 2008 (UTC)[reply]

Sensitivity of Taq polymerase to heat

My lab supervisor (I'm an MSc student and she's a PhD student) often seems obsessive about keeping Taq polymerase on ice when it's not in the freezer and minimising the time spent between. I figured that if it still functions after being subject to 95 degrees C, then it ought to be fine for a few minutes at room temperature. The way that I was told do set up a PCR reaction was to insert the DNA samples into those strips of 0.2 ml tubes, and then add the 'master-mix' containing everything else. After three rounds of contaminated PCR reactions, presumably a result of contamination of the pipette tip used to add the master-mix, I decided to avoid the possibility altogether by adding the master-mix to all empty tubes and then adding the samples to the master-mix. Now that I've imaged the products, I see that almost all reactions failed except for a few which gave really light bands and the positive controls which worked fine. It could be that there were too many primers in the outside flanking mix (I'm doing nested PCR)... could it be that the Taq was at room temperature for about 15-20 minutes? ----Seans Potato Business 16:18, 9 May 2008 (UTC)[reply]

I keep Taq on ice while its out, but I've definitely had it in the master mix at room temperature for at least 15 minutes while filling a 96 well plate. I've also always gone mastermix first, then samples, because the mastermix has always been the larger volume, for me, and to avoid a simply excessive use of pipette tips. Have you tried any positive controls during your PCR runs? Someguy1221 (talk) 17:31, 9 May 2008 (UTC)[reply]
Yeah, the postive controls worked. I'm suspecting the samples. I will try it again with a greater quantity of sample and reduced water in the master-mix. ----Seans Potato Business 19:13, 9 May 2008 (UTC)[reply]
I agree with Someguy. I usually keep it on ice at all times, as it's become an automatic habit. However, whenever I do any type of PCR reaction (whether it's mutagenesis or straight up generating amplicons for subcloning), I always make the master mix without the enzyme. I add master mix, template, then lastly the enzyme. However, that being said, if you are getting "contaminated" looking lanes, it's most likely the primers and the annealing temperature. Very rarely does DNA contamination really affect the PCR reaction, at least in my experience. Don't get me wrong, it could, and you should wear gloves and use DNase/RNase free tips and tubes, but, I've never had that sort of problem if such precautions are being taken. Have you tried a gradient PCR? Wisdom89 (T / C) 18:09, 9 May 2008 (UTC)[reply]
Oh, and just a recommendation, regular Taq Pol is good, but I would suggest trying to use Pfu turbo or ultra. Wisdom89 (T / C) 18:10, 9 May 2008 (UTC)[reply]
The contaminations I talked about were in the water controls so it was likely the master-mix which was contaminated. The primers have all been optimised using gradient PCR to find the right annealing temperature and number of cycles. I don't have much say in the reagents we buy, particularly since Red Taq Pol usually works just fine. I'll try using more sample as per above and see how I go. I'll be back on Tuesday to report/whine. ----Seans Potato Business 19:13, 9 May 2008 (UTC)[reply]
Like the others, I always keep Taq on ice or in an ice block when not at -20. Its good lab practice. That said, most types of Taq Pol are pretty temperature resistant. I recall a student in our lab doing some PCR then leaving his Taq on the bench overnight and doing the rest the next day. Those amplified after 16hrs at room temperature worked just as well as those done immediately. If you store your Taq is small enough aliqouts, then some raised temperatures are unlikely to have much effect. However, if you store a large volume of Taq in one tube, you should always keep it cold, because eventually the Taq will begin die. I doubt very much Taq is the issue with your PCR. Sounds like a sample problem to me. Rockpocket 01:12, 10 May 2008 (UTC)[reply]


May 10

Could magnetism be said to warp space-time in a similar way to gravity?

And spare me “jokes” about luminiferous ether. It’s just that there are obvious similarities between the attractive effects of those two forces on distant objects. (Although there seems to be no repulsive action with gravity). So, can magnetism curve space and slow down time like gravity? And if not, how does a magnet ATTRACT a distant object? Myles325a (talk) 00:19, 10 May 2008 (UTC)[reply]

I don't think so. As I understand it, the electromagnetic force (of which magnetism is one aspect) works by an exchange of photons, rather than a curvature of space. However, a magnetic field does involve energy, and I believe energy does warp spacetime (E=mc2 and all that), but that warping isn't what we feel as magnetic force. Physicists are working on theories that unify the various fundamental forces, so it's possible there are theories in which EM works in a similar way to gravity by curving spacetime, but in the standard theories EM is described by quantum field theory (exchange of particles) and gravity is described by general relativity (curvature of space) - trying to describe one in terms of the other is very difficult and no-one's quite managed it yet. --Tango (talk) 00:31, 10 May 2008 (UTC)[reply]
This is not such a crazy idea. See Kaluza–Klein theory. Nothing has really worked though. Quantum mechanics gets in the way. I have to run though. Sorry for the shoddy explanation. — gogobera (talk) 02:00, 10 May 2008 (UTC)[reply]
Magnetism follows an inverse cube law not an inverse square law like gravity. The electric force would therefore be more similar to gravity. Magnetism is similar to frame dragging though. Em3ryguy (talk) 12:57, 10 May 2008 (UTC)[reply]
It follows an inverse square law. It's just that every north pole is paired with a south pole and vice versa. The further you get, the proportionally closer the two poles are, and the more their effects cancel. — DanielLC 23:18, 10 May 2008 (UTC)[reply]

Dogs and cats

I sometimes see dogs and cats licking their privates. Do they ever do this to the point that they, shall we say obtain their jollies? FairmontMN (talk) 01:54, 10 May 2008 (UTC)[reply]

You may find Animal_sexual_behaviour#Autoeroticism_.28masturbation.29 interesting. --Tango (talk) 01:58, 10 May 2008 (UTC)[reply]

Conditioning

If you reward a behavior irregularly, it will condition that behavior very strongly, even when the rewards disappear. If you punish a behavior irregularly, it will tend to prevent that behavior very strongly, even when the punishments disappear. What happens if you both punish and reward a behavior, both irregularly? Will the animal shoot up a convenience store? Black Carrot (talk) 02:53, 10 May 2008 (UTC)[reply]

The idea behind this question is so good I imagine animal experiments must have been done on it, although I don’t know of any off hand. One would think it would involve how intense the drives to obtain a particular reward versus escaping a specific penalty were. In the case of things like food, where the need to obtain it can be absolutely vital, then perhaps a subject will risk much more than if the reward was less important. In human experience, some people will rob a bank at the risk of incurring long jail terms IF the potential reward is a very great deal of money. I suppose we all unconsciously work out negatives and positives for any action we are likely to take. After all, any action involves a cost in terms of effort, so everything we do is weighed up against the scenario of what would happen if we did not do it. Myles325a (talk) 03:08, 10 May 2008 (UTC)[reply]

sauna temperature control

How does the copper tubing temperature sensor in a sauna provide feedback to the electrical control box to maintain the temperature?

Will throwing some water on the copper tubing sensor damage the control box?

Will intermittent water squirted on the copper sensor eventually damage the sensor over a period of time? Jaimegonzo (talk) 03:05, 10 May 2008 (UTC)[reply]

I can't say I know exactly what the “copper tubing temperature sensor in a sauna” is, but I imagine that you might start with Bi-metallic strip#thermostats. Maybe that's entirely not what you're wondering about, though?
About the water… it seems likely to depend on the housing for the control box -- is it water resistant? Does the thermostat rely on electrical conductance at all? I would hazard a guess that in sauna temperatures, if “throwing some water on [it]” doesn't cause problems, then “intermittent water squirted on [it] … over a period of time” won't do much either. Of course, “period of time” is a bit vague. :P  — gogobera (talk) 06:02, 10 May 2008 (UTC)[reply]
If you have a real Finnish sauna it will tolerate water, as throwing water on the stove ("kiuas") is an integral part of sauna. A Finnish kiuas that doesn't tolerate water is a nonsensical concept. Check the stove manufacturer's instructions though; other types of steam rooms from other countries might not be designed for water. If the temperature sensor is inside the stove it will be protected from water. The sensor can also be inside a thermometer on the wall, turning the heating elements on and off based on air temperature rather than stove temperature. Weregerbil (talk) 09:43, 10 May 2008 (UTC)[reply]
The copper tube is likely a thermocouple. (A friend of mine recently cut one like this on a hot tub when he assumed it was a grounding wire). -- Flyguy649 talk 14:31, 10 May 2008 (UTC)[reply]

Rube Goldberg pendulum

In a Rube Goldberg machine I am building, a marble hits 'something' which causes a pendulum on the top of a tower to swing back and forth a few times before it hits another marble which rolls down the tower. I'm having some trouble deciding what must be used to do what I just described. What should that 'something' I mentioned earlier be? --hello, i'm a member | talk to me! 03:29, 10 May 2008 (UTC)[reply]

In truth I have some ideas, it wouldn't hurt if I get some more. --hello, i'm a member | talk to me! 03:30, 10 May 2008 (UTC)[reply]

No ideas but what you said doesn't really make sense to me. The pendulum will swing the greatest distance on the first swing. Thus it will reach the marble on the other side of the swing on the first, not necessarily the second, third, etc. swing. Dismas|(talk) 03:38, 10 May 2008 (UTC)[reply]
True if the pendulum swings in one plane. If the 'something' can provide an offset that causes the pendulum to oscillate in two dimensions, it can oscillate through several perioed before striking the target. Example: The pendulum bob might be held at an angle tothe vertical by a hook. the 'something' strikes the bob at an angle that dislodges it from the hook and imparts momentum that is not in the plane of the top of the pendulum. -Arch dude (talk) 04:07, 10 May 2008 (UTC)[reply]
This is what is so difficult about creating a pendulum to the marble on top: how do I create the nesscesary torque to cause the pendulum to oscillate like such (so that it eventually hits the marble)? --hello, i'm a member | talk to me! 04:21, 10 May 2008 (UTC)[reply]
Have you considered driving the pendulum by using another pendulum? Suspend two pendula from one crossbar such that both pendula have the same period of oscillation. Set one pendulum in motion. The system will be a bit 'leaky', and energy will be transferred from the moving pendulum to the stationary pendulum, driving it to oscillate. (The effect only works if both pendula have the same period.) TenOfAllTrades(talk) 06:00, 10 May 2008 (UTC)[reply]
Addendum: It's a fairly popular physics demo; here's a YouTube video of the principle in action: [25]. TenOfAllTrades(talk) 06:03, 10 May 2008 (UTC)[reply]
For another idea of how to get torque look at Driving wheel. We may have a page that describes it better, but I don't know where. Maybe s.o. else does. Otherwise just look at a model train. 71.236.23.111 (talk) 13:47, 10 May 2008 (UTC)[reply]

What is a "geographic feature"?

I've just created the article geographic feature, but I'm not sure I've defined the class entirely correctly. I took some of the material from other Wikipedia articles, and I don't know if they were correct.

For example, are countries and other administrative divisions geographic features? What about imaginary lines like borders, the Equator, etc.

I said they weren't (except for settlements), but I'm not 100% sure.

Wikipedia isn't consistent on geographic features, landforms, etc., and it isn't clear what is or is not a geographic feature. The article landform implies that landforms are not geographic features.

I couldn't find a definitive treatment of what is or is not one, so that leaves a bunch of things up in the air:

What about national parks?

Nature reserves?

What about orchards?

And then there's the sea floor, and its features, like trenches, submerged reefs, etc.

I look forward to your edits and comments.

The Transhumanist    07:28, 10 May 2008 (UTC)[reply]

Political boundaries and the like are not geographical features. That's why they are not invariably found on geographic (=physical) maps; they are found on political maps. - Nunh-huh 07:41, 10 May 2008 (UTC)[reply]
In the field of GIS, as well as cartography to some degree, a geographic feature is anything spatial you are representing on a map or in a geodatabase. The usual term is just "feature", but it's not uncommon to hear "geographic feature". I would argue that political boundaries and the like are geographic features. The field of geography includes political geography, not just physical geography. Pfly (talk) 07:59, 10 May 2008 (UTC)[reply]
I agree. It seems bizarre to me to say that political boundaries are not geographical features; maybe this is not as well-defined a concept as you seem to think. And while this is not the place to debate it, I don't see what the point is of having an article about geographical features in the first place. If they're things shown on maps, why shouldn't that article say whatever is to be said about them? --Anonymous, 08:30 UTC, edited 10:27, May 10, 2008.
Here is a great method of finding out what is a geographical feature and what isn't. Position yourself in a landscape, and point at anything that you know is there. Is it visible? If so, you've located a geographical feature. If it is a resident house, it is a very minor geographical feature, and a man-made one at that. If it is a tall mountain, it can be considered a major geographical feature. If it is a river, it is also a geographical feature, notable if it is big, not notable if not so big. What does the geography feature? It may feature a border station with customs, barracks and wire. The border control station is a feature - the political border is not. The wall of wire is a feature (major if big, minor if not), but is entirely disjoint from what is defined to be the end of a geographically defined administrative region. Hereupon lies the source of confusion with regards to wether or not a border is a geographical feature. Political boundaries are not geographical features. The wall the Israelis built just recently is, however, a geographical feature that signifies a political boundary.
I don't wish to take to the refdesk what should otherwise be on the article's talkpage, but you aired a question here, and as such it deserves my best answer. Scaller (talk) 12:19, 10 May 2008 (UTC)[reply]
Note that some non-geographic features are often included on geographic maps simply to help us locate the geographic features. State, provincial, and national boundaries and large cites might be included, for example. In the case of a map of California fault lines, it would be useful to know where Los Angeles and San Francisco are located. StuRat (talk) 20:58, 10 May 2008 (UTC)[reply]
I think what Anonymous said about the terms being only loosely defined is right. I know the word "geographic" is often used for physical things only, which would leave political boundaries out. But this usage is not the only one, nor necessary "correct". I just went and picked up a book I have called "Modeling our World: The ESRI Guide to Geodatabase Design". Here's a quote from page 25: "Geographic features are located at or near the surface of the earth. They can occur naturally (rivers, vegetation, and peaks), can be constructions (roads, pipelines, and buildings), and can be subdivisions of land (counties, land parcels, and political divisions)." Pfly (talk) 21:21, 10 May 2008 (UTC)[reply]
Another example is the USGS Geographic Names Information System. Although they mostly use just the word "feature", as in "feature name", "feature class", etc, they do use the term "geographic feature" on their FAQ page: "GNIS public Web site: Directly queries the database for official geographic feature names, their locative attributes, variant names, and other data..." The geographic features in the GNIS database all kinds of non-physical things, like counties, states, etc. Pfly (talk) 21:49, 10 May 2008 (UTC)[reply]

Screech of Chalk on a Blackboard

Has anyone done a Fourier analysis on the screeching sound which can be made by chalk or fingernails on a blackboard. Does anyone know why this sound is so unpleasant. 90.206.167.42 (talk) 10:49, 10 May 2008 (UTC)[reply]

Yes [26], and surprisingly the answer is the middle frequencies make it horrible, not the high frequencies. There is some suggestion that the sound is similar to primordial alarm calls and this is what makes us sit up and take notice. You can take part online in that sites study into what makes horrible sounds horrible. But be ready to cringe before you turn up the volume! SpinningSpark 13:20, 10 May 2008 (UTC)[reply]
And of course, I should have known, Wikipedia really does have an article on everything.SpinningSpark 13:23, 10 May 2008 (UTC)[reply]

determining the sugar content of grapes

hi, can anybody tell me how i would find the sugar content of grapes using the equipment found in a small lab? —Preceding unsigned comment added by 82.45.96.96 (talk) 10:56, 10 May 2008 (UTC)[reply]

Homework alert! Makey melly (talk) 11:10, 10 May 2008 (UTC)[reply]
Have you another looked at Sugar#measuring sugar ? SpinningSpark 14:48, 10 May 2008 (UTC)[reply]

Aircraft as flying disc

Can an aircraft able to transport a human being have the form of a flying disc? 217.168.4.133 (talk) 12:44, 10 May 2008 (UTC)[reply]

Yes it can, take a look at Avrocar, British Rail flying saucer and also Military flying saucers is interesting. There is also Lenticular Reentry Vehicle although this was not intended as human transport. SpinningSpark 13:03, 10 May 2008 (UTC)[reply]

Accelerated Thought.

The concept of gaining command over the matter, that is the entire nature through the process of accelerated thought demands attention. Is there any model which presents the operative dynamics of "accelerated thought"? —Preceding unsigned comment added by Dr.Ramakrishnan (talkcontribs) 13:22, 10 May 2008 (UTC) Dr.Ramakrishnan (talk) 13:31, 10 May 2008 (UTC)[reply]

Psychokinesis and James Randi --71.236.23.111 (talk) 14:19, 10 May 2008 (UTC)[reply]
I don't think this is about telekinesis. I think he means "How can we improve web-based education?". --Heron (talk) 14:24, 10 May 2008 (UTC)[reply]
Perhaps you are asking about the Future Shock effect, which predicted an ever-increasing rate of technological advance. IMHO this effect has failed to materialize. Can you clarify your question please ? StuRat (talk) 20:50, 10 May 2008 (UTC)[reply]

Photon energies in cosmological redshift.

Asked my physics teacher a rather obvious question after being taught about cosmological redshift the other day: If the energy of a photon is given by E=hf (planck's constant * frequency), yet the frequency decreases over time due to the expansion of the universe, then doesn't this violate conservation of energy? Won't photons from distant stars lose energy before they reach us? Where does the energy go?

I've had four thoughts about this:

1. Is planck's constant really a fundamental constant? Could it change related to the age of the universe so as to conserve photon energies?

2. Is the problem here that cosmological redshift (though not redshift due to relative motion, which I understand has energy conserved in a specific reference frame) is a prediction of general relativity, which is incompatible with quantum mechanics (which is where I assume the E=hf equation comes from)?

3. Moving on from the last point, does general relativity actually include conservation of energy? what about other conservation laws?

4. If it does, then is photon energy somehow conserved in a specific reference frame within general relativity, and the apparent loss of energy due to considering the photon from different reference frames?

Or is it something else altogether?

JMatopos (talk) 13:26, 10 May 2008 (UTC)[reply]

Photons do lose energy as the universe expands, and the simplest explanation is that it becomes potential energy of the gravitational field. But in a broader sense, energy actually isn't conserved in general relativity. There's a good (though somewhat technical) discussion here. On your four thoughts: 1. It's possible that the fundamental constants could change with time, but this has been tested and ruled out to pretty high precision. So this can't explain the loss of energy, and anyway it's predicted by general relativity without changing the constants. 2. There's nothing wrong with E = hf, but you could say the lack of gravitation in the standard model is the problem. 3&4. General relativity does predict differential energy conservation (the energy at a point equals the energy that was there before plus the energy that flowed in from elsewhere minus the energy that flowed out). But this has to be taken with a grain of salt because the "energy" here doesn't include gravitational field energy. Energy from the photons gets transferred to the gravitational field despite this law seeming to say it can't. Defining an "energy of the gravitational field" is surprisingly difficult. The page I linked says more. -- BenRG (talk) 14:46, 10 May 2008 (UTC)[reply]

Proving alcohol

Suppose, I'm filming a glass of vodka (or, generally any alcohol). How can I prove to the audience that this is vodka, not water? A litmus paper? --85.132.14.38 (talk) 13:43, 10 May 2008 (UTC)[reply]

Alcohol is flameable, right? Setting fire to it would prove its not water. Would spoil the drink though. —Preceding unsigned comment added by Makey melly (talkcontribs) 13:48, 10 May 2008 (UTC)[reply]
The simplest way isn't exactly scientific, but take a new, sealed bottle of vodka and open it in front of the camera. Any scientific test is just likely to confuse people, and could probably be faked anyway. I mean, if litmus paper worked, you might just end up trying to prove that it's real litmus paper. If you set fire to it, what's to say it's not just a liquid hydrocarbon or some other flammable substance? JMatopos (talk) 13:51, 10 May 2008 (UTC)[reply]
Well what is to prove you didn't fill a vodka bottle with water and then reseal it? Or change the label on the bottle? You could do all three; open a vodka bottle, drink some, set fire to some etc to prove beyond doubt. Or have a witness to say "yes, that is vodka". Makey melly (talk) 13:58, 10 May 2008 (UTC)[reply]
Yet in my mind, any attempt to 'prove' that it's vodka on camera would immediately raise my suspicions that it wasn't. Removing the foil seal on a new bottle of vodka is believable, natural and easy to do. Not that all these tests are a bad idea; I mean if you really wanted to 'prove' it, you could set some on fire, use a pH meter to show it's very mildly acidic and give some to a Russian who swears on his life it tastes like vodka. Seems like a lot of hassle, though. :-/ Besides, if I were watching this video, there's no way you'd prove to me 'beyond doubt' it was alcohol unless you personally gave me a sample to run through an IR spectrometer, and I figure that isn't an option. :) JMatopos (talk) 14:20, 10 May 2008 (UTC)[reply]
Having someone in the audience take a sip sounds like a best idea to me. --Tango (talk) 14:04, 10 May 2008 (UTC)[reply]
Dropping in some oil might work. It would separate out and float to the top in water. Breathalyzers use IR spectroscopy if you want something fancy. 71.236.23.111 (talk) 14:10, 10 May 2008 (UTC)[reply]
Ah, but how does he prove that the bottle of oil actually contains oil? Really, there's essentially no way to conclusively 'prove' the identity of a liquid in a videotape—virtually any conceivable test could be faked through mislabeling the reagents used, tampering with the instruments, swapping liquids during a cut between cameras, or using special effects. At some point, you have to rely on the trust of your audience.
The question here might be, "What property of the liquid is actually important?"
  • To show that it's nontoxic, just drink some.
  • To show that it's not straight water, use it to dissolve an ink stain. Test in advance to find an ink that will run in 40% alcohol but not (very much) in water.
  • To show it's not very viscous, slosh it around a bit.
To demonstrate much more than that, you're into a difficult place. Those three tests will demonstrate that it's a drinkable liquid of comparable viscosity to water with some hydrophobic solvent properties. The things that fit those criteria (especially the "I'm willing to drink it" test) are mostly distilled beverage spirits: vodka, white rum, gin, etc. Is that sufficient for your purposes? TenOfAllTrades(talk) 16:26, 10 May 2008 (UTC)[reply]
Of course one further problem is that people could easily feel the video may be faked. For example, it wouldn't be that hard to design a video that to the untrained eye looks like a continuous shot but in fact contains different shots and you could use one drink for the non-toxic part, one drink for the ink part and one drink for the not viscious part and one final drink for where you actually perform the final test on the alcohol solution (I guess there's a point to proving something is alcohol). Of course, you could add a billowing curtain or something similar in the background to help make this more difficult and you story more plausible. It would also be a lot more difficult, even without a curtain to ensure your video fools an experts. This really depends on a lot of things. If for example you are attempting a magic trick/illusion, some of the suggestions from TOAT or others combined with your assurances to the audience that it is alcohol and no camera tricks were used may be sufficient for many viewers. Obviously not everyone is going to be satisfied, but these sort of things generally require some degree of trust from the audience anyway Nil Einne (talk) 19:58, 10 May 2008 (UTC)[reply]
The problem to me isn't that you can "prove" it, but that it's illustrated to be Vodka -- whether the demo is faked or real, for the purposes of a video it illustrates your point. If it's to record an experiment then you;d be playing it straight for teaching purposes. Otherwise, I'm thinking advertising e.g. with two glasses, one labelled (our brand), and one (other brand) for example. Viewers know it's a setup but they suspend disbelief operating through the filmic "wall". Actors don't exactly scull drinks with litmus paper in them. Otherwise like comments above, you can also discredit your demo. Btw, if using a glass of vodka with the oil drop, it seems appropriate to have a glass of water + oil to compare results. Apols if this is all too beside the point, Julia Rossi (talk) 02:24, 11 May 2008 (UTC)[reply]

aurora borealis from space

Can the aurora borealis be seen from space? Mr Beans Backside (talk) 13:55, 10 May 2008 (UTC)[reply]

Yes; see http://en.wikipedia.org/wiki/Image:Aurora_australis_20050911.jpg and http://en.wikipedia.org/wiki/Image:Aurora_Borealis.jpg - Bear in mind that the first image is a composite and has been enhanced, though - it's not really that bright. JMatopos (talk) 14:01, 10 May 2008 (UTC)[reply]
Yes. You can even see aurora on other planets (see Aurora (astronomy)#On other planets). --Tango (talk) 14:03, 10 May 2008 (UTC)[reply]
Absolutely. Here are links to some NASA images: [27], [28]. POES (Polar Operational Environmental Satellite) observes aurorae at the north and south poles; reports are available here, among other places. TenOfAllTrades(talk) 14:09, 10 May 2008 (UTC)[reply]

Why are magpie's tails so long?

What is the explanation for the length of the magpie's tail. It must be an adaptation to something, but to what? Since both sexes have the same tail length it cannot be sexual selection. 217.42.89.22 (talk) 14:17, 10 May 2008 (UTC)[reply]

I disagree that it can't be sexual selection. Both males and females may choose mates that have a long tail, as that's a sign of good health and genes. StuRat (talk) 15:17, 10 May 2008 (UTC)[reply]
Just a thought: Long tails are good for balancing. This may be a factor here. Since there are long and short tailed species (e.g. cats, monkeys), this is not a very selective trait, but it might cause a new species to split off. 71.236.23.111 (talk) 15:19, 10 May 2008 (UTC)[reply]
Every magpie in my area has a certain green-blue tint to their tails, very much like the plumage of a pigeon (bar then the red/purple). I will therefore present the argument that it can have a sexual purpose. Scaller (talk) 17:39, 10 May 2008 (UTC)[reply]
Edit: See this -> http://upload.wikimedia.org/wikipedia/commons/8/8b/Elster_wikipedia2.jpg Ah, what a beautiful picture. Serves my purpose perfectly, even. Scaller (talk) 17:41, 10 May 2008 (UTC)[reply]
See this research paper on the correlation between tail length and general health in black billed magpies. It looks like they are an honest signal on the overall health of the bird to prospective mates in boths sexes. Multiple elements of the black-billed magpie's tail correlate with variable honest information on quality in different age/sex classes. Lumos3 (talk) 22:19, 10 May 2008 (UTC)[reply]

machine that destroyeds itself

Who invented a machine that destroyed itself on purpose? Mr Beans Backside (talk) 14:44, 10 May 2008 (UTC)[reply]

The Manhattan Project. FairmontMN (talk) 14:46, 10 May 2008 (UTC)[reply]
...or for that matter any explosive, used in war or peace, like blasting caps. I suppose an artist might also create such a thing for "artistic reasons". There are also disposable machines, like a disposable camera. Finally, there is "planned obsolescence", whereby a device intentionally fails after some period of time to make you go out and buy another. StuRat (talk) 15:23, 10 May 2008 (UTC)[reply]
Possibly you are thinking of Jean Tinguely [29].--Eriastrum (talk) 19:32, 10 May 2008 (UTC)[reply]
If with "machine" you are referring to the device commonly known as homo sapiens, all you have to do is to find the name of the inventor. --Cookatoo.ergo.ZooM (talk) 23:22, 10 May 2008 (UTC)[reply]
Someone recently reported in the Risks digest that he had been awakened one night by the carbon monoxide detector in his home beeping every 30 seconds, like the way some devices signal a low battery, but in this case it was displaying Err. He located the manual on the Web and learned that the manufacturer said the device should be replaced after 7 years -- and that they enforced this by having it count 7 years and then automatically stop working and go into this error state. (Wouldn't it be fun to be the company's lawyers if someone died of CO poisoning because this happened and then there was a genuine, undetected CO emergency before they could get to a store to replace the detector?) --Anonymous, 09:00 UTC, May 11, 2008.

airport security system x-rays (part I)

Are the airport security system x-rays the same in strength at those at the dentists? If so, why do the dentists run out of the room when they use the x-rays but the airport staff are next to them all day long? Mr Beans Backside (talk) 14:55, 10 May 2008 (UTC)[reply]

The X-rays at airports are contained within the box that checks your luggage by lead, whereas your dentist has no such protection from the X-rays.
Not sure about the relative strength, but the X-ray machines at the airport are shielded by those strips that fall down on either end, while the one at the dentist's office is not shielded, so that makes a big difference. StuRat (talk) 15:14, 10 May 2008 (UTC)[reply]
As a note, there are different grades of X-rays at airports. Checked baggage undergoes more intense x-raying than carry-on baggage, to the point that undeveloped film in checked baggage can end up getting fogged, whereas film in carry-on luggage will not. --98.217.8.46 (talk) 17:30, 10 May 2008 (UTC)[reply]

airport security system x-rays (part II)

Does the average airport security system x-ray the contents of your stomach? For example if you were hiding something illegal in there would they find it? Mr Beans Backside (talk) 14:56, 10 May 2008 (UTC)[reply]

Only if you jump on the conveyor belt along with your luggage. The thing you walk through is just a metal detector, not an X-ray machine, so it would only detect large metal objects in your stomach. StuRat (talk) 15:05, 10 May 2008 (UTC)[reply]
Oh, thank you. That clears a few things up. Mr Beans Backside (talk) 15:11, 10 May 2008 (UTC)[reply]
The metal detector would not detect anything other than metal in your stomach. You should probably check out this legal disclaimer and this medical disclaimer though, I seriously wouldn't advise trying it. It's dangerous from a legal and medical point of view. Regards, CycloneNimrodTalk? 15:13, 10 May 2008 (UTC)[reply]

rechargeable batteries

Can i use rechargeable batteries in television remotes? The instruction manual says not to use them. What damage would it cause? Mr Beans Backside (talk) 14:58, 10 May 2008 (UTC)[reply]

Generally it is best to follow instructions for the sake of safety, but in this case i'm pretty sure the only difference would be reduced performance in terms of the life of the battery. Regards, CycloneNimrodTalk? 15:08, 10 May 2008 (UTC)[reply]
I think it's more of an issue of the rechargeable batteries not having sufficient voltage to operate the remote, especially after they've been recharged hundreds of times (they get progressively weaker with each recharge). StuRat (talk) 15:09, 10 May 2008 (UTC)[reply]
Yes, the difference is 300mV per cell (NiMH or NiCd rechargeable against Alkaline). So assuming your remote takes two batteries it will start off 0.6V too low and then steadily deteriorate. Couple that with NiMH do not perform at their best in low drain/infrequent use applications you may find you have to take the batteries out to recharge quite often. On the other hand, disposable batteries in a TV remote would typically last a year or two, close to the shelf life of the battery. Another common problem with rechargeables in this area (though possibly not usual for TV remotes) is that any device with a low battery warning calibrated for alkalines is going to go off way too early. In short, I agree with the first answer - follow the instructions. SpinningSpark 17:08, 10 May 2008 (UTC)[reply]

Soil moisture measurement (copied from Misc. desk)

The following is on the misc desk. Doesn't seem to work there, so I copied it here: (don't know how to move stuff)71.236.23.111 (talk) 16:40, 10 May 2008 (UTC) Which method would be more reliable to measure soil moisture: resistance or capacitance? Does different soil type contribute much to the measured resistance/capacitance? Does capacitance method still work at freezing temperatures? I'm working for a automatic watering controller using soil moisture measurements and the accuracy doesn't have to be very high, but I do want it to work reliably between different soil types. --antilivedT | C | G 07:01, 10 May 2008 (UTC)[reply]

Take a look at Frequency domain sensor which seems to give some of the answers to this and Soil moisture#Geophysical methods has links to some other probe types. SpinningSpark 17:22, 10 May 2008 (UTC)[reply]

What's the name of the fish species...

... in which the male is far tinier than the female, and ends up living parasitically inside her vagina (or fishy equivalent)?

thanks, Adambrowne666 (talk) 17:59, 10 May 2008 (UTC)[reply]

Anglerfish. --98.217.8.46 (talk) 18:04, 10 May 2008 (UTC)[reply]
You might note that although the male does become a parasite attached to the female's body, this is not inside her oviduct.--Eriastrum (talk) 19:42, 10 May 2008 (UTC)[reply]
Thanks very much - so am I conflating two species then? - is there a happy, nonparasitic fish that does live in the oviduct? Adambrowne666 (talk) 23:23, 10 May 2008 (UTC)[reply]

Ridiculous genomic databases and their stupid lies

I'm trying to get a gene sequence around its transcription start-site, so I'm using this Japanese website: http://dbtss.hgc.jp/ - I am interested in the TJP1 gene: http://ensembl.genomics.org.cn/Homo_sapiens/geneview?db=core;gene=ENSG00000104067

When, on the Japanese database page, I set the category to Ensemble(ENST), and insert the code ENSG00000104067, I get the message: Sorry, there is no hit data. Your search keyword is 'ENSG00000104067'. Is there an alternative to way? ----Seans Potato Business 20:00, 10 May 2008 (UTC)[reply]

Sorry this may be way off, but would this help? [30] —Preceding unsigned comment added by Lisa4edit (talkcontribs) 22:45, 10 May 2008 (UTC)[reply]
Thanks, Lisa. If I go to the page you linked and click on the "genomic sequence" link on the left panel, it seems to give some potentially-useful information but I'm having trouble figuring out what's being displayed. The 'flanking sequence' form fields seem to have no effect and I can't find the transcription-start site. ----Seans Potato Business 23:09, 10 May 2008 (UTC)[reply]
That would be under "Gene information" and there's a link there for the start, at least I think that's it. --Lisa4edit (talk) 23:38, 10 May 2008 (UTC)[reply]

Seagulls

What was the natural habitat of the seagull before humans started interfering with the environment and encouraged them into towns and cities? Are there any seagulls left that still live as nature intended? --62.136.203.112 (talk) 23:30, 10 May 2008 (UTC)[reply]

We have an article on seagulls, you know. Anyway, as the name might suggest, they're seabirds, typically nesting on seasides. Their natural diet includes fish and other sealife, although they're opportunists and will readily exploit other available food sources. —Ilmari Karonen (talk) 23:50, 10 May 2008 (UTC)[reply]
Beat me to it. : Seagull isn't all the same. Our Gull page lists quite a few species. Some fish, some scavenge, some hunt other birds and eat eggs and some steal the catch from others. Most gulls live in costal areas. Only some species live near humans. (I guess if you eat dead tuna or dead duck it doesn't matter much if it has been in a can or trashcan before, or is "fresh" from the beach. :-) Lisa4edit (talk) 23:58, 10 May 2008 (UTC)[reply]

absolute zero

Does all motion stop at absolute zero —Preceding unsigned comment added by 74.47.169.87 (talk) 01:05, 11 May 2008 (UTC)[reply]

Take a look at Absolute zero, the first couple of lines should answer your question. --Tango (talk) 01:10, 11 May 2008 (UTC)[reply]

Why does this microwave have electric-plug-shaped ventilation holes?

The microwave ventilation holes at the bottom are shaped like a variety of electric plugs.

Microwaves have ventilation holes in the rear; but after closely investigating many microwaves, we noticed that there is consistently inclusion of "ventilation holes" which are exactly shaped and sized like various international electric plugs. Any ideas why this exists? We have exhausted a lot of possible explanations to no avail. There is no electric connectivity to these plug holes, they just go through the sheet metal casing. Ideas? Saket (talk) 04:47, 11 May 2008 (UTC)[reply]

Just a wild guess, but it could be because they used those holes to test whether plugs were shaped correctly during some sort of Q&A analysis. Seems a bit weird, though... Titoxd(?!? - cool stuff) 04:54, 11 May 2008 (UTC)[reply]
Here's a random idea - maybe that rear panel of the microwave is used in several different models of microwaves, some of which have power inputs in that location? That could explain the various power-plug shaped holes. Although it seems odd that the female power junction is on the microwave - usually it's just on the wall outlet... it's just a wild guess. --Bmk (talk) 05:02, 11 May 2008 (UTC)[reply]
Or perhaps it's part of some metal-shaping quality control procedure in the factory? Kind of like the colored patches you can find under cardboard flaps on cereal boxes? I can't think what the purpose would be though... --Bmk (talk) 05:03, 11 May 2008 (UTC)[reply]
Is it a place to "park" the plug while in its box? Julia Rossi (talk) 05:24, 11 May 2008 (UTC)[reply]
We thought of all of these possibilities. We should mention that one of our microwaves (not the one in the photo) only has european-style holes in the ventilation panel EVEN THOUGH it is an American model with a US-style plug. This seems to invalidate many of the above possibilities... Saket (talk) 05:31, 11 May 2008 (UTC)[reply]
The one not-yet-mentioned possibility is that this particular model was designed to konfuze WP Reference Desk editors. :) -- Fullstop (talk) 05:35, 11 May 2008 (UTC)[reply]
Aww, my microwave is boring, no plug holes. How about this: a built-in template for the workers to figure out what country the microwave is going to, so they don't get them mixed up in the factory. The workers don't have to be able to read or anything, it's all done with diagrams. For the exception above, that factory is just wiring North America and Europe-style plugs, so they only need one template to check against. Franamax (talk) 06:43, 11 May 2008 (UTC)[reply]
By my count, there are 9 hole patterns in that pic. Thats covers pretty much every type, not just the North Am + European ones. -- Fullstop (talk) 07:01, 11 May 2008 (UTC)[reply]
Yes, that one came from a factory supplying the whole world. The other one mentioned above at 05:31 is from a factory with less scope. Franamax (talk) 07:04, 11 May 2008 (UTC)[reply]
I think it's for packaging purposes; when the microwave was packaged, the plug was perhaps inserted into the relevant holes and excess cable bunched up with a wire tie thing. ----Seans Potato Business 07:51, 11 May 2008 (UTC)[reply]
Packaging maybe not, but assembly more likely. You could feed the terminal end of a pre-assembled cord through a hole in the panel and then store the plug. That way it doesn't dangle around while you move that sub-assembly to the next step. Process wise feeding the cord through is a process (grip, position, feed, pull) that is quite different from fixing the cord and putting the back panel on (grip, position, fix screws). Either a human or a machine would have to change tools, so it would make sense to do it at 2 separate stations. But who knows. Lisa4edit (talk) 09:15, 11 May 2008 (UTC)[reply]
I vote for packaging, I have unpacked items myself that had the plug stored in this way. SpinningSpark 09:47, 11 May 2008 (UTC)[reply]
See reference at 5:31 to a model of a microwave oven with an American plug and only European holes on the back panel. That only makes sense for assembly if the plug was replaced after assembly (say, as part of the importing process), and makes even less sense for packaging. Or did they just screw up with that model? 128.12.187.10 (talk) 11:34, 11 May 2008 (UTC)[reply]
Well it could still make sense for packaging. UK plugs are horrible blocky things with pins sticking out sideways causing problems getting it into the box. You need somewhere for the pins to go. US plugs on the other hand are slimmer and with the pins pointing along the cable axis. Coiling the cable and tieing may be sufficient. Also (not sure if this point applies), are not some US power cords detachable like IEC power cords are? If it is not captive then it is not such a problem packing, it is put in the box after the main item. SpinningSpark 12:14, 11 May 2008 (UTC)[reply]

Concentrate Juice vs Non-Concentrate

<moved from miscellaneous desk>
How much of a difference is there between fruit juices that are "made from concentrate" and those that are "not made from concentrate"? Suppose if Juice A is made from concentrate with just filtered water and juice concentrate as its ingredient and Juice B is not made from concentrate (like Tropicana), how much of a nutrient difference is there? Is the juice not made from concentrate "healthier" for you? Acceptable (talk) 21:31, 10 May 2008 (UTC)[reply]

Well the effective food miles of from concentrate is way lower so you really shouldn't touch the "not from concentrate" stuff. I have tried blind taste off from concentrate with bits versus not from concentrate and they are indistinguishable but real freshly squeezed of course is quite different. --BozMo talk 08:30, 11 May 2008 (UTC)[reply]
I guess you've seen our articles food miles, concentrate and juice? In the last, sugar's a health issue. Julia Rossi (talk) 10:10, 11 May 2008 (UTC)[reply]

Platinum

<moved from miscellaneous desk>
I have a piece of what apears to be platinum that was melted down in a crucible and was wondering how do I go about finding out what it is?--66.191.106.19 (talk) 08:31, 11 May 2008 (UTC) Probably just as simple as taking it to a jeweler, if that doesn't work, ask said jeweler for any contacts of his. —Preceding unsigned comment added by 121.222.146.142 (talk) 10:32, 11 May 2008 (UTC)[reply]